File:  [Local Repository] / db / baza / okmar12.txt
Revision 1.19: download - view: text, annotated - select for diffs - revision graph
Sat Nov 10 13:27:16 2018 UTC (5 years, 6 months ago) by rubashkin
Branches: MAIN
CVS tags: HEAD
*** empty log message ***

Чемпионат:
Окский марафон - 2012 (Пущино)

URL:
/znatoki/boris/reports/201201Puschino.html

Дата:
28-Jan-2012

Тур:
1 тур. "Общество книголюбов" (Ивантеевка - Москва)

Редактор:
Антон Волосатов (Ивантеевка)

Инфо:
Команда благодарит Константина Сахарова (Ивантеевка) за неоценимую
помощь вопросами, ценными замечаниями, редактурой и моральной поддержкой
во время подготовки тура.

Вопрос 1:
Считается, что ПРОПУСК была написана в конце XVI века. Именно там было
изложено довольно расхожее определение мира. Наш тур только начинается,
поэтому мы пока не можем сказать, ПРОПУСК. Заполните пропуск четырьмя
словами.

Ответ:
Как вам это понравится.

Зачет:
As you like it.

Комментарий:
В пьесе Уильяма Шекспира "Как вам это понравится", написанной
предположительно в 1599 году, встречается знаменитый монолог "Весь мир -
театр...". Мы надеемся, что наш тур вам понравится. :-)

Источник:
   1. http://ru.wikipedia.org/wiki/Как_вам_это_понравится
   2. http://www.russianplanet.ru/filolog/evropa/england/shakespeare/like2.htm

Автор:
Антон Волосатов (Ивантеевка)

Вопрос 2:
Какая фирма в 1997 году первой выпустила DVD-привод с функцией записи?

Ответ:
"Pioneer".

Зачет:
"Пионер".

Комментарий:
Японская фирма "Pioneer" нередко соответствовала своему
"первопроходческому" названию.

Источник:
http://ru.wikipedia.org/wiki/DVD

Автор:
Антон Волосатов (Ивантеевка)

Вопрос 3:
Собираясь лишить Евгения Коновальца АЛЬФЫ, Павел Судоплатов получил
совет использовать слабости жертвы и замаскировал средство под БЕТУ.
Назовите персонажа, сравнившего АЛЬФУ с БЕТОЙ.

Ответ:
Форрест Гамп.

Зачет:
Мать Форреста Гампа.

Комментарий:
Лидер украинских националистов Евгений Коновалец очень любил шоколадные
конфеты, что его и подвело. Персонаж Тома Хэнкса цитировал свою маму:
"Жизнь - как коробка шоколадных конфет: никогда не знаешь, какая начинка
тебе попадется".

Источник:
   1. http://ru.wikipedia.org/wiki/Убийство_Евгения_Коновальца
   2. К/ф "Форрест Гамп".

Автор:
Александр Шевченко (Москва)

Вопрос 4:
[Ведущему: кавычки не озвучивать.]
   Режиссер Пол Томас Андерсон, работая над сценарием к фильму, можно
сказать, находился под НИМ добровольно в течение двух недель, так как
испугался увиденной змеи. С НЕГО в 1992 году началась история "Лицея".
Назовите ЕГО двумя словами.

Ответ:
Домашний арест.

Комментарий:
Режиссер боялся выйти из дома, опасаясь, что змея еще где-то рядом. В
результате сценарий к фильму "Магнолия" был написан за две недели.
Первый альбом группы "Лицей" назывался "Домашний арест".

Источник:
   1. http://www.imdb.com/title/tt0175880/trivia
   2. http://ru.wikipedia.org/wiki/Лицей_(группа)

Автор:
Антон Волосатов (Ивантеевка)

Вопрос 5:
[Ведущему: кавычки не озвучивать.]
   "ОНА" впервые увидела свет почти сто лет назад. ТАКАЯ ОНА была
предметом поиска персонажа с необычным именем. Ответьте одним словом:
какая ТАКАЯ?

Ответ:
Сермяжная.

Комментарий:
Персонаж с необычным именем Васисуалий Лоханкин часто задумывался о
сермяжной правде. Первый выпуск газеты "Правда" датируется 5 мая 1912
года.

Источник:
   1. http://ru.wikipedia.org/wiki/Правда_(газета)
   2. http://lib.ru/ILFPETROV/telenok.txt

Автор:
Антон Волосатов (Ивантеевка)

Вопрос 6:
Внимание, в вопросе есть замена.
   В абсурдистской пьесе под названием "БРЭЙН" читатель узнаёт, что ее
действие происходит в разные времена года. О других обстоятельствах
остается только догадываться. Какие два слова мы заменили словом
"БРЭЙН"?

Ответ:
"Что где".

Зачет:
"Quoi où", "What Where".

Комментарий:
В пьесе Сэмюэля Беккета главный персонаж каждый раз фиксирует, когда
происходит действие. При этом из диалогов крайне затруднительно понять,
что и где происходит. Первую часть названия интеллектуальной игры мы
заменили первой частью названия другой интеллектуальной игры.

Источник:
С. Беккет. В ожидании Годо: Пьесы. - М.: Текст, 2010.

Автор:
Антон Волосатов (Ивантеевка)

Вопрос 7:
В одной телеигре был задан вопрос: "Как называется перевод, выполняемый
непосредственно вслед за произнесением фраз на иностранном языке?". А
кто вел эту игру?

Ответ:
[Мария] Киселева.

Комментарий:
Перевод называется синхронным, а Мария Киселева до роли ведущей "Слабого
звена" была известна по синхронному плаванию.

Источник:
"Слабое звено", выпуск от 15.06.2003 г.

Автор:
Екатерина Бруй (Ивантеевка)

Вопрос 8:
Главный герой пьесы Уильяма Йейтса - находчивый бродяга. Решить одну
проблему ему в итоге удается при помощи камня, которым он отпугивал
псов. А какой предмет в похожей ситуации использовал персонаж
отечественного произведения?

Ответ:
Топор.

Комментарий:
Сюжет пьесы "Горшочек с похлебкой" Йейтса почти полностью аналогичен
русской народной сказке "Каша из топора". Отличие только в предмете, из
которого была "приготовлена" пища.

Источник:
   1. http://lib.ru/POEZIQ/JEJTS/yeats1_3.txt
   2. http://www.lukoshko.net/rus/rush1.shtml

Автор:
Антон Волосатов (Ивантеевка)

Вопрос 9:
   <раздатка>
   А известно ли вам, что он из старообрядцев, да и не то чтоб из
старообрядцев, а просто сектант; у него в роде бегуны бывали, и сам он
еще недавно целых два года в деревне у некоего старца под духовным
началом был.
   </раздатка>
   Перед вами цитата из классического произведения русской литературы.
Догадавшись, из какого именно, ответьте, какое слово мы в ней заменили.

Ответ:
Раскольников.

Комментарий:
Вместо "старообрядцев". Произведение, соответственно, - "Преступление и
наказание".

Источник:
Ф.М. Достоевский. Преступление и наказание.

Автор:
Александр Шевченко (Москва)

Вопрос 10:
По одной из версий, источником для сюжета фильма 2006 года под названием
"ИКС" стал эпизод из жизни дона Хуана. ИКС установлен на станции метро
"Римская". Назовите ИКС словом латинского происхождения.

Ответ:
Фонтан.

Комментарий:
Сюжет фильма "Фонтан" Даррена Аронофски перекликается с путешествием
испанского конкистадора Хуана Понсе де Леона, искавшего источник вечной
юности. На станции московского метрополитена установлен фонтан со
скульптурами Ромула и Рема. Слово "источник" в вопросе - подсказка.

Источник:
   1. http://ru.wikipedia.org/wiki/Понсе_де_Леон,_Хуан
   2. http://ru.wikipedia.org/wiki/Римская_(станция_метро)

Автор:
Антон Волосатов (Ивантеевка)

Вопрос 11:
В статье Википедии о московском рынке "Горбушка" говорится, что рынок в
первые годы своего существования стал сакральным местом для людей,
почуявших запах свободы и алчущих перемен. Автор вопроса сравнил
"Горбушку" с НИМ. Назовите роман, в котором ОН обрушился.

Ответ:
"Парфюмер. История одного убийцы".

Зачет:
"Парфюмер".

Комментарий:
Рынок аудиолюбителей первое время во многом напоминал мост Менял, так
как был площадкой для обмена редкими или зарубежными пластинками. В
романе Патрика Зюскинда мост Менял частично обрушается, унося с собой
жизни нескольких человек.

Источник:
   1. http://ru.wikipedia.org/wiki/Горбушка_(рынок)
   2. http://vishnyakov.pp.ru/lib/library3-3.htm

Автор:
Антон Волосатов (Ивантеевка)

Вопрос 12:
В своем произведении ЭТОТ ЧЕЛОВЕК приводит ошибочно одно из вычислений.
По его результатам выходит почти на двести тысяч меньше заявленного
числа, в чем может убедиться даже ребенок. Назовите ЭТОГО ЧЕЛОВЕКА.

Ответ:
[Даниил] Хармс.

Комментарий:
В детском стихотворении "Миллион" во всех строфах, кроме последней,
вычисления сходятся. Но нетрудно увидеть, что:
   Раз, два, три, четыре,
   и четырежды четыре,
   сто четыре на четыре,
   полтораста на четыре,
   двести тысяч на четыре!
   И еще потом четыре!
   в результате дает 801040, а вовсе не миллион.

Источник:
   1. http://www.stihi-rus.ru/1/harms/19.htm
   2. Вычисления автора вопроса.

Автор:
Антон Волосатов (Ивантеевка)

Вопрос 13:
   <раздатка>
   По дороге Дженнифер [ПРОПУСК 1] Дока вопросами о будущем, и Доку
приходится ее [ПРОПУСК 2].
   </раздатка>
   Заполните любой из пропусков в описании киноэпизода.

Ответ:
"... засыпает...", "... усыпить".

Источник:
http://ru.wikipedia.org/wiki/Назад_в_будущее_2

Автор:
Константин Сахаров (Ивантеевка)

Вопрос 14:
В стихотворении без рифмы Вадима СаханЕнко автор предлагает
альтернативное название для одной иностранной фирмы в России. В одной из
строк стихотворения встречается семь одинаковых гласных букв подряд.
Назовите эту фирму.

Ответ:
"Audi".

Комментарий:
Полностью стихотворение-пирожок, обыгрывающее логотип этой фирмы, звучит
так:
   а фирма ауди в россии
   для простоты и красоты
   должна не ауди назваться
   а ооо оооо

Источник:
http://vkontakte.ru/topic-28142790_24812059

Автор:
Антон Волосатов (Ивантеевка)

Вопрос 15:
Максим ТрефАн пишет, что тезка известного ЧЕЛОВЕКА вполне может быть
мистификацией, так как 23 июня в Испании отмечается день его святого.
Назовите имя и фамилию того, кто впервые описал учение этого тезки.

Ответ:
[Карлос] Кастанеда.

Комментарий:
Источник цитирует первую книгу "Учения Дона Хуана": "Мои заметки о
встречах с доном Хуаном начинаются 23 июня 1961 года. С этого дня
началось мое обучение". Возможно, это просто совпадение. Другой дон Хуан
уже известен вам из десятого вопроса.

Источник:
   1. http://fa.livejournal.com/1362892.html
   2. http://lib.ru/KASTANEDA/kastan1.txt
   3. http://ru.wikipedia.org/wiki/Кастанеда,_Карлос

Автор:
Антон Волосатов (Ивантеевка)

Вопрос 16:
(pic: 20120978.jpg)
   Перед вами ИКС монеты, выполненной в виде ИГРЕКА КСИ. Назовите ИКС и
КСИ в любом порядке.

Ответ:
Орёл, Рёло.

Комментарий:
На раздаточном материале изображена монета, повернутая орлом. Сама
монета отчеканена в форме правильного многоугольника Рёло - кривой,
состоящей из дуг окружностей одного и того же радиуса.

Источник:
   1. http://www.worldofcoins.eu/forum/index.php?topic=9903.15
   2. http://ru.wikipedia.org/wiki/Многоугольник_Рёло

Автор:
Антон Волосатов (Ивантеевка)

Вопрос 17:
Дуплет.
   Внимание, в дуплете присутствуют замены, связанные одной логикой.
   1. По словам автора, "ЧИНОВНИКИ" появились как поток сознания и
символизировали советских коллег. А зарубежных коллег ОН назвал
"заморскими богами". Назовите ЕГО.
   2. В песне середины 80-х годов ОН воспевает удобный график работы
ОВЧИННИКА. Назовите ЕГО.

Ответ:
   1. [Вячеслав] Бутусов.
   2. [Виктор] Цой.

Комментарий:
Словом "ЧИНОВНИКИ" было заменено название известной песни "Синоптики".
Анаграмма слова "синоптик" - "истопник". Виктор Цой пел: "Я хочу быть
кочегаром, кочегаром, кочегаром, работать сутки через трое, через трое,
через трое...".

Источник:
   1. http://ru.wikipedia.org/wiki/Синоптики_(песня)
   2. http://www.amdm.ru/akkordi/kino/7716/a_hochy_bit_kochegarom/

Автор:
Антон Волосатов (Ивантеевка)

Вопрос 18:
Рассказывают, что в мае 1944 года в одном из берлинских баров произошла
чудовищная драка между немецкими офицерами. Один из присутствующих в
баре летчиков послал сидящим за соседним столом горку тарелок, сложенных
в шахматном порядке. Невинная на первый взгляд шутка обыгрывала одну из
самых болезненных "специфических" проблем оскорбленных ею офицеров. А
кому конкретно была адресована эта шутка?

Ответ:
Экипажу танка "Тигр".

Зачет:
Танкистам.

Комментарий:
Ходовая часть танка "Тигр" отличалась сложностью конструкции, излишней
"паркетностью" и слабой ремонтопригодностью. Так, замена весьма часто
выходивших из строя катков четвертого, "внутреннего", ряда занимала
около 20 часов.

Источник:
   1. http://koshkin-lib.livejournal.com/5393.html
   2. http://armor.kiev.ua/Tanks/WWII/PzVIH/pzVI.php?page=1
   3. http://ru.wikipedia.org/wiki/Тигр_(танк)

Автор:
Вадим Штанников (Королёв - Калуга)

Вопрос 19:
Продолжим тему.
   Японская шутка утверждает, что на свете есть три самые большие и
бесполезные вещи: ПЕРВЫЕ из них до сих пор можно увидеть в северной
Африке, ВТОРУЮ - в Китае. А вот увидеть своими глазами ТРЕТИЙ (или то,
что от него осталось) вы вряд ли сможете. В 2010 году ТРЕТИЙ стал
космическим. Назовите ТРЕТИЙ точно.

Ответ:
Линкор Ямато.

Зачет:
Крейсер Ямато.

Комментарий:
"На свете есть три самые большие и бесполезные вещи: египетские
пирамиды, Великая Китайская стена и линкор Ямато". Линкор Ямато был
уничтожен американской палубной авиацией 7 апреля 1945 года во время
операции Тен-ити-го; доступ к его останкам на дне Восточно-Китайского
моря закрыт по религиозным причинам. В 2010 году на экраны вышел фильм
режиссера Такаси Ямадзаки "2199: Космическая одиссея" (в оригинале -
"Space Battleship Yamato"), в котором фигурирует космический линкор
Ямато - поднятый со дна моря и переоборудованный для космических полетов
корабль времен Второй мировой войны.

Источник:
   1. http://www.battleships.spb.ru/OldMK/18/index.html
   2. http://fan.lib.ru/c/chekmarew_w_a/19453.shtml
   3. http://www.nevstol.ru/cat-68.html
   4. http://ru.wikipedia.org/wiki/Ямато_(линкор)
   5. http://ru.wikipedia.org/wiki/2199:_Космическая_одиссея

Автор:
Вадим Штанников (Королёв - Калуга)

Вопрос 20:
Внимание, блиц "не для всех"!
   1. Василий Ключевский утверждал, что ЭТО проходило в обстановке
равнодушия и пренебрежения, которые впоследствии развили
подозрительность и недоверие к людям. Назовите ЭТО двумя словами.
   2. В логотипе одного современного телеканала использованы серп и
молот. Воспроизведите название этого телеканала одним словом.
   3. Интернет-пользователь Asher [Эшер] пишет, что первое братоубийство
на самом деле было ЭТИМ, так как о зависти Каина в Библии ничего не
сказано. Назовите ЭТО одним словом.

Ответ:
   1. Иваново детство.
   2. Ностальгия.
   3. Жертвоприношение.

Зачет:
   1. Детство Ивана [Грозного].

Комментарий:
В ответах содержатся названия фильмов режиссера Андрея Тарковского,
которые нередко относят к категории "Кино не для всех".

Источник:
   1. http://www.hrono.ru/libris/lib_k/klyuch30.php
   2. http://www.nostalgiatv.ru/
   3. http://www.asher.ru/library/religen/vbible

Автор:
Антон Волосатов (Ивантеевка)

Вопрос 21:
   <раздатка>
   Искусство образного выражения мысли в слове, словесное художественное
творчество.
   &nbsp;
   Существует своего рода легенда,
   что перед расстрелом он увидел,
   как над головами солдат поднимается
   солнце. И тогда он произнес:
   "А все-таки восходит солнце...".
   Возможно, это было началом стихотворения.
   </раздатка>
   Перед вами цитаты из двух разных ИХ. Второе из НИХ в 1959 году было
посвящено Федерико Гарсиа Лорка. Назовите ЕГО двумя словами.

Ответ:
Определение поэзии.

Комментарий:
Первой приведена цитата из словарного определения поэзии, а второй - из
стихотворения Иосифа Бродского "Определение поэзии", в котором он
описывает последние минуты жизни испанского поэта.

Источник:
   1. http://dic.academic.ru/dic.nsf/ushakov/966082/
   2. http://www.world-art.ru/lyric/lyric.php?id=7343
   3. http://ru-lorca.livejournal.com/105388.html

Автор:
Антон Волосатов (Ивантеевка)

Вопрос 22:
(pic: 20120979.jpg)
   Этот вопрос требует хорошей раскрутки.
   Автор вопроса назвал возничего именем персонажа произведения 1960-х
годов. По немногочисленным данным о нем можно сказать, что управление
людскими массами давалось ему легко. Назовите это произведение.

Ответ:
"Трудно быть богом".

Комментарий:
Речь идет о Ваге Колесе - предводителе разбойников в Арканаре из романа
братьев Стругацких "Трудно быть богом". Wagon Wheels - можно сказать,
перевод этого имени на английский. "Давалось ему легко" - подсказка.

Источник:
   1. http://lib.ru/STRUGACKIE/be_god.txt
   2. http://www.abstrugatskie.ru/
   3. Острый ум автора вопроса.

Автор:
Антон Волосатов (Ивантеевка)

Вопрос 23:
В 2005 году команда из Финикса переехала в другой пригород и
переименовалась, а на старом месте возник новый клуб с тем же названием.
Какой город мы заменили на "Финикс"?

Ответ:
Воскресенск.

Комментарий:
Хоккейный "Химик" воскрес, как Феникс из пепла, а старый теперь известен
как "Атлант" (Мытищи).

Источник:
http://ru.wikipedia.org/wiki/Химик_(хоккейный_клуб,_Воскресенск)

Автор:
Константин Сахаров (Ивантеевка)

Вопрос 24:
Героиня фантастического романа Александра Казанцева не могла покинуть
больничную палату, но в то же время очень хотела выступить с
танцевальным номером. Ей это удалось, однако конфуз произошел, когда
один из зрителей СДЕЛАЛ ЭТО. Напишите два слова, начинающиеся на одну и
ту же букву, которые мы заменили на "СДЕЛАЛ ЭТО".

Ответ:
Бросил букет.

Комментарий:
Голографическое изображение танцовщицы, остававшейся в своей палате,
транслировалось на сцену, и зрители думали, что видят ее во плоти, до
тех пор, пока один из зрителей не бросил букет, пролетевший сквозь нее.
А наше сегодняшнее выступление на сцене окончено, спасибо всем за
внимание!

Источник:
А. Казанцев. Сильнее времени.

Автор:
Александр Шевченко (Москва)

Тур:
2 тур. "Пожарные машинки" (Москва)

Вопрос 1:
Внимание, вопрос простой!
   Это слово предваряет предупреждение в названии фильма 1992 года. А
еще это - название высочайшей вершины массива Боржава. Назовите это
слово.

Ответ:
Стой.

Комментарий:
Мы предупреждали, что вопрос про Стой.

Источник:
   1. http://www.kinopoisk.ru/level/1/film/9556/
   2. http://ru.wikipedia.org/wiki/Стой

Автор:
Денис Никитенко (Архангельск - Москва)

Вопрос 2:
Внимание, в вопросе есть замены.
   В анекдоте дирижер ругает музыкантов и говорит, что во время
исполнения в оркестр кидались гнилыми помидорами. "Но ведь были и
аплодисменты!" - говорит скрипач. "Да, если случалось ОНО", - напоминает
дирижер. "ОНО" - название психологической программы по достижению
намеченных планов. Назовите ЕГО тремя словами.

Ответ:
Попадание в цель.

Зачет:
По смыслу из трех слов.

Источник:
   1. Анекдот.
   2. http://www.vistara.ru/target/

Автор:
Денис Никитенко (Архангельск - Москва)

Вопрос 3:
На рекламе модного вида отдыха автор вопроса заметил сложносоставное
слово. Это же слово можно получить, заменив в названии популярного
движения начала XX века одну букву на три. Назовите эту букву.

Ответ:
У.

Комментарий:
Речь идет о фототуризме. Слово "фототуризм" можно получить, заменив "у"
на "ото" в слове "футуризм".

Источник:
   1. Наблюдения автора вопроса.
   2. http://slova.org.ru/n/futurizm/

Автор:
Денис Никитенко (Архангельск - Москва)

Вопрос 4:
Внимание, в вопросе есть замены.
   В статье о пьесах Бернарда Шоу Светлана Андреева пишет: "Одна из
новаций Шоу, привлекающая внимание литературоведов и театроведов, -
ШИЛЛЕРЫ, занимающие в его пьесах не меньше места, чем ВОЗРАЖЕНИЯ...
ШИЛЛЕРЫ - полноправный элемент драматургической поэтики, фокусирующий
... ситуацию". Догадавшись, что мы заменили на "ШИЛЛЕРЫ", ответьте, что
мы заменили на "ВОЗРАЖЕНИЯ".

Ответ:
Реплики.

Комментарий:
С латинского "replico" - "возражаю". На "ШИЛЛЕРЫ", соответственно,
заменено слово "РЕМАРКИ".

Источник:
   1. http://e-filolog.ru/zarubezhnaja-literatura/35-zarubezhnaja-literatura/73-problems-of-poetics-of-english-qnew-dramaq-in-the-domestic-and-foreign-criticism-plays-jb-shaw-and-j-galsworthy.html
   2. http://ru.wikipedia.org/wiki/Реплика

Автор:
Денис Никитенко (Архангельск - Москва)

Вопрос 5:
Прослушайте четверостишие из стихотворения Д. Быкова, в котором автор
жалуется на свою судьбу:
   За всё платить - моя прерогатива.
   Мой прототип - АЛЬФА.
   А ежели судьба мне чем платила,
   То лучше бы она была должна.
   Предание гласит, что АЛЬФА была захвачена в морском сражении у
Свиного острова. Какие два слова мы заменили на АЛЬФУ?

Ответ:
Персидская княжна.

Источник:
   1. http://shkolazhizni.ru/archive/0/n-19534/
   2. http://www.100stix12.ru/publ/13-1-0-2272

Автор:
Денис Никитенко (Архангельск - Москва)

Вопрос 6:
Восстановите двумя словами, одно из которых - глагол, пропуски в
заглавии новостной статьи на "Ленте.ру": "[ДВА СЛОВА ПРОПУЩЕНО] не
называть полицейских господами".

Ответ:
"Нургалиев разрешил...".

Источник:
http://lenta.ru/news/2011/02/28/mastercop/

Автор:
Денис Никитенко (Архангельск - Москва)

Вопрос 7:
В статье в "Новой газете" вымышленный журналист Филипп Шариков
иронизирует над прошедшим 23-24 сентября 2011 года съездом партии
"Единая Россия". В частности, первый тост он предлагает выпить за ВВП, а
второй - ЗА ТЕХ, КТО В МОРЕ. Какие три буквы мы заменили на "ТЕХ, КТО В
МОРЕ"?

Ответ:
ДАМ.

Источник:
http://www.novayagazeta.ru/politics/48749.html

Автор:
Денис Никитенко (Архангельск - Москва)

Вопрос 8:
В вопросе есть замена.
   В эфире программы украинского телевидения "Большая политика" 12 марта
2011 года произошла драка между двумя знаменитостями - поэтом Олесем
Бузиной и художником Сергеем Поярковым. Запись в блоге, посвященная
этому инциденту, называется "Драка ЕЕ и ЕГО". В экранизации 1994 года
ЕГО сыграл Вадим Захарченко, а в экранизации 2005 года - Роман Карцев.
Надеемся, что знатоки смогут понять автора: какие два слова мы заменили
на "ОН"?

Ответ:
Киевский дядька.

Зачет:
Киевский дядя.

Комментарий:
Пост назывался "драка Бузины и киевского дядьки", а товарищ Поплавский,
которого сыграли Захарченко и Карцев, - дядя Берлиоза из Киева.

Источник:
   1. http://vadimb.livejournal.com/1137454.html
   2. http://ru.wikipedia.org/wiki/Мастер_и_Маргарита_(фильм,_1994)
   3. http://ru.wikipedia.org/wiki/Мастер_и_Маргарита_(телесериал)

Автор:
Денис Никитенко (Архангельск - Москва)

Вопрос 9:
[Ведущему: кавычки голосом не выделять.]
   В книге Кирилла Разлогова "Мировое кино. История искусства экрана"
сеанс в кинотеатре, где луч света, падающий из аппаратной, переносит на
находящийся перед зрителями экран запечатленные на пленке изображения,
сравнивается с объектом в одном "Государстве". Назовите этот объект
двумя словами, начинающимися на одну и ту же букву.

Ответ:
Пещера Платона.

Зачет:
Платоновская пещера.

Источник:
Кирилл Разлогов. Мировое кино. История искусства экрана. - М.: Эксмо,
2011. - С. 9.

Автор:
Айдар Бекчинтаев (Москва)

Вопрос 10:
Восстановите пропуски в перожке:
   рассвет стою на склоне [ПРОПУСК 1]
   вершину закрывает мгла
   там впереди во мгле [ПРОПУСК 2]
   со скуки водят хоровод
   В "Ответах mail.ru" второй пропуск - и символ вечности, и символ
российской сборной по футболу. Восстановите пропуски.

Ответ:
"... Фудзи...", "... улитки".

Источник:
   1. http://www.perashki.ru/piro/25442/
   2. http://otvet.mail.ru/question/43821713/

Автор:
Денис Никитенко (Архангельск - Москва)

Вопрос 11:
Внимание, в вопросе есть замены.
   Персонаж сериала "Клиника" доктор Персиваль Кокс, узнав, что
пациентка отказывается от определенной процедуры, произносит: "Пусть
этот Иисус будет мне спасителем, что я от всего этого устал!". Какие два
слова мы заменили в этой цитате?

Ответ:
Иегова, свидетелем.

Зачет:
Иегова, свидетель.

Комментарий:
Пациентка является Свидетелем Иеговы и отказывается от переливания
крови.

Источник:
Сериал "Клиника".

Автор:
Денис Никитенко (Архангельск - Москва)

Вопрос 12:
Внимание, в вопросе есть замена.
   Поль Эрвье, говоря о своей встрече с д'ИКСом [дэ иксом], упоминает
Эфесский храм. А один из вариантов Вики сообщает, что ИК чаще всего
составляет от 1/6 до 1/4 ставки. Какие четыре буквы мы заменили на ИК?

Ответ:
Анте.

Комментарий:
Пол Эрвье, встретив Дантеса, сказал себе: "Вот тот, кто принес смерть
Пушкину, а Пушкин даровал ему бессмертие, точно так же, как Эфесский
храм - человеку, который его сжег". Анте в покере - принудительная
ставка до начала раздачи.

Источник:
   1. http://www.hronist.com/2010/02/5.html
   2. http://www.poker-wiki.ru/poker/Анте

Автор:
Денис Никитенко (Архангельск - Москва)

Вопрос 13:
Склонный к оппозиции Дмитрий Быков назвал проекты действующей власти
мероприятиями. Какие две буквы мы пропустили в предыдущем предложении?

Ответ:
хи.

Комментарий:
Химероприятиями.

Источник:
http://ru-bykov.livejournal.com/1222232.html

Автор:
Денис Никитенко (Архангельск - Москва)

Вопрос 14:
В сериале "Империя под ударом" о министре Плеве эстонцы говорят, что он
иуда, предатель и еще Бог знает кто. "ИКС, вот это о нем". Террорист
Егор Созонов говорил, что полагает свою душу на алтарь революции, а
остальные эсеры считают его мучеником и дают кличку ИГРЕК.
Парадоксально, что планы эсеров удаются, и Созонов убивает Плеве, а вот
в более древней истории умирал как раз ИГРЕК. Какое имя мы заменили на
ИГРЕК?

Ответ:
Авель.

Источник:
Сериал "Империя под ударом", серия "Авель и Каин".

Автор:
Денис Никитенко (Архангельск - Москва)

Вопрос 15:
Внимание, в вопросе есть замены.
   Герой Иэна Бэнкса говорит, что один из аспектов его социальной
деятельности - давать деньги на ТАКИЕ дела СЯКИХ организаций. ТАКАЯ и
СЯКАЯ - это реки на Камчатке. Отличаются они, в том числе и тем, как
впадают в водотоки. Напишите в правильном порядке, что мы заменили на
"ТАКИЕ" и "СЯКИЕ".

Ответ:
Правые, левые.

Комментарий:
Герой Бэнкса жертвует деньги на правые дела "левых" организаций. Реки
Правая и Левая впадают с двух сторон в Вахвину и Еловку соответственно.

Источник:
   1. И. Бэнкс. Улица отчаяния.
   2. http://ru.wikipedia.org/wiki/Вахвина_Левая
   3. http://ru.wikipedia.org/wiki/Еловка_(приток_Камчатки)

Автор:
Денис Никитенко (Архангельск - Москва)

Вопрос 16:
Иэн Бэнкс в своей книге рассказывает о ветке метро в Глазго. После того
как кожаные сидения в вагонах заменили на пластиковые оранжевые, эту
ветку в народе прозвали так же, как назвал свое произведение уроженец
Манчестера. Назовите этого человека.

Ответ:
Энтони Бёрджесс.

Комментарий:
Ветка метро - кольцевая, ее стали называть "Заводной апельсин".

Источник:
   1. Иэн Бэнкс. Улица отчаяния.
   2. http://ru.wikipedia.org/wiki/Бёрджесс,_Энтони

Автор:
Денис Никитенко (Архангельск - Москва)

Вопрос 17:
Статья о человеке, родившемся в 1925 году, и его преступлениях
называется "Раз-раздеватель" [раз тире раздеватель]. Догадавшись, что мы
заменили на "раздеватель", назовите того, о ком идет речь в вопросе.

Ответ:
Пол Пот.

Зачет:
Салот Сар.

Комментарий:
Статья называлась "Пот-потрошитель". Джек Раздеватель - прозвище, данное
неустановленному серийному убийце, совершившему в 1960-х годах ряд
преступлений (так называемые "Голые убийства", или "Хаммерсмитские
убийства") в Лондоне. Прозвище дано по аналогии с Джеком Потрошителем.

Источник:
   1. http://www.profile.ru/items/?item=4545
   2. http://ru.wikipedia.org/wiki/Пол_Пот
   3. http://ru.wikipedia.org/wiki/Джек_Раздеватель

Автор:
Денис Никитенко (Архангельск - Москва)

Вопрос 18:
В журнале "Andy Warhol's Interview" кроме интервью публикуется еще масса
информации. Однако тематику издатели пытаются сохранить во всём. Так,
раздел о ярких и, как правило, молодых женщинах называется так же, как
отечественный фильм 1989 года. Как называется этот раздел?

Ответ:
"Interдевочка".

Зачет:
"Интердевочка".

Комментарий:
Слово "интервью" и его составные части издатели используют постоянно.

Источник:
"Andy Warhol's Interview", N 1, декабрь 2011 г. - январь 2012 г.

Автор:
???

Вопрос 19:
Корней Чуковский в работе "Нат Пинкертон" 1908 года предложил
классифицировать кинематограф на несколько групп: фантастические
произведения, комические, трагические и ТАКИЕ. Христианская церковь
долгое время считала ТАКИМИ шахматы. Какое прилагательное мы заменили
словом "ТАКИЕ"?

Ответ:
Азартные.

Источник:
Кирилл Разлогов. Мировое кино. История искусства экрана. - М.: Эксмо,
2011. - С. 27.

Автор:
Айдар Бекчинтаев (Москва)

Вопрос 20:
В описании создания фильма "Великолепная семерка", снятого по картине
Акиры Куросавы "Семь самураев", который в свою очередь представлял собой
вариант американского вестерна, упоминается "своеобразная
транскультурная ОНА". Назовите ЕЕ тремя, пятью или девятью символами.

Ответ:
Рокировка.

Зачет:
0-0, 0-0-0.

Источник:
Кирилл Разлогов. Мировое кино. История искусства экрана. - М.: Эксмо,
2011. - С. 370.

Автор:
Айдар Бекчинтаев (Москва)

Вопрос 21:
В первом российском художественном фильме 1908 года мотив известного
убийства объясняется обнаружением главным героем подложного письма,
начинающегося словами "Милый принц Гасан...". Назовите имя главного
героя этого фильма.

Ответ:
Стенька.

Зачет:
Степан.

Комментарий:
Фильм поставлен по песне "Из-за острова на стрежень...", а письмо
подкинули соратники Разина, чтобы избавиться от княжны, доставлявшей им
неудобства.

Источник:
Кирилл Разлогов. Мировое кино. История искусства экрана. - М.: Эксмо,
2011. - С. 60.

Автор:
Айдар Бекчинтаев (Москва)

Вопрос 22:
В своем письме Ольге Книппер-Чеховой литературный критик Мария Каллаш
дает негативную рецензию современному ей фильму, в котором допускаются
такие несообразности, как ИКС, который Анна Каренина приносит своему
сыну. На аукционе Кристи самый дорогой ИКС, проданный за 90 тысяч
долларов, был сделан из мохера. Назовите ИКС двумя словами.

Ответ:
Плюшевый мишка.

Зачет:
Медвежонок Тедди.

Источник:
Кирилл Разлогов. Мировое кино. История искусства экрана. - М.: Эксмо,
2011. - С. 64.

Автор:
Айдар Бекчинтаев (Москва)

Вопрос 23:
В фильме "Монахини в бегах" два жулика, переодевшись, скрываются в
женском монастыре. Однако подозревать их начинают только после того, как
замечают, что ОНО регулярно меняет положение. ОНО, сделанное из
вращающихся цилиндров неправильной формы, делает невозможными любые
попытки встать на НЕГО ногами. Назовите ЕГО двумя или тремя словами.

Ответ:
Сидения для унитаза.

Зачет:
Сидение унитаза, унитазное сидение.

Источник:
   1. Фильм "Монахини в бегах".
   2. http://www.dezinfo.net/foto/37172-porazitelno-do-chego-nashi-predki-dodumalis.html

Автор:
Денис Никитенко (Архангельск - Москва)

Вопрос 24:
Внимание, в вопросе есть замена.
   С самого начала XX века и кинематографисты, и основатель психоанализа
Зигмунд Фрейд, каждый со своей стороны, исследовали значение иллюзий,
сновидений и галлюцинаций. В связи с этим кино нередко называли
рубайатом для бедных. Какое слово мы заменили в вопросе?

Ответ:
Диван.

Зачет:
Кушетка.

Комментарий:
Беседы с личным, дорогим психоаналитиком по традиции ведутся на
диване/кушетке. Рубайат, как и диван, - это сборник восточных
стихотворений.

Источник:
Кирилл Разлогов. Мировое кино. История искусства экрана. - М.: Эксмо,
2011. - С. 25.

Автор:
Айдар Бекчинтаев (Москва)

Тур:
3 тур. "Synapse" (Москва)

Вопрос 1:
Знакомая автора вопроса во время метели представляла, что находится
внутри НЕГО. А герою "Калевалы" старцу Вяйнямёйнену однажды даже
пришлось завязать ЕГО в узел. Назовите ЕГО.

Ответ:
Яйцо.

Комментарий:
Во время метели всё было совершенно белым. А "завязать яйцо в узел, так
чтобы узел не был виден" Вяйнямёйнена попросила красавица Похъёлы.

Источник:
Элиас Лённрот. Калевала. Руна 8.

Автор:
Максим Кудрин

Вопрос 2:
Строительство первого в Петрограде ИКСА завершилось в 1920 году. ИКС
располагался в помещениях бывших бань на Васильевском острове. В 1942
году ИКС был запущен в Колпино на термическом участке цеха N 3 Ижорского
завода. В настоящее время в России имеется 15 ИКСОВ в 12 городах. Самый
большой ИКС находится в Москве и занимает площадь 210 га. Назовите ИКС.

Ответ:
Крематорий.

Источник:
http://ru.wikipedia.org/wiki/Крематорий

Автор:
Максим Кудрин

Вопрос 3:
По мнению шизофреников, самолет, карандаш и ботинок объединяет
возможность СДЕЛАТЬ ЭТО. Герою песни группы "Anacondaz" СДЕЛАТЬ ЭТО
мешал недуг, никак не связанный с шизофренией. Какие слова мы заменили
на "СДЕЛАТЬ ЭТО"?

Ответ:
Оставить след.

Комментарий:
Оставить след в истории мешало плоскостопие.

Источник:
   1. http://www.psy.msu.ru/science/public/zeigarnik/24.html
   2. http://www.anacondaz.tk/?page_id=1036

Автор:
Надежда Подвальная, Максим Кудрин

Вопрос 4:
На бортах советских вертолетов, участвовавших в спецоперациях во время
Афганской войны, нередко появлялись ТАКИЕ ИКСЫ. Но они могли быть не
совсем ТАКИМИ и не совсем ИКСАМИ. Однако мы знаем, что даже самый
известный ТАКОЙ ИКС был не совсем ТАКИМ и не совсем ИКСОМ. Назовите
ТАКОЙ ИКС.

Ответ:
Черный квадрат.

Комментарий:
(pic: 20120980.jpg)
   Черными квадратами закрашивались бортовые номера вертолетов. Черный
квадрат мог быть не черным и не квадратом.

Источник:
   1. Виктор Марковский. Жаркое небо Афганистана. 1979-1989. - М.:
Техника - молодежи, 2000.
   2. http://otvaga2004.narod.ru/publ_w1/2007-03-30.htm

Автор:
Максим Кудрин

Вопрос 5:
В одном произведении ОНИ разрезали город. Стоит отметить, что это был
город с самой протяженной сетью ИХ в России. Назовите ИХ.

Ответ:
Трамвайные рельсы.

Комментарий:
Песня Александра Васильева: "Кто-то разрешил трамвайным рельсам
разрезать этот город", а живет он в Питере.

Источник:
   1. "Сплин", "Спи в заброшенном доме".
   2. http://ru.wikipedia.org/wiki/Трамвай

Автор:
Максим Кудрин

Вопрос 6:
Прослушайте русский перевод предложения, написанного на языке хопи:
"Тогда, на следующий день, довольно рано утром, в час, когда люди
молятся солнцу, он снова разбудил девушку". Этим предложением Экксхарт
Малотки отвергал гипотезу Бенджамина Уорфа о лингвистической
относительности, согласно которой индейцы не воспринимали ИКС как поток
дискретных элементов. Назовите ИКС.

Ответ:
Время.

Источник:
http://www.nkj.ru/archive/articles/19812/

Автор:
Вера Фоминых

Вопрос 7:
Об этой гуманной профессии весь мир узнал благодаря британским военным
летчикам, принимавшим участие в боях на Фолклендских островах. Для того
чтобы опровергнуть саму необходимость этой профессии, ВМС Великобритании
выделили около 20 тысяч фунтов стерлингов, патрульный катер и два
вертолета. Назовите эту редкую профессию.

Ответ:
Подниматель пингвинов.

Комментарий:
Биологи в течение пяти недель вели наблюдение за пингвинами, но ни
одного падения пингвина, вызванного пролетающим над ним вертолетом,
выявить не удалось.

Источник:
http://www.newsru.com/world/04feb2001/pingvin.html

Автор:
Максим Кудрин

Вопрос 8:
По определению Толкового словаря Ефремовой, ОНИ - в том числе и цепочка
кругов на поверхности воды, образующаяся в результате последовательных
ударов по ней брошенного по касательной плоского камня. А вот русская
пословица говорит, что ОНИ не могут расколоть брюхо. Назовите ИХ.

Ответ:
Блины.

Комментарий:
Блин не клин, брюха не расколет.

Источник:
   1. http://dic.academic.ru/dic.nsf/efremova/142516/
   2. http://dic.academic.ru/dic.nsf/dahl_proverbs/16328/

Автор:
Надежда Подвальная

Вопрос 9:
В качестве ИКСА нередко используют камни, коробки, консервные банки,
пустые бутылки. А некоторые племена эскимосов верили, что полярное
сияние возникает из-за того, что духи предков в качестве ИКСА используют
череп моржа. Назовите ИКС.

Ответ:
Мяч.

Комментарий:
Эскимосы считали, что духи предков перебрасываются на небе черепом
моржа.

Источник:
Рэй Дж. Карлтон, М.Дж. Маккормик-Рэй. Живой мир полярных районов. - Л.:
Гидрометеоиздат, 1988.
http://antarctic.su/books/item/f00/s00/z0000017/st003.shtml

Автор:
Надежда Подвальная

Вопрос 10:
ТАКИМИ обычно бывают ИКСЫ, реже - катера и автомобили. Было даже два
ТАКИХ вагона-лаборатории, при создании которых донорами послужили именно
ТАКИЕ ИКСЫ. Что мы заменили словами "ТАКИЕ ИКСЫ"?

Ответ:
Реактивные самолеты.

Комментарий:
В США и СССР были созданы вагоны-лаборатории с реактивными двигателями
от самолетов B-36D и ЯК-40, соответственно.

Источник:
http://www.popmech.ru/article/5323-dorogi-ognedyishaschego-poezda/

Автор:
Максим Кудрин

Вопрос 11:
По мнению героя романа Жоржи Амаду, бразильская столица какао - город
Ильеус своим богатством обязан "пешишика", ведь они спасли урожай от
вредителей. Не знаем, как с "пешишика", а в случае с "гвардейцами"
полезное влияние подтверждено научно. Кто же такие "пешишика" и
"гвардейцы"?

Ответ:
Муравьи.

Комментарий:
Муравьи защищают соцветия от поедания вредителями.

Источник:
   1. Жоржи Амаду. Город Ильеус.
   2. http://www.betonolog.ru/30/index.html

Автор:
Максим Кудрин

Вопрос 12:
ТАКИЕ машинисты работали на паровозах серии Ку, обладавших неглубокой
топкой, но даже в годы войны ТАКИХ машинистов было не больше 1%. Какое
существительное мы заменили прилагательным "ТАКИЕ"?

Ответ:
Женщины.

Комментарий:
Относительно неглубокая топка этих паровозов не требовала от работавших
на них кочегаров и помощников машиниста большой физической силы.
Благодаря этой особенности именно на "кукушках" зародилось движение
женских паровозных бригад. Так, первая в СССР женщина-машинист Зинаида
Троицкая начинала свою трудовую деятельность с того, что бросала в топку
уголь на паровозе "Ку" N 6...

Источник:
   1. http://vivovoco.rsl.ru/VV/PAPERS/HISTORY/KY.HTM
   2. http://www.vparavoz.com/typec/typec_7.html

Автор:
Максим Кудрин

Вопрос 13:
"Общество анонимных алкоголиков" появилось во Франции во времена
Июльской революции 1830 года. В Сталинских лагерях к анонимным
алкоголикам были причислены взяточники, убийцы и рецидивисты. Какие два
существительных мы заменили словами "анонимный алкоголик"?

Ответ:
Друг народа.

Комментарий:
"Друзья народа" были осуждены по уголовным статьям, а не по
политическим.

Источник:
   1. http://ru.wikipedia.org/wiki/Друг_народа
   2. Варлам Шаламов. Сгущенное молоко.

Автор:
???

Вопрос 14:
В 2005 году в зоопарке Загреба были оборудованы два вольера для таких
особей. Один предназначался для хорошей особи, второй - для плохой. В
2007 году в зоопарке Аделаиды в вольер поместили группу таких особей.
Стоит заметить, что до 1930-х годов такие особи содержались не менее чем
в 15 зоопарках всего мира. Что гласили надписи на вольерах в Загребе?

Ответ:
Homo sapiens.

Зачет:
Человек разумный.

Комментарий:
В 2005 и 2007 годах это были экологические акции, а в начале XX века в
зоопарках содержали коренных жителей европейских колоний. Возможно, Homo
sapiens упоминался не только на табличках в Загребе, но достоверных
фактов автор больше не нашел.

Источник:
   1. http://www.utro.ru/articles/2005/09/20/478504.shtml
   2. http://lenta.ru/news/2007/01/09/zoo/
   3. http://ribalych.ru/2011/07/14/lyudi-v-zooparkax/

Автор:
Максим Кудрин

Вопрос 15:
Шутка с ИКСОМ стала визитной карточкой одного сериала, там он ломался,
переворачивался, похищался и даже убегал самостоятельно. А реальном мире
ИКСЫ могут иметь различные механические схемы, среди которых аккордеон,
дельфин и клик-клак. Назовите ИКС.

Ответ:
Диван.

Комментарий:
"Диванная шутка" есть в каждой серии "Симпсонов", а аккордеон, дельфин и
клик-клак - это механизмы трансформации складных диванов.

Источник:
   1. http://lenta.ru/articles/2009/02/18/simpsons/
   2. http://www.snpp.com/guides/couch.openings.html
   3. http://ru.wikipedia.org/wiki/Диван

Автор:
Максим Кудрин

Вопрос 16:
Герой мультфильма "Мэри и Макс" использовал шлем с двумя рогами для
защиты от нападения ворон. Двурогими также были Брилиант и его
многочисленные аналоги. Что мы заменили словом "рога"?

Ответ:
Глаза.

Комментарий:
Макс нарисовал на своем шлеме два больших глаза. Бриллиант - это
двухобъективный зеркальный фотоаппарт Voigtl&auml;nder Brillan, этот тип
камер принято назвать двуглазыми.

Источник:
   1. Мультфильм "Мэри и Макс".
   2. http://r-s-u.livejournal.com/3581.html

Автор:
Максим Кудрин

Вопрос 17:
Многие герои комиксов являются ИМИ. А героя одной киноновеллы - человека
в ростовом костюме креветки ИМ называют в шутку. Назовите ИХ.

Ответ:
Мутанты.

Комментарий:
"Не простудись, мутант", - говорит капитан речного судна, обращаясь к
главному герою.

Источник:
Фильм "Короткое замыкание".

Автор:
Максим Кудрин

Вопрос 18:
При проведении подводных съемок с увеличением глубины предметы
постепенно теряют ЕГО. А целый ряд советских кинофильмов недавно ЕГО
приобрел. Назовите ЕГО.

Ответ:
Цвет.

Комментарий:
На большой глубине все предметы имеют сине-зеленый цвет, а многие
советские черно-белые фильмы были колоризированы в 2009-2011 годах.

Источник:
   1. Энциклопедия "Техника". - М.: Росмэн, 2006.
   2. http://ru.wikipedia.org/wiki/Колоризация

Автор:
Максим Кудрин

Вопрос 19:
По мнению Варлама Шаламова, образцом для архитектуры сталинских высоток
послужила АЛЬФА, ставшая главной идеей времени. Назовите АЛЬФУ.

Ответ:
Вышка.

Зачет:
Караульная вышка, лагерная вышка и т.д. по смыслу.

Источник:
В. Шаламов. Колымские рассказы. По лендлизу.

Автор:
Максим Кудрин

Вопрос 20:
Одними из первых испытателей плаща были французские заключенные. По злой
иронии, советский инженер, получивший два патента на использование в
плащах резиновых тросов, смягчающих удар, был арестован в 1937 году.
Какое слово французского происхождения мы заменили словом "плащ"?

Ответ:
Парашют.

Комментарий:
Советский инженер - это Петр Владимирович Бехтерев.

Источник:
   1. http://ru.wikipedia.org/wiki/История_парашютизма
   2. http://ru.wikipedia.org/wiki/Бехтерев,_Пётр_Владимирович

Автор:
Рашид Шакуров

Вопрос 21:
Немецкий сатирик Георг Кристоф Лихтенберг утверждает, что ДЕЛАЮТ ЭТО
только из-за себя, а ДЕЛАЮТ ТО и за себя, и за других. Можно сказать,
что на протяжении 1918-1920 годов многие люди успевали "СДЕЛАТЬ ТО" и
"СДЕЛАТЬ ЭТО" по нескольку раз. Какой глагол мы заменили словами "ДЕЛАТЬ
ТО"?

Ответ:
Краснеть.

Комментарий:
ДЕЛАТЬ ЭТО - бледнеть.

Источник:
http://dic.academic.ru/dic.nsf/aphorism/1135/

Автор:
Максим Кудрин

Вопрос 22:
По утверждению Ильи Ильфа, на НИХ мы можем увидеть дворницкие лица,
тонкую сатирическую улыбку, глуповатую немецкую красавицу, репу и даже
малиновые ягодицы. Стоит заметить, что привычный для нас рисунок на НИХ
появился около 150 лет назад. Назовите ИХ двумя словами.

Ответ:
Игральные карты.

Зачет:
Атласные карты.

Комментарий:
Лица дворников - у королей, улыбка - у валета треф, немецкая красавица -
дама бубен, репа - туз пик, малиновые ягодицы - черви.

Источник:
   1. Илья Ильф. Из записных книжек. - Л.: Художник РСФСР, 1966.
   2. http://ru.wikipedia.org/wiki/Атласная_колода

Автор:
Максим Кудрин

Вопрос 23:
В соответствии с приказом от 30 мая 1985 года, ТУДА нельзя доставлять
депутатов, военнослужащих, работников органов внутренних дел,
государственной безопасности и прокуратуры, а также женщин с явными
признаками беременности. Для доставки ТУДА других граждан запрещается
использовать мотороллеры, мотоциклы, самосвалы. Куда - ТУДА?

Ответ:
В вытрезвитель.

Зачет:
В медвытрезвитель.

Источник:
   1. Приложение N 1 к Приказу МВД СССР от 30 мая 1985 г. N 106.
   2. http://tipolog.atspace.com/doc_vitrezvitel.htm

Автор:
Максим Кудрин

Вопрос 24:
Чаще всего на этом месте устанавливались статуэтки, изображающие
млекопитающих, птиц и античных богов. Но были и курьезы: так, после
нескольких побед нелепую улитку сменил на этом месте триумфатор.
Назовите это место.

Ответ:
Капот автомобиля.

Зачет:
Радиатор и т.д. по смыслу.

Комментарий:
До 1923 года с капота чешских автомобилей Praga сползала улитка, но
после нескольких побед в ралли ее заменили триумфатором с лавровым
венком.

Источник:
Журнал "Автопилот", N 11, февраль 1995 г.

Автор:
Максим Кудрин

Тур:
4 тур. "131" (Серпухов)

Вопрос 1:
Блиц. Корреспондент Lenta.ru в обзоре перед жеребьевкой на Евро-2012
предложил разделить команды на группы по принципу алкогольных напитков.
   1. Водочную группу составляли бы команды таких стран: [ПРОПУСК],
Россия, Швеция, Дания. Заполните пропуск одним словом.
   2. Винную группу составляли бы команды таких стран: Испания,
[ПРОПУСК], Греция, Франция. Заполните пропуск одним словом.
   3. Пивную группу составляли бы команды таких стран: Польша, Германия,
[ПРОПУСК], Чехия. Заполните пропуск одним словом.

Ответ:
   1. Украина.
   2. Италия.
   3. Хорватия.

Комментарий:
Подсказкой должно было послужить то, что Украина в первой корзине,
Италия - во второй, Хорватия - в третьей. Наша команда поднимает за
хорошую игру рюмку водки, бокал вина и кружку пива.

Источник:
http://lenta.ru/online/2011/12/02/euro/

Автор:
Павел Бордачев

Вопрос 2:
Еще будучи слушателем военной академии, ОН был послан на берег Финского
залива произвести съемку местности. Возвращаясь из леса, ОН увяз в
болоте. Впоследствии, вспоминая этот случай, ОН писал: "Если где
придется мне на лошади ездить, так, чтобы эту сивку помнить, всегда буду
белую выбирать". Назовите ЕГО.

Ответ:
[Михаил Дмитриевич] Скобелев.

Комментарий:
После этого случая у Михаила Дмитриевича возникло мистическое
пристрастие к лошадям белой масти; белый мундир был продолжением белизны
его коня; сам Скобелев постепенно внушил себе и другим, что в белом он
заговорен от пуль и не может быть убит неприятелем.

Источник:
http://www.rnns.ru/109390-belyj-general.html

Автор:
???

Вопрос 3:
   <раздатка>
   Человек с ИКСОМ открыт и доступен для других, принимает других людей,
не видит для себя опасности в том, что другие талантливы и добры,
поскольку он твердо уверен в себе, понимая, что является частью большего
целого, и, наоборот, человек с ИКСОМ угнетен, когда других оскорбляют
или унижают, пытают или угнетают.
   </раздатка>
   Перед вами определение, данное в 1999 году архиепископом Десмондом
Туту ИКСУ как южноафриканской идеологии. Мы надеемся, что оно льстит
присутствующим здесь программистам, ведь пользование ИКСОМ - тоже
отчасти идеология. А основатель ИКСА Марк Шаттлворт даже спустился с
небес на землю 5 мая 2002 года. Назовите ИКС.

Ответ:
Убунту.

Зачет:
Ubuntu. Незачет: Linux.

Комментарий:
Марк Шаттлворт - второй космический турист.

Источник:
   1. http://ru.wikipedia.org/wiki/Убунту_(идеология)
   2. http://ru.wikipedia.org/wiki/Шаттлворт,_Марк

Автор:
Константин Горохов

Вопрос 4:
Сотрудники компании "Nestl&eacute;" выпустили бесплатные веб-приложения
для iPad для не совсем обычной целевой аудитории. По экрану во время
работы этих приложений перемещаются изображения рыбы, цветастых фигурок
или ИКСА. Догадайтесь, кто составляет эту аудиторию, и назовите ИКС
двумя словами, начинающимися на одну и ту же букву.

Ответ:
Кошачий корм.

Комментарий:
Кошачий корм "Friskies" - торговой марки, которой владеет компания
"Nestl&eacute;".

Источник:
http://www.iphones.ru/iNotes/143032

Автор:
Павел Бордачев

Вопрос 5:
Ив Сен-Лоран как-то заменил, что вечернее платье от пеньюара отличается
только количеством ИХ. Количеством ИХ различаются также "Титаник" и
"Самолет президента". Назовите ИХ одним словом.

Ответ:
Зрители.

Источник:
   1. Даниель Дэкубе. 100 советов парижанки.
   2. http://www.kinopoisk.ru/level/1/film/2213/
   3. http://www.kinopoisk.ru/level/1/film/2991/

Автор:
???

Вопрос 6:
ИКС, получивший в России популярность благодаря Александру Зиновьеву,
стал персонажем известного компьютерного анимационного ролика. Автор
вопроса назвал словом "ИКС" своего знакомого из-за того, что во время
стремительного перемещения этого знакомого на остановке между вагонами
электрички ему изменила жена. Воспроизведите слово, которое обозначено
как "ИКС".

Ответ:
Кролень.

Комментарий:
Знакомый автора вопроса был безбилетником, так что в момент измены жены
его можно было бы назвать "рогатым зайцем", или "кроленем". В известном
анимационном ролике 2003 года "Барашек" есть похожий персонаж, которого
в русском переводе назвали "Кроленем".

Источник:
   1. http://ru.wikipedia.org/wiki/Рогатый_заяц
   2. http://www.your-mind.ru/lasta/multfilm-pro-barashka/

Автор:
Сергей Бондаренко

Вопрос 7:
В музее "Сотворение мира", созданном креационистами, посетителей
встречает огромное количество динозавров. Их отсутствие в современном
мира создатели музея объясняют трагическим происшествием. Назовите
абсолютно точно объект, на котором произошла эта трагедия.

Ответ:
Ноев ковчег.

Комментарий:
Были смыты за борт ковчега во время потопа; креационисты - это
христиане, верующие в Сотворение мира Богом.

Источник:
"GEO", апрель 2009 г.

Автор:
???

Вопрос 8:
(pic: 20120981.jpg)
   Воспроизведите три слова, закрытые на данной карикатуре.

Ответ:
"... Москву на область".

Комментарий:
(pic: 20120982.jpg)

Источник:
http://caricatura.ru/art/zhigadlo/url/daily/zhigadlo/435/

Автор:
???

Вопрос 9:
В газете "Спорт-Экспресс" есть материал о футболистах, которым чуть-чуть
не хватило для вступления в клуб Григория Федотова. Название этого
материала из одного слова, по мнению автора вопроса, могло бы также
охарактеризовать следующие моменты времени: 4-5 февраля 1962 года;
сентябрь 1994 года; час дня 2 минуты 3 секунды 4 числа пятого месяца
2006 года; 4 часа утра 11 августа 2002 года; 29 апреля 2007 года.
Воспроизведите название материала.

Ответ:
"НесоСТОявшиеся".

Зачет:
"Несостоявшиеся".

Комментарий:
В клуб Григория Федотова входят футболисты, забившие сто и более мячей;
приведенные в вопросе моменты времени считались моментами, когда должен
был состояться конец света.

Источник:
   1. http://www.sport-express.ru/newspaper/2003-07-08/8_2/
   2. http://www.mystic-news.com/articles/227-the-ends-predicted.html

Автор:
???

Вопрос 10:
Думаю, вам известны слова Сталина "Сын за отца не отвечает". После
значительного события в мировой политике, произошедшего 17 декабря 2011
года, в прессе появился заголовок, в котором сталинская фраза лишилась
слова "не" и еще одной буквы. Назовите отца, которому была посвящена
статья под этим заголовком.

Ответ:
Ким Чен Ир.

Комментарий:
Заголовок выглядит так: "Ын за отца отвечает" и посвящен похоронам Ким
Чен Ира его преемником и младшим сыном - Ким Чен Ыном.

Источник:
http://www.rg.ru/2011/12/28/pohoroni-site.html

Автор:
Константин Горохов

Вопрос 11:
Рассказывают, что однажды Виктора Понедельника хотели, вопреки его
желанию, перевести из ростовского "СКА" в московский "ЦСКА". Отец
футболиста обратился за помощью к Михаилу Шолохову. Воспроизведите
фразу, которую в сердцах произнес Шолохов, узнав об этом переводе.

Ответ:
"С Дона выдачи нет!".

Источник:
http://www.sport-express.ru/newspaper/2011-10-21/7_3/

Автор:
???

Вопрос 12:
Как-то раз Джина Лоллобриджида заметила: "ОНА - лучшая косметика". Это
высказывание Ив Сен-Лоран продолжил словами: "Зато косметику легче
достать". С 2007 года ОНА у многих россиян стала ассоциироваться с
растением. Назовите это растение.

Ответ:
Морковь.

Комментарий:
ОНА - любовь.

Источник:
   1. Д. Дэкубе. 100 советов парижанки.
   2. http://ru.wikipedia.org/wiki/Любовь-морковь

Автор:
???

Вопрос 13:
(pic: 20120983.jpg)
   Внимание, в вопросе есть замены.
   На картинке изображен "ИКС". Другой ИКС создал "ТАКОЙ ИГРЕК", не
входящий в привычный перечень ИГРЕКОВ. Мы не просим вас назвать ИКС или
перечислить все ИГРЕКИ. Какое порядковое числительное мы заменили в этом
вопросе словом "ТАКОЙ"?

Ответ:
Тринадцатый.

Комментарий:
На картинке изображен комплекс "Искандер"; Фазиль Искандер написал
произведение "Тринадцатый подвиг Геракла".

Автор:
Игорь Трескунов

Вопрос 14:
По свидетельству современников, комментатор Вадим Синявский зачастую
позволял себе приукрашивать события, происходившие на футбольном поле. В
этом предложении мы пропустили пять букв. Напишите эти буквы.

Ответ:
Радио.

Комментарий:
Вадим Синявский был радиокомментатором.

Источник:
http://ford.championat.com/news/item/id/58

Автор:
Павел Бордачев

Вопрос 15:
ПО одной из версий, Отто фон Бисмарк, будучи послом Пруссии в России, не
переносил ЭТО СЛОВО, поскольку не верил, что такой замечательный народ,
как русские, могут вкладывать в ЭТО СЛОВО хоть малую толику смысла. Дело
дошло до того, что Бисмарк... Возможно, некоторые из вас помнят, чем
дело кончилось. А может, и не помнят. Ну, ничего. Напишите ЭТО СЛОВО.

Ответ:
Ничего.

Источник:
В.С. Пикуль. Битва железных канцлеров. - М.: Современник, 1989. - С. 91.

Автор:
???

Вопрос 16:
В романе Михаила Кликина "Страж могил" ИКС, порождающий полчища
уродливых теней, сравнивается с некромантом, поднимающим мертвых. Этот
вопрос стать источником ИКСА не может. Назовите ИКС двумя словами.

Ответ:
Огонек свечи.

Зачет:
По смыслу.

Комментарий:
Этот вопрос "свечкой" не является.

Источник:
http://www.klikin.ru/txt/tombg.html

Автор:
???

Вопрос 17:
Известный двукратный олимпийский чемпион утверждал, что армия этого
государства составляла 840.000 воинов, 120.000 всадников и 10.000
колесниц, флот насчитывал 1200 кораблей с 240.000 матросов. Назовите это
государство.

Ответ:
Атлантида.

Комментарий:
Платон был не только философом, но и олимпийским чемпионом. Дважды он
выигрывал соревнования по панкратиону - смеси бокса и борьбы без правил.

Источник:
   1. http://chudesa.by.ru/atlanplat.html
   2. http://www.krugosvet.ru/enc/sport/OLIMPISKIE_IGRI_DREVNE_GRETSII.html?page=0,1

Автор:
???

Вопрос 18:
(pic: 20120984.jpg)
   Блиц.
   В связи с событиями в начале декабря 2011 года, происходившими в
Москве и Санкт-Петербурге, появилась следующая карикатура, которую вам
раздали.
   1. У первой обезьянки закрыт логотип ИКСА, который изначально был
вторым. Что мы заменили на ИКС?
   2. У второй обезьянки закрыт логотип АЛЬФЫ, которая успела побывать
четвертой, второй и вторым. Какое название мы заменили на АЛЬФУ?
   3. У третьей обезьянки закрыт логотип ЗЕТ, который, если верить
создателю, изначально расшифровке не подлежал. Что мы заменили на ЗЕТ?

Ответ:
   1. "Первый канал".
   2. "Россия-1".
   3. "НТВ".

Зачет:
   1. "1".

Комментарий:
(pic: 20120985.jpg)
   1. "Первый канал" изначально был вторым телеканалом в СССР.
   2. История телеканала "Россия-1" начинается 4 ноября 1967 года, когда
была создана "Четвертая программа". 1 января 1982 года телеканал
получает новое название - "Вторая программа". В 1984 году "Вторая
программа" получила еще одно название - "Второй канал".
   3. Аббревиатуру "НТВ" решили вообще не расшифровывать.

Источник:
http://relax.ru/post/40587/vse_o_vyborakh_37_foto.html

Автор:
Павел Бордачев

Вопрос 19:
Прообразом одной из НИХ является жена Антуана де Сент-Экзюпери. Другая
ОНА родилась в Польше и убита в Берлине в начале XX века. Назовите ее
фамилию.

Ответ:
Люксембург.

Источник:
   1. http://ru.wikipedia.org/wiki/Маленький_принц
   2. http://ru.wikipedia.org/wiki/Люксембург,_Роза

Автор:
Константин Горохов

Вопрос 20:
С Ричардом III и Борисом Ельциным ЭТО произошло, в общем-то, без
серьезных последствий. А для персонажа, прообразом которого, по одной из
версий, считается Ричард III, ЭТО закончилось куда трагичнее. Назовите
этого персонажа.

Ответ:
Шалтай-Болтай.

Комментарий:
ЭТО - падение.

Источник:
   1. http://ru.wikipedia.org/wiki/Падение_Ельцина_с_моста
   2. http://ru.wikipedia.org/wiki/Шалтай-Болтай

Автор:
Константин Горохов

Вопрос 21:
В Советском Союзе продукция завода "Полимер" была хорошо известна
большинству жителям СССР, хоть и считалась сезонной. Назовите город в
Ленинградской области, в котором располагался завод.

Ответ:
Сланцы.

Комментарий:
Некоторые покупатели полагали, что выдавленное на подошвах шлепанцев
слово "Сланцы" - это название обуви, а не место ее изготовления. Далее
слово вошло в активный словарный запас и превратилось в синоним слова
"шлепанцы".

Источник:
http://ru.wikipedia.org/wiki/Сланцы

Автор:
???

Вопрос 22:
   <раздатка>
   ... взят Эяэцурс, и завоевана слава всей земли!
   </раздатка>
   Он использовался орденом тамплиеров, которые переняли его от катаров.
Последние получили его от гностиков. А сначала он, скорее всего,
использовался ессеями. Мы не спрашиваем о нем. Ответьте, что
подразумевал автор древнего произведения под словом, которое мы заменили
в строке раздатки.

Ответ:
Вавилон.

Зачет:
Бавель, &#1489;&#1489;&#1500;.

Комментарий:
Речь идет о подстановочном шифре "Атбаш", принцип которого заключается в
замене каждой буквы текста другой буквой, отстоящей от конца алфавита на
столько же позиций, насколько исходная буква отстоит от его начала. Для
раздатки была взята цитата из книги Иеремии "... взят Сесах, и завоевана
слава всей земли!" (Иер. 51:41). Слово &#1513;&#1513;&#1498; ("шешах",
или "сесах" в синодальном переводе) - это атбаш слова
&#1489;&#1489;&#1500; ("Бавель" - Вавилон). Мы же применили атбаш к
слову "Вавилон" и получили "Эяэцурс".

Источник:
   1. П. Ланде. Тайные знаки. - М.: Вокруг света, 2011. - С. 48.
   2. http://ru.wikipedia.org/wiki/Атбаш
   3. Иер. 51:41.

Автор:
???

Вопрос 23:
Эта должность появилась в перечне должностей отеля "Уэстин Сент-Фрэнсис"
в 1935 году. Уже 20 лет, как эту должность занимает Роберт Холсен. Если
верить его собственной шутке, за это время Холсен заработал на хороший
срок. Ответьте устойчивым выражением, чем занимается Холсен.

Ответ:
Отмывает деньги.

Зачет:
Отмывает монеты.

Комментарий:
Моет и полирует монеты находящиеся в обороте внутри отеля.

Источник:
http://www.vokrugsveta.ru/vs/article/7469/

Автор:
???

Вопрос 24:
В одном фильме, снятом по мотивам давно известного сюжета, ОМИКРОН
говорит АЛЬФЕ, что эпилог всегда интереснее пролога. Назовите в
правильном порядке двух героев, имена которых мы заменили буквами АЛЬФА
и ОМИКРОН.

Ответ:
Ахиллес и Одиссей.

Комментарий:
С этих букв начинаются их имена при письме на древнегреческом.

Источник:
Фильм "Троя" (2004).

Автор:
Сергей Ясинский

Тур:
5 тур. "Понаразваливались" (сборная)

Инфо:
Команда выражает благодарность за помощь в подготовке пакета Наталье
Кравченко, Дарье Пересветовой, Максиму Кулько, Наталье и Галине
Максимовым, Андрею Кульянову, Сергею Кузнецову, Александру Рудневу,
Артему Агапову и Геворгу Авакянцу.

Вопрос 1:
[Ведущему: первый вопрос зачитать сразу после чтения благодарностей, без
паузы; название сайта произнести не очень разборчиво, чтобы чтение его
по буквам выглядело естественно.]
   Пользователь нетематического форума yurclub.ru (Юрий, Рубен,
Константин, Леонид, Ульяна, Борис) предлагает использовать в
интернет-общении аббревиатуру КВЭЛ (Кирилл, Валентин, Эдуард, Лазарь).
Как она расшифровывается?

Ответ:
Кто все эти люди?

Комментарий:
Стандартная фраза, означающая полное непонимание темы. В старой шутке
так же реагируют на произнесение слов по именам, как это было сделано в
вопросе. Список благодарностей, выраженных никому не известным знатокам,
мог вызвать у вас тот же вопрос. :-)

Источник:
http://forum.yurclub.ru/index.php?showtopic=224224&st=1420

Автор:
Николай Крапиль (Москва)

Вопрос 2:
Рассуждая о своем приходе в европейскую культуру, еврей по происхождению
Генрих Гейне сравнил крещение с НИМ. Самого Гейне можно было увидеть на
НЕМ Дюссельдорфского театра. Назовите ЕГО двумя словами.

Ответ:
Входной билет.

Источник:
http://lib.rus.ec/b/245156/read

Автор:
Михаил Малкин (Люберцы, Московская область)

Вопрос 3:
Трое героев книги Татьяны Луганцевой практически пропадают из
повседневной жизни, пытаясь разобраться в своих взаимоотношениях. Какие
два прилагательных входят в название книги?

Ответ:
Бермудский, любовный.

Зачет:
В любом порядке.

Комментарий:
Книга называется "Бермудский любовный треугольник".

Источник:
http://lib.rus.ec/b/98383/read

Автор:
Николай Крапиль (Москва)

Вопрос 4:
На болгарский ОНИ переводятся как "ваденки", а на чешский как "налепки".
Назовите ИХ.

Ответ:
Переводные картинки.

Комментарий:
Переводиться - вообще их основное занятие. :-)

Источник:
   1. http://translate.google.ru/#bg/ru/ваденки
   2. http://translate.google.ru/#cs/ru/n%C3%A1lepky

Автор:
Михаил Малкин (Люберцы, Московская область)

Вопрос 5:
Вратарь Сергей Овчинников считает, что и его карьера футболиста, и его
тренерский дебют были прерваны другими людьми, когда он сам был полон
решимости двигаться дальше. "То, что было исключено на поле, - пишет
Овчинников, - настигло меня в жизни". Одно из заглавий книги Овчинникова
- "ТАКОЙ ОН". Что мы заменили на "ТАКОЙ ОН"?

Ответ:
Искусственный офсайд.

Комментарий:
Вратарь, как отмечает Овчинников, - единственный, кто не может попасть в
искусственный офсайд на поле, зато это происходит в жизни - из-за
действий других игроков он оказывается "вне игры", хотя сам хочет и
может двигаться вперед.

Источник:
http://www.loko4you.ru/library/19713-iskusstvennyj-ofsajd-boss-vsegda-prav-predislovie.html

Автор:
Николай Крапиль (Москва)

Вопрос 6:
Персонаж Юрия Светлакова был амнистирован и без каких-либо проблем
вернулся домой. В его воспоминаниях фигурирует теплое ОНО. Назовите ЕГО
точно.

Ответ:
Лето пятьдесят третьего.

Комментарий:
В отличие от героев фильма "Холодное лето пятьдесят третьего", у него
амнистия прошла хорошо, поэтому, видимо, и воспоминания другие.

Источник:
Ю. Светлаков. Съемщик. Шаг за горизонт. - Кемерово: Сибирский писатель,
2000.

Автор:
Николай Крапиль (Москва)

Вопрос 7:
Каждому туристу, отправляющемуся на экскурсию по румынским замкам,
журнал "Euguide" советует взять с собой правду. Какие два слова мы
заменили в этом вопросе?

Ответ:
По чесноку.

Комментарий:
Должно помочь, если встретите вампира. Слова "правду" и "по чесноку" -
контекстуальные синонимы.

Источник:
http://www.euguide.ru/2054.html

Автор:
Николай Рябых (Королёв, Московская область)

Вопрос 8:
Отмечая снижение уровня экшена в киносаге "Сумерки", знакомая автора
вопроса говорит, что в последней части фильма вообще нет ни одной АЛЬФЫ
вплоть до сцены родов главной героини. Назовите АЛЬФУ одним словом.

Ответ:
Схватка.

Комментарий:
В сцене родов схватки, разумеется, есть, но не те, которые
подразумеваются жанром экшен-фильма.

Источник:
   1. ЛОАВ.
   2. Фильм "Сумерки. Рассвет. Часть 1" (2011).

Автор:
Николай Крапиль (Москва)

Вопрос 9:
Два политика с одинаковыми взглядами встречаются не так часто, но, когда
это происходит, результат может превзойти ожидания. Арам Абрамян
сравнивает политику с НИМИ. Макс Фрай, рассуждая о НИХ, отмечает
понятность человеческого стремления поскорее вернуться домой. Назовите
ИХ.

Ответ:
Нарды.

Комментарий:
Дубль (две одинаковых кости) в нардах тоже дает существенную синергию.
Одна из целей в нардах - вернуть шашки домой.

Источник:
   1. А. Абрамян. Всё зависит от поставленной цели.
http://www.aravot.am/ru/home/archive/0/view/2011-08-18
   2. Макс Фрай. Про нарды.
http://www.svenlib.sandy.ru/pugovichki/fignya/stories/37.htm

Автор:
Николай Крапиль (Москва)

Вопрос 10:
Герой современного израильского писателя Меира Шалева отправляет с НИМ
пробирку своей юной подруге. ОН изображен на советской марке. Назовите
ЕГО.

Ответ:
[Почтовый] голубь.

Комментарий:
У Шалева библейский сюжет, а на почтовой марке - почтовый голубь.

Источник:
   1. http://ru.wikipedia.org/wiki/Голубь_и_мальчик
   2. http://ru.wikipedia.org/wiki/Почтовый_голубь

Автор:
Михаил Малкин (Люберцы, Московская область)

Вопрос 11:
Максим Горький сравнил с НЕЙ пропитавшуюся кровью тряпку на макушке
героя. Свое распространение ОНИ начали из одного марокканского города.
Назовите ИХ.

Ответ:
Фески.

Комментарий:
Из Феса.

Источник:
   1. http://www.flibusta.net/b/19681/read
   2. http://ru.wikipedia.org/wiki/Феска

Автор:
Михаил Малкин (Люберцы, Московская область)

Вопрос 12:
Узнав, что игравший против него защитник соперников не щадит, Егор
Титов, обращаясь к нему после матча, произнес вторую часть названия
российского фильма. Мы не спрашиваем, кто сыграл в этом фильме главного
героя. Назовите фамилию этого героя.

Ответ:
Высоцкий.

Комментарий:
Реплика Титова - "Спасибо, что живой". Официально неизвестно, кто сыграл
главную роль в фильме "Высоцкий. Спасибо, что живой".

Источник:
http://ru-spartak-msk.livejournal.com/2037354.html

Автор:
Николай Крапиль (Москва)

Вопрос 13:
В фильме "Высоцкий. Спасибо, что живой" показаны многие негативные
аспекты жизни в СССР, особенно касающиеся работы КГБ. Кого из своих
предшественников, по словам Дмитрия Быкова, мог процитировать Владимир
Путин, похваливший работу сценариста фильма?

Ответ:
Николая I.

Комментарий:
По популярной легенде, после премьеры "Ревизора" Николай I произнес: "Ну
и пьеска! Всем досталось, а мне более всех".

Источник:
http://ru-bykov.livejournal.com/1225865.html

Автор:
Николай Крапиль (Москва)

Вопрос 14:
Ботаник Отто Брунфельс, разделивший растения на "совершенные" и
"несовершенные", известен своей борьбой с Лютером и составлением полного
гербария, в котором дал растениям названия на немецком - каждому свое. В
каком году он родился?

Ответ:
1488.

Комментарий:
1488 - кодовый лозунг нацистов. Слова "его борьба" и "каждому свое" в
тексте вопроса - подсказки.

Источник:
http://en.wikipedia.org/wiki/Otto_Brunfels

Автор:
Сергей Даровских (Москва)

Вопрос 15:
[Ведущему: "П раздник" произносится так, чтобы было очевидно, что это
два разных слова, а не "праздник".]
   В 2010 году в Петербурге прошла выставка под названием "П раздник".
Мы не спрашиваем, что мы заменили на "П раздник". Ответьте, свои
вариации на тему чьей известной картины разные художники представляли на
этой выставке.

Ответ:
[Диего] Веласкеса.

Комментарий:
Выставка называлась "И Менины". "Менины" - одна из самых известных
картин Веласкеса.

Источник:
http://www.erarta.com/timetable/23/

Автор:
Николай Крапиль (Москва)

Вопрос 16:
Несмотря на все усилия редакторов, этот вопрос мало чем отличается от
остальных вопросов пакета.
   По воспоминаниям, до того как этот человек получил широкую
известность, он обильно использовал ненормативную лексику. С этим
Дмитрий Зеркалов связывает известную особенность. Назовите этого
человека.

Ответ:
[Виктор Степанович] Черномырдин.

Комментарий:
По воспоминаниям, в обычной жизни Черномырдин прекрасно выражал свои
мысли, но много матерился - в публичных выступлениях мат приходилось
чем-то заменять, отсюда и возникает знаменитое косноязычие политика.
Первая фраза вопроса намекает на самое известное высказывание
Черномырдина - "Хотели как лучше, получилось как всегда".

Источник:
http://zerkalov.org.ua/node/6018

Автор:
Николай Крапиль (Москва)

Вопрос 17:
Грустная ария героини российского мюзикла заканчивается словами:
   Распни меня над пропастью и причини мне боль,
   На письма на мои не отвечая.
   Но почему, о, Господи, ты ИКС унес с собой,
   А вместе с ним и ИГРЕК?
   Мы не спрашиваем, что мы заменили на ИКС. Назовите ИГРЕК тремя
словами.

Ответ:
Ситечко для чая.

Комментарий:
ИКС - стул. Это ария Грицацуевой из мюзикла "Двенадцать стульев".

Источник:
http://jemal-tetruashvili.narod.ru/tvor_tekst_argr.html

Автор:
Николай Крапиль (Москва)

Вопрос 18:
В 2011 году футболист Александр Филимонов стал победителем чемпионата
мира. За это Филимонова отметили премией "ОНИ". Болгарская Википедия
отмечает, что ИХ ширина - около 200 метров. Назовите ИХ двумя словами.

Ответ:
Золотые пески.

Комментарий:
Филимонов победил на чемпионате мира по пляжному футболу. Золотые пески
- известный болгарский пляжный курорт.

Источник:
   1. http://www.marimedia.ru/news/sports/item/14587/
   2. http://bg.wikipedia.org/wiki/Златни_пясъци

Автор:
Михаил Малкин (Люберцы, Московская область)

Вопрос 19:
Рассуждая о довольно неискусных попытках изобразить единство всех
участников недавних оппозиционных митингов, Валерий Дэйв пишет, что
ленточки митинговавших были [ПРОПУСК]. И в этом с ним довольно трудно
поспорить. Восстановите пропуск тремя словами.

Ответ:
"... шиты белыми нитками".

Комментарий:
Валерий имеет в виду образное значение выражения, но по факту он прав -
белые ленточки и правда шиты белыми нитками.

Источник:
http://www.gidepark.ru/community/politic/content/861879

Автор:
Николай Крапиль (Москва)

Вопрос 20:
Семен Комаров предлагает называть словом "ИКС" топового андоррского
блоггера. Назовите имя ИКСА, который в конце пьесы Ольги Чигиринской
стоит, тяжело опираясь на копье.

Ответ:
Лонгин.

Комментарий:
Вообще популярные блоггеры - "тысячники", но в карликовом государстве
читателей не так много, поэтому там не тысячники, а "сотники". Сотник
Лонгин пронзил копьем тело распятого Иисуса.

Источник:
   1. http://ru-football.livejournal.com/6926626.html?thread=379036194#t379036194
   2. http://www.e-reading.org.ua/bookreader.php/85239/Chigirinskaya_-_Longin.html

Автор:
Николай Рябых (Королёв, Московская область)

Вопрос 21:
Герою фильма "Ирония судьбы. Продолжение" с огромным трудом удается
выставить из дома незваного гостя. "Не дай бог!" - произносит этот
герой, увидев на полу дома ИКС. Назовите ИКС словом из трех букв.

Ответ:
Нож.

Комментарий:
Согласно примете, упавший нож предвещает приход гостя-мужчины.

Источник:
Фильм "Ирония судьбы. Продолжение" (2007), реж. Т. Бекмамбетов.

Автор:
Николай Крапиль (Москва)

Вопрос 22:
В семь часов вечера героиня Олега Зайончковского в приподнятом
настроении держит в руках молоток. АЛЬФА на столе этой героини в десять
часов является в рассказе символом одиночества этой героини. Назовите
АЛЬФУ двумя словами, начинающимися на одну и ту же букву.

Ответ:
Остывшая отбивная.

Комментарий:
Молоток, разумеется, для отбивания мяса. Но долгожданный мужчина на ужин
так и не пришел.

Источник:
О. Зайончковский. Прогулки в парке.

Автор:
Николай Крапиль (Москва)

Вопрос 23:
В конце XIX века Жорж Мельес сделал две версии некоего сюжета - "ИКС
Жуанвиля" и "ИКС Венсанна". Какие четыре слова мы заменили на ИКС?

Ответ:
Прибытие поезда на вокзал.

Комментарий:
Это последовавшие после выхода фильма братьев Люмьер "Прибытие поезда на
вокзал Ла-Сьота" короткометражки. Сюжет стал "бродячим", и подобных
фильмов было снято несколько.

Источник:
http://www.kinopoisk.ru/level/1/film/89544/

Автор:
Иделия Айзятулова (Ульяновск - Санкт-Петербург)

Вопрос 24:
Русский философ XIX века Федоров мечтал воскресить всех людей, но
считал, что они не поместятся на Земле. Внебрачным сыном представителя
какой княжеской фамилии был Федоров?

Ответ:
Гагарины.

Комментарий:
Федоров - основатель космизма - предполагал, что нужно осваивать
пространства Солнечной системы, чтобы поселить всех людей там. Вопреки
традиции, вопросами про прибытие поезда и Гагарина мы не открываем тур,
а заканчиваем.

Источник:
http://ru.wikipedia.org/wiki/Фёдоров,_Николай_Фёдорович

Автор:
Михаил Малкин (Люберцы, Московская область)

Тур:
6 тур. "Какая разница" (Серпухов)

Редактор:
Евгений Кононенко (Москва)

Вопрос 1:
Менеджеры подмосковного спортивно-развлекательного парка ДрАкино
уверены, что 2012 год должен стать для их заведения особенно успешным.
Залогом прогноза служат несколько факторов. Во-первых, лётная школа
Дракино используется для детских интерактивных игр. Во-вторых, летом
аэродром примет чемпионат мира по вертолетному спорту. Третий фактор
действует, как водится, с понедельника. Назовите этот фактор двумя
словами.

Ответ:
Год Дракона - для места с названием Дракино он предполагается удачным.

Комментарий:
Год Дракона по китайскому календарю начался в минувший понедельник, 23
января. Кстати, это год Черного Водяного Дракона, на что намекает фраза
"как водится". :-)

Источник:
   1. ЛОАВ.
   2. http://www.drakino.com/
   3. http://www.bolshoyvopros.ru/questions/828-kogda-nachinaetsja-god-drakona-2012.html

Вопрос 2:
По одной из версий, "ОНА" была задумана Павлом Федотовым после смерти
свояка. Поставлявшаяся в вермахт зажигалка IMCO Triplex [имко триплекс]
по сей день считается непревзойденной по безотказности и
неприхотливости, за что была прозвана советскими солдатами "ЕЮ".
Назовите ЕЕ.

Ответ:
Вдовушка.

Комментарий:
Замысел картины "Вдовушка" возник у Федотова после смерти мужа сестры.
Австрийские зажигалки были названы "вдовушками" за безотказность и
неприхотливость.

Источник:
   1. http://www.nearyou.ru/fedotov/51vdov2.html
   2. http://www.vokrugsveta.ru/vs/article/7463/

Вопрос 3:
В постапокалиптическом романе Олега Дивова мутанты видят в ИКСЕ древнего
идола, к которому можно приблизиться по высохшему руслу реки, пройдя
мимо черного культового здания. ИКС упоминается Веллером в
биографической легенде "Гулливер". Напишите число, содержащееся в
официальном названии ИКСА.

Ответ:
300.

Комментарий:
ИКС - памятник "300 лет российскому флоту" работы Церетели (1997),
изображающий Петра I ("Памятник Петру" - Неофициальное название). Под
"Черной церковью" у Дивова выведен обгоревший Храм Христа Спасителя.
Веллер - автор "Легенд Арбата", Арбат + Гулливер = Церетели.

Источник:
   1. О. Дивов. Лучший экипаж Солнечной.
   2. М. Веллер. Гулливер.
   3. http://ru.wikipedia.org/wiki/Памятник_Петру_I_(Москва)

Вопрос 4:
Согласно шутке Кондратия Биркина, безвольная фаворитка Людовика XIV
Луиза де Лавальер была лишь ИМ. В самой известной серии ИХ количество
возрастает от двух до пятнадцати. Назовите создателя этой серии.

Ответ:
Винсент Ван Гог.

Комментарий:
Для Короля-Солнца фаворитка была лишь подсолнечником. В картинах серии
"Подсолнухи" Ваг Гога количество цветков увеличивается от двух до
пятнадцати.

Источник:
   1. К.П. Биркин. Временщики и фаворитки.
   2. http://www.arttower.ru/forum/index.php?showtopic=19095

Вопрос 5:
Оксфордские ученые опытным путем установили, что восприятие этого
знакомого всем звука делает человека более проворным. Для объяснения
такого эффекта выдвинуты несколько исторических гипотез, в том числе
"препятствие появлению конкурентов" и "угроза коллективной
безопасности". Назовите источник этого звука абсолютно точно.

Ответ:
Младенец.

Зачет:
Новорождённый, маленький ребенок.

Комментарий:
При звуках детского плача действия взрослого инстинктивно ускоряются.
Ученые объясняют это, в частности, тем, что в первобытном коллективе
необходимость быстро реагировать на детский плач уменьшала опасность
привлечения хищников и препятствовала особенно активному деторождению.

Источник:
   1. http://www.itogi.ru/telegr2/2012/3/173652.html
   2. http://www.huminfakt.ru/dozor11.html

Вопрос 6:
В комментариях к статье на сайте "Луркмор", посвященной передаче "Поле
чудес", указано, что в статье не хватает ИХ. Согласно афоризму Геннадия
Малкина, применительно к светлым надеждам ИМИ являются дети. Назовите
ИХ.

Ответ:
Рекламная пауза.

Источник:
   1. http://lurkmore.to/Поле_чудес
   2. http://www.gmalkin.ru/work/1026/

Вопрос 7:
То, что одни сочли бы оскорблением, для других - высшее признание. По
замечанию профессора Сухорукова, многие выдающиеся физики так и не
смогли уменьшить ЕЕ. Например, Вернеру фон Сименсу это удалось. Назовите
ЕЕ четырьмя словами.

Ответ:
Первая буква своей фамилии.

Зачет:
Заглавная буква своей фамилии.

Комментарий:
Многие фамилии ученых увековечены в единицах измерения системы СИ.
Сименс - единица измерения электрической проводимости.

Источник:
   1. ЛОАВ.
   2. http://www.tutoronline.ru/blog/edinicy-izmerenija-nuzhno-znat-v-lico.aspx

Вопрос 8:
(pic: 20120986.jpg)
   В начале Второй мировой войны в Великобритании был создан
агитационный плакат с текстом "Соблюдайте спокойствие и продолжайте в
том же духе". Плакат с небольшими изменениями часто использовался в
2000-х годах. Перед вами вариант, посвященный сериалу, вышедшему в 2010
году. Какое слово закрыто на плакате?

Ответ:
Locked.

Зачет:
Lock.

Комментарий:
При прочтении должно получаться "ШЕРЛОК". Плакат посвящен выходу сериала
"Шерлок".

Источник:
http://www.kinopoisk.ru/level/1/film/502838/

Вопрос 9:
В обращении к читателям редакция журнала отрицает "топтание на месте" и
призывает: "Когда в твоей жизни чего-то не хватает, посмотри вокруг!".
Название журнала совпадает с наименованием метода оценки персонала,
учитывающего мнения о сотруднике его начальников, коллег, подчиненных и
клиентов. Воспроизведите это название, используя минимальное количество
знаков.

Ответ:
360°.

Комментарий:
Журнал призывает совершить полный оборот, оговаривая, что в результате
этого действия вы уже не будете в исходной точке. Метод "360 градусов"
позволяет как бы поместить сотрудника в центр круга из мнений различных
участников рабочего процесса.

Источник:
   1. http://www.360gradusov.lv/magazine/about/
   2. http://www.ippnou.ru/article.php?idarticle=001240

Вопрос 10:
Герой романа Михаила Ахметова привез из Южной Европы очень красивую
кошку редкой породы белого окраса с большими, блестящими, как изумруд,
глазами. Какую кличку он ей придумал?

Ответ:
Belladonna.

Зачет:
Белладонна, Белладонья.

Комментарий:
"Belladonna" [белладонна] происходит от итальянских слов и в переводе на
русский язык означает "красивая женщина". Итальянские дамы закапывали
сок белладонны в глаза для расширения зрачков и придания глазам особого
блеска.

Источник:
   1. Михаил Ахметов. Повелитель джина.
   2. http://ru.wikipedia.org/wiki/Белладонна

Вопрос 11:
По одной из версий, этот известный всем знак восходит к культу Афродиты
Каллипиги, по другой - к надгробному растению, хорошо известному еще
древним грекам. В некоторых вариантах заменой этого знака является
кубок. Воспроизведите этот знак на бланках.

Ответ:
&hearts; ["сердечко"].

Комментарий:
Объясняется происхождение "сердечка" от листа плюща или очертания
женских ягодиц. Каллипига - Прекраснозадая. Заменой масти "черви" в
картах Таро является "кубок".

Источник:
   1. http://origin.iknowit.ru/paper1239.html
   2. http://www.za-adonay.ru/nayti_svoyu_lyubov/simvolyi_otkuda_proishodit_izobrazhenie_serdtsa/
   3. http://ru.wikipedia.org/wiki/Символ_сердца
   4. http://www.vokrugsveta.ru/quiz/678/
   5. http://ru.wikipedia.org/wiki/Черви_(масть)

Вопрос 12:
Интервьюируя футболиста ЦСКА Томаша Нецида, корреспондент
"Спорт-Экспресс" поинтересовался, отдает ли тот предпочтение "Барселоне"
перед "Реалом" или же наоборот. Мы не уверены, что Нецид понял вопрос.
Мы не знаем, является ли Нецид по структурному гороскопу Котом или
Кабаном. Воспроизведите абсолютно точно ответ футболиста на поставленный
вопрос.

Ответ:
"Я реалист".

Комментарий:
Вероятно, Нецид имел в виду не свою свободу от иллюзий, а предпочтение
клуба "Реал". В соответствии со "структурным гороскопом", к "реалистам"
относятся мужчины, родившиеся в год Кота, Козы и Кабана.

Источник:
   1. http://football.sport-express.ru/reviews/19055/
   2. http://www.xsp.ru/sh/structures/ide/ide.php

Вопрос 13:
По мнению одного остроумного знатока, ОНИ "позволяют одним махом срубить
лес растущих пальцев". Назовите тех, кто в профессиональной практике
разогревает ИХ перед использованием.

Ответ:
Балерины.

Зачет:
Танцовщицы.

Комментарий:
Речь в цитате идет о пуантах.

Источник:
   1. http://club-igc.narod.ru/lib-hum-katax.html
   2. http://ru.wikipedia.org/wiki/Пуанты

Вопрос 14:
По данным Википедии, этот топоним встречается всего шесть раз. Во всех
посвященных топониму отдельных статьях указана численность населения
менее 20 человек - вероятно, остальные действительно получили
образование и разъехались. Данные о повышенной плотности воды в
окрестных водоемах не приводятся. Напишите этот редкий топоним.

Ответ:
[Село] Кукуево. Незачет: Чугуево (в Википедии только один такой
топоним).

Комментарий:
Существует идиома "интеллигенты кукуевского происхождения" и знаменитая
матерная частушка о плывущем топоре из села Кукуева... А плавать топор
мог бы только в очень плотной воде. :-)

Источник:
   1. http://ru.wikipedia.org/wiki/Кукуево
   2. http://www.iknowit.ru/words/word120350.html

Вопрос 15:
В 2011 году компания BBC [Би-Би-Си] организовала специальный семинар по
правилам этикета во время ЭТОГО СОБЫТИЯ. Поводом для семинара послужили
жалобы на бывшего телеведущего Питера Сиссонса, вышедшего в эфир 30
марта 2002 года в бордовом галстуке. К какому событию готовятся в BBC?

Ответ:
К смерти королевы Елизаветы II.

Зачет:
К похоронам в королевской семье и т.п. по смыслу.

Комментарий:
Свадьбы прошли, остались похороны.

Источник:
http://lenta.ru/news/2011/10/31/funeral/

Вопрос 16:
Внимание, в тексте вопроса слово "секс", как ни странно, является
заменой.
   Если для вас, как и для героя книги Олега Стожара, источниками
удовольствия являются путешествия и СЕКС, то вам подойдет примерно такой
замкнутый маршрут: Москва - Пекин - Дели - Дакка - Тегеран - Мехико -
Гранада - Аддис-Абеба - Иерусалим - Гонолулу, причем не важно, из какой
из этих точек вы стартуете. Путешествуйте чаще и напишите, что мы
заменили словом "СЕКС".

Ответ:
Новый год.

Зачет:
Празднование Нового года.

Комментарий:
Перечислены места, где традиционный Новый год празднуется в различное
время с января по декабрь. Замена намекает на известный анекдот "Новый
год бывает чаще, чем секс".

Источник:
   1. http://ru.wikipedia.org/wiki/Новый_год
   2. http://www.poedem.ru/news/3195/
   3. http://www.myworld.cv.ua/nov_god/nov_god.htm

Вопрос 17:
Гипотезу выдвинул венгерский фантаст ФрИдеш КАринти в 1929 году в
рассказе "Звенья цепи". Специальное приложение социальной сети
"ВКонтакте" позволяет проверить гипотезу на практике. Назовите
отечественное произведение, иллюстрирующее эту гипотезу.

Ответ:
[Фильм] "Елки".

Комментарий:
В рассказе предложена "теория шести рукопожатий", согласно которой любые
два жителя планеты могут связаться друг с другом через пять посредников.

Источник:
   1. http://ru.wikipedia.org/wiki/Теория_шести_рукопожатий
   2. http://www.i-fakt.ru/teoriya-shesti-rukopozhatij/
   3. http://ru.wikipedia.org/wiki/Ёлки

Вопрос 18:
Достопримечательностью острова Святой Елены является лестница,
построенная еще в 1829 году. В ней 699 ступеней, она не самая длинная в
мире, но не все туристы отваживаются пройти маршрут до конца.
Путеводители присвоили объекту название из двух слов, совпадающее с
названием триллера 1990 года. Напишите это название.

Ответ:
Лестница Иакова.

Комментарий:
Лестница частично повреждена, и подъем на нее небезопасен.

Источник:
   1. http://www.bugaga.ru/interesting/1146730536-samaya-dlinnaya-lestnica-v-mire.html
   2. http://www.kinopoisk.ru/level/1/film/7355/

Вопрос 19:
[Ведущему: не Лёля, а Леля, через "Е".]
   О роли Лели, дочери славянской богини плодородия Лады, говорит не
только девичий праздник "лельник", но и восходящие к ее имени слова
"лелеять", "лялька", "люлька". О ней же напоминает и посредник, в
народной традиции свидетельствовавший, что Леля не безучастна к своим
почитательницам. Назовите этого посредника.

Ответ:
Аист.

Зачет:
Лелека, лелеко.

Комментарий:
Леля была покровительницей будущих матерей. В ряде славянских языков
аист называется "лелека".

Источник:
   1. http://www.forum.rodnovery.ru/index.php?/topic/2613-леля-жива-марена/
   2. http://www.legendami.ru/bod/russia/russia25.htm
   3. http://ru.wiktionary.org/wiki/лелека

Вопрос 20:
Устали? На одном форуме среди ТАКИХ мобильных телефонов называют Samsung
e2370 и Philips x513. ТАКИЕ изделия, характеризуемые различной угловой
скоростью, выпускались с 1953 года известной подмосковной фабрикой. На
нашем турнире наверняка присутствуют ТАКИЕ игроки. А какие - ТАКИЕ?

Ответ:
Долгоиграющие.

Источник:
   1. http://forum.mobile-review.com/showthread.php?83441-%D1%E0%EC%FB%E9-%E4%EE%EB%E3%EE%E8%E3%F0%E0%FE%F9%E8%E9-%EB%E5%F2%EE-2011
   2. http://ru.wikipedia.org/wiki/Грампластинка

Вопрос 21:
Пробуя себя в боди-арте, графический дизайнер ДжанИн РевЕлл создала
темные фигурные виниловые наклейки, которыми ее модель пользовалась в
течение нескольких кратких сеансов. Английское название данного проекта
можно перевести как "Любить человека". В предыдущем предложении мы
заменили две буквы. Напишите две исходные буквы.

Ответ:
Д, У.

Комментарий:
Основой проекта является "солнечная татуировка", то есть загар не
закрытой наклейками кожи человека. Модели пришлось выдержать несколько
сеансов в солярии. Проект называется "Tan the man" [тэн зэ мэн], глагол
"tan" переводится как "загорать" и как "дубить" - именно дубильные
вещества вызывают потемнение кожи.

Источник:
   1. http://www.kulturologia.ru/blogs/310709/11347/
   2. http://ru.wikipedia.org/wiki/Дубильные_вещества

Вопрос 22:
Для борьбы с ЭТИМ НЕДУГОМ древние римляне готовили канареек во фритюре,
китайцы пьют крепкий зеленый чай, корейцы варят овощной суп с бычьей
кровью, а сотрудники университета Ньюкасла сочли, что можно ограничиться
беконом и хлебом с высоким содержанием углеводов. Мы не спрашиваем, что
предпочитаете вы. Назовите ЭТОТ НЕДУГ.

Ответ:
Похмелье.

Комментарий:
Все перечисленные продукты обладают повышенным содержанием аминокислот.

Источник:
http://www.bugaga.ru/interesting/1146723754-kak-spravitsya-s-pohmelem.html

Вопрос 23:
Для приготовления мцвАди по традиционному рецепту повару понадобятся
гранатовый сок, перец, кориандр, репчатый лук. Согласно одному из новых
рецептов, для мцвади лучше использовать йогурт, киви, чеснок и имбирь.
Каждый готовит по-своему, но известный источник утверждает, что ОНИ
способны испортить блюдо. Назовите ИХ двумя словами.

Ответ:
Женские руки.

Комментарий:
"Мцвади" в переводе с грузинского - "шашлык". Герой фильма "Москва
слезам не верит" утверждал, что "шашлык женских рук не терпит".

Источник:
   1. http://www.shashlick.ru/marinad/marinades/
   2. Фильм "Москва слезам не верит" (1979).

Вопрос 24:
Софи Лорен утверждала, что ЕГО роль в моде играет юбка. На одном из НИХ
наряду с советской символикой неизменно воспроизводилась "Жизнь за
царя". Назовите ЕГО.

Ответ:
Занавес.

Комментарий:
По мнению Софи Лорен, юбка, как занавес, открывает и закрывает ноги. На
менявшихся главных занавесах Большого театра даже в советское время
воспроизводились ноты оперы Глинки. А это был вопрос "под занавес"
нашего пакета.

Источник:
   1. http://aforizmer.ru/aforizmi/o-mode
   2. http://www.bolshoi.ru/about/history/
   3. http://www.vtbrussia.ru/culture/gabt/news/99738/

Тур:
7 тур. "Церебрум" (Москва)

Инфо:
Команда благодарит редактора Олесю Доронину за ценные замечания, а также
Игоря Тюнькина за неоценимую помощь в подготовке пакета.

Вопрос 1:
В одной из серий "Симпсонов" в рукопашной сходятся Халк и Нечто из
великолепной четверки. Назовите страну, внутреннее противостояние
которой описывается.

Ответ:
Ирландия.

Комментарий:
Зеленый и оранжевый - это цвета противоборствующих сторон в Ирландии.
Халк и Нечто - соответственно зеленый и оранжевый.

Источник:
   1. Мультсериал "Симпсоны", s20e04.
   2. http://ru.wikipedia.org/wiki/Халк_(Marvel_Comics)
   3. http://ru.wikipedia.org/wiki/Существо_(Marvel_Comics)

Автор:
Игорь Тюнькин

Вопрос 2:
В одной рекламе ОН, расположенный перед бутылкой пива, превращается в
знак запрета. Назовите ЕГО.

Ответ:
[Автомобильный] руль.

Комментарий:
Иллюстрация принципа: выпил - за руль не садись.

Источник:
Реклама пива "Клинское".

Автор:
Игорь Тюнькин

Вопрос 3:
Один из покемонов представляет собой растение, стилизованное под
человека, которое днем зарывается в землю так, что торчит только
травяная макушка. Считается, что прототипом этого покемона была ОНА,
однако звуковые атаки у покемона отсутствуют. Назовите ЕЕ одним словом.

Ответ:
Мандрагора.

Комментарий:
Покемон зарывается в землю, поэтому его "корень", как и у мандрагоры,
похож на человека. По преданиям, выкопанный корень мандрагоры издает
смертоносный крик. Покемону такой способности не дали.

Источник:
   1. http://wiki.pokeliga.com/Оддиш
   2. http://myfhology.narod.ru/monsters/mandragora.html

Автор:
Игорь Тюнькин

Вопрос 4:
В книге Пратчетта сокольничий находит феникса, однако птица оказывается
слишком опасной. В переводе говорится, что сокольничий испытывал [два
слова пропущено] избавиться от феникса. Восстановите два пропущенных
слова, начинающихся на одну букву.

Ответ:
"... жгучее желание...".

Комментарий:
Жгучее во всех смыслах.

Источник:
Т. Пратчетт. Дамы и господа.

Автор:
Игорь Тюнькин

Вопрос 5:
Одна из серий мультсериала "Пингвины из Мадагаскара", рассказывающая о
лягушке, выделявшей токсичный яд из кожи, называется "ТАКОЙ". Среди
ТАКИХ было много кожевников. Какое слово мы заменили на "ТАКОЙ"?

Ответ:
Неприкасаемый.

Комментарий:
К лягушке не стоило прикасаться, чтобы не отравиться. А с коровьей кожей
в Индии можно было работать только касте неприкасаемых.

Источник:
   1. Мультсериал "Пингвины из Мадагаскара", s01e08.
   2. http://ru.wikipedia.org/wiki/Неприкасаемые_(касты)

Автор:
Игорь Тюнькин

Вопрос 6:
Согласно передаче "Из чего это сделано", ежедневно на завод по
производству пряжи привозят шесть тысяч рулонов овечьей шерсти. В одно
из слов предыдущего предложения мы добавили четыре буквы. Воспроизведите
исходное слово.

Ответ:
Рун.

Комментарий:
Овечьи шкуры измеряются в рунах.

Источник:
Передача "Из чего это сделано", s05e16.

Автор:
Игорь Тюнькин

Вопрос 7:
Рассказывая об открытии элементарных частиц, авторы передачи "100
великих открытий" утверждали, что изучение ПЕРВОЙ ВТОРОГО далеко от
завершения. ПЕРВАЯ отличается от ВТОРОГО добавлением четырех букв. Что
мы заменили на ПЕРВУЮ ВТОРОГО?

Ответ:
Анатомия атома.

Комментарий:
Элементарные частицы - основные составляющие структуры атома. Слово
"анатомия" можно получить, добавив четыре буквы к слову "атом".

Источник:
Дискавери. 100 великих открытий. Химия.

Автор:
Игорь Тюнькин

Вопрос 8:
В одной из серий мультсериала "Черный Плащ" главному герою противостоит
негодяйка, способная замораживать противников. В предыдущем предложении
мы пропустили одну букву в неологизме. Восстановите этот неологизм в
исходном виде.

Ответ:
Снегодяйка.

Комментарий:
Негодяйка, повелевающая снегом.

Источник:
Мультсериал "Черный Плащ", s01e28.

Автор:
Игорь Тюнькин

Вопрос 9:
По словам журналиста "Комсомольской правды", обсуждая итоги одного
мероприятия, состоявшегося в 2004 году, оставшиеся на пятом месте шведы
не унывали: "Все равно победили ОНИ". Назовите ИХ, используя дефис.

Ответ:
Желто-синие.

Зачет:
Желто-голубые.

Комментарий:
Речь идет о конкурсе "Евровидение-2004", победу в котором одержала
украинская певица Руслана.

Источник:
http://dlib.eastview.com/browse/doc/6258040

Автор:
Сергей Цибульский, Юлия Верхолаз

Вопрос 10:
Птица АЛЬФА получила свое название из-за отвратительного вкуса и запаха
ее мяса. Джозеф Хеллер сравнил АЛЬФЫ с торчащими пальцами мертвецов.
Какое слово мы заменили на АЛЬФУ?

Ответ:
Поганка.

Комментарий:
Поганка - это еще и птица, мясо которой непригодно в пищу.

Источник:
   1. http://ru.wikipedia.org/wiki/Поганки
   2. Дж. Хеллер. Уловка-22.

Автор:
Дмитрий Дягилев

Вопрос 11:
Рассуждая о том, что олимпийские объекты должны послужить следующим
поколениям, президент оргкомитета "Сочи-2014" Дмитрий Чернышенко
отметил, что нужно не помять, а оставить память. Какие слова мы заменили
словами "помять" и "память"?

Ответ:
Наследить, наследие.

Источник:
http://www.mk.ru/sport/interview/2011/06/29/601638-dmitriy-chernyishenko-nuzhno-ne-nasledit-a-ostavit-nasledie.html

Автор:
Александр Тобенгауз

Вопрос 12:
Блиц.
   В книге Вагрича Бахчаняна приведены несколько шуточно-парадоксальных
высказываний, приписанных известным людям.
   1. Назовите известного композитора, которому приписывается
высказывание "Мы услышим ангелов...".
   2. Назовите известного поэта, которому приписывается высказывание
"... Мы увидим небо в алмазах".
   3. Назовите "островитянку", которой приписывается высказывание "Быть
можно дельным человеком и думать о красе ногтей".

Ответ:
   1. Бетховен.
   2. Гомер.
   3. Венера Милосская.

Зачет:
   2. Мильтон, Борхес.

Комментарий:
Глухой, слепой и безрукая.

Источник:
http://www.fictionbook.ru/author/bahchanyan_vagrich/muh_uyima_hudojestva_ne_hlebom_ediniym_m/read_online.html?page=3

Автор:
Дмитрий Дягилев

Вопрос 13:
В одном из эпизодов японской игры в жанре "хоррор" "Silent hill" в
невысоком здании внезапно появляется ОН. В некоторых странах ОН иногда
помечается латинской буквой F. Назовите ЕГО двумя словами.

Ответ:
Четвертый этаж.

Комментарий:
Создатели игры - японцы - воспользовались известным суеверием, и самую
жуткую сцену игры сделали на внезапно появившемся четвертом этаже
трехэтажной больницы. Ну а в Корее и Китае из-за аналогичного суеверия
вместо цифры 4 на кнопках лифтов пишут букву "F" (four - четыре
по-английски).

Источник:
   1. ЛОАВ.
   2. http://en.wikipedia.org/wiki/Tetraphobia

Автор:
Дмитрий Дягилев

Вопрос 14:
В мультсериале "Симпсоны" демонстрировался детский прибор, который
озвучивал буквы алфавита. А какую букву зажимал во время сна Гомер
Симпсон?

Ответ:
Z.

Комментарий:
Zzz - обычное описание сна в английском языке.

Источник:
Мультсериал "Симпсоны", s15e13.

Автор:
Игорь Тюнькин

Вопрос 15:
Статья о похищении бронзовой скульптуры, оцененной в полмиллиона фунтов
стерлингов, называлась "Скульптуры ДЕЛАЮТ ЭТО". На одной карикатуре под
названием "Люди ДЕЛАЮТ ЭТО" изображены в том числе провода под
напряжением. Какие три слова мы заменили на "ДЕЛАЮТ ЭТО"?

Ответ:
Гибнут за металл.

Комментарий:
Дорогую скульптуру украли, чтобы сдать в металлолом. Так часто поступают
и с проводами.

Источник:
   1. http://www.kommersant.ru/doc/1843131
   2. http://www.anekdot.ru/id/423623/

Автор:
Иван Фролов

Вопрос 16:
Юбилейный выпуск одного издания продается по очень низкой цене, однако с
лихвой должен окупиться за счет рекламы. Говоря об этом, Антон Ключин
утверждает, что уши [ДВА СЛОВА ПРОПУЩЕНО] по-прежнему держит востро.
Какое английское слово содержится в пропуске?

Ответ:
Playboy.

Зачет:
Плейбой.

Комментарий:
"... уши символ Playboy по-прежнему держит востро". Юбилейный выпуск
журнала "Playboy" продается по рекордно низкой цене, однако их символ
по-прежнему на страже.

Источник:
http://lenta.ru/articles/2011/09/20/playboy/

Автор:
Игорь Тюнькин

Вопрос 17:
(pic: 20120987.jpg)
   Перед вами фрагмент картины Неврева под названием "Неряха". В
названии картины мы заменили одну букву на две. Воспроизведите исходное
название.

Ответ:
"Пряха".

Комментарий:
У девушки на картине под левой рукой моток пряжи, нить идет от левой
руки к правой.

Источник:
http://ru.wikipedia.org/wiki/Файл:Pryaha.jpg

Автор:
Игорь Тюнькин

Вопрос 18:
(pic: 20120988.jpg)
   Перед вами фрагмент фонаря из немецкого города Вайсвассер. Назовите
мастеров, которыми славится этот город.

Ответ:
Стеклодувы.

Зачет:
Выдувщики стекла.

Комментарий:
Полностью фонарь выглядит, как стеклодувы, выдувающие его колбу.

Источник:
   1. Передача "Из чего это сделано", s05e19.
   2. http://commons.wikimedia.org/wiki/File:Glasmacherbrunnen_Wei%C3%9Fwasser_R%C3%BCckseite.JPG

Автор:
Игорь Тюнькин

Вопрос 19:
(pic: 20120989.jpg)
   Перед вами шуточная картинка с политическим уклоном, с которой стерты
все подписи, а одна из четырех частей закрыта. Мы не спрашиваем, что
было изображено на этой закрытой части. Восстановите подпись к ней.

Ответ:
Путинка.

Комментарий:
Изображены жилые дома - сталинка, хрущёвка, брежневка. А на закрытой
части, конечно, водка.

Источник:
http://joyreactor.cc/post/171255

Автор:
Сергей Цибульский, Юлия Верхолаз

Вопрос 20:
(pic: 20120990.jpg)
   В фильме "Необычайные приключения Адель", действие которого
происходит в начале XX века, один из героев, очнувшись после
трехтысячелетнего покоя, выходит из здания и восхищается красотой
площади, изображенной на раздаточном материале, однако замечает, что для
идеала ей не хватает некоторого сооружения. Назовите это сооружение.

Ответ:
Пирамида.

Комментарий:
Ожившая мумия фараона Рамзеса II выходит из Лувра на площадь, где сейчас
находится стеклянная пирамида.

Источник:
Фильм "Необычайные приключения Адель".

Автор:
Сергей Цибульский, Юлия Верхолаз

Вопрос 21:
(pic: 20120991.jpg)
   Воспроизведите подпись к этой картинке с сайта flickr.com, состоящую
из трех английских слов.

Ответ:
Just undo it.

Комментарий:
Стрелка сделана из логотипа "Nike" и напоминает значок отмены
предыдущего действия, т.е. "Undo".

Источник:
http://farm4.staticflickr.com/3221/2311214114_feb8f41eaf.jpg

Автор:
Александр Тобенгауз

Вопрос 22:
   <раздатка>
   Вот еще светило мира,
   Кетчер, друг шампанских вин;
   Перепёр он нам Шекспира
   На <...>.
   </раздатка>
   В эпиграмме И.С. Тургенева на Кетчера, сделавшего перевод Шекспира на
русский в простой прозаической форме, мы пропустили ставшее крылатым
выражение из трех слов. В книге Вагрича Бахчаняна это выражение иронично
приписывается уроженцу средиземноморского города, которому, по
противоречивым сведениям, в 1918 году установили памятник после казни
настоятеля Свияжского монастыря. Назовите имя этого уроженца.

Ответ:
Иуда.

Комментарий:
Пропущенные слова - "язык родных осин", выражение, ставшее крылатым и
обозначающее плохой перевод с иностранного на русский. По одной из
версий, Иуда повесился именно на осине. А памятник Иуде - событие таки
не получившее документальное подтверждение.

Источник:
   1. http://www.bibliotekar.ru/encSlov/26/35.htm
   2. http://www.fictionbook.ru/author/bahchanyan_vagrich/muh_uyima_hudojestva_ne_hlebom_ediniym_m/read_online.html?page=3
   3. http://ru.wikipedia.org/wiki/Иуда_Искариот

Автор:
Дмитрий Дягилев

Вопрос 23:
В начале одной компьютерной игры злобный ОН крадет приз главного героя,
вынуждая того последовать за НИМ в пропасть. В другой компьютерной игре
ОН использует в качестве оружия часы. Назовите ЕГО двумя словами.

Ответ:
Белый кролик.

Зачет:
Белый заяц.

Комментарий:
Как и в "Алисе в Стране чудес", всё начинается с погони за кроликом в
нору.

Источник:
   1. Компьютерная игра "Fance pants".
   2. http://ru.wikipedia.org/wiki/Белый_кролик#.D0.9A.D0.BE.D0.BC.D0.BF.D1.8C.D1.8E.D1.82.D0.B5.D1.80.D0.BD.D1.8B.D0.B5_.D0.B8.D0.B3.D1.80.D1.8B

Автор:
Игорь Тюнькин, в редакции Александра Тобенгауза

Вопрос 24:
   <раздатка>
   В первый раз лежу одна.
   </раздатка>
   В конце мы решили поставить забавный и не самый сложный вопрос.
   Перед вами вторая строка двустишия Юрия Энтина. Воспроизведите первую
строку, представляющую собой устойчивое выражение из двух слов.

Ответ:
Гробовая тишина.

Комментарий:
Надеюсь, после оглашения ответа в зале не установится гробовая тишина.
:-)

Источник:
И. Раскин. Энциклопедия хулиганствующего ортодокса.

Автор:
Александр Тобенгауз

Тур:
8 тур. "Чехардак" (Ярославль)

Инфо:
Команда благодарит за помощь в подготовке пакета и ценные советы Тимура
Сайфуллина (Уфа), Александра Макарова (Орел), Александра Коробейникова
(Санкт-Петербург), а также редактора Олега Холодова (Серпухов).

Вопрос 1:
Персонаж фильма "Хранители" Доктор Манхэттен, бывший ученый-физик,
обладает сверхчеловеческими способностями. Многие считают его человеком
N 1 в современном мире. Что означает его фирменный знак, состоящий из
окружности и двух точек?

Ответ:
Модель атома водорода.

Зачет:
По слову "водород".

Комментарий:
Элемент N 1, как-никак.

Источник:
http://ru.wikipedia.org/wiki/Хранители_(фильм)

Автор:
Арсений Глазовский (Ярославль)

Вопрос 2:
После первого сражения Конана-варвара и ХалАр ЗИма у каждого из них
появилось ЭТО. "ЭТО" - название фильма 1983 года. Назовите этот фильм.

Ответ:
"Лицо со шрамом".

Источник:
   1. Фильм "Conan the Barbarian" (2011).
   2. http://ru.wikipedia.org/wiki/Лицо_со_шрамом_(фильм,_1983)

Автор:
Арсений Глазовский (Ярославль)

Вопрос 3:
Героиня российского фильма 2005 года во время разговора с НИМ
произносит: "Вы как будто вальс танцуете!". Назовите ЕГО.

Ответ:
[Эраст] Фандорин.

Комментарий:
Обыгрывается манера Фандорина расставлять всё по пунктам "это раз... это
два... это три...".

Источник:
Фильм "Турецкий гамбит" (2005).

Автор:
Арсений Глазовский (Ярославль)

Вопрос 4:
Закончите четырьмя словами высказывание Владимира Софроницкого о Льве
Оборине: "Велик Оборин - он...".

Ответ:
"... и робок, и Лев".

Комментарий:
Старый добрый палиндром.

Источник:
http://www.ng.ru/culture/2007-09-13/11_oborin.html

Автор:
Юлия Кулакова, в редакции Арсения Глазовского (оба - Ярославль)

Вопрос 5:
Автор вопроса назвал подачу заявки на марафон АЛЬФОЙ ИГРЕК подготовки
пакета. В описании работы трамвая можно прочитать, что АЛЬФА - это
провода, а ИГРЕК - рельсы. Что мы заменили на "АЛЬФА ИГРЕК"?

Ответ:
Фаза ноль.

Источник:
   1. Больная фантазия автора вопроса.
   2. Общие знания.

Автор:
Арсений Глазовский (Ярославль)

Вопрос 6:
Одна из песен группы "Ундервуд" называется "Вишневый ОН". Известно, что
ОН занял пост своего отца, став королевским генеральным наместником в
провинциях БрессЕ, БюжЕ, ВальромЕ и Же. Назовите ЕГО.

Ответ:
[Маркиз] де Сад.

Источник:
   1. http://www.oduvanchik.net/view_song.php?id=5676
   2. http://ru.wikipedia.org/wiki/Маркиз_де_Сад

Автор:
Арсений Глазовский (Ярославль)

Вопрос 7:
Комментатор матча "Локомотив" - "Андерлехт" в одном из эпизодов упомянул
ЕГО, после чего вынужден был оговориться, что футбольная команда тут ни
при чем. ОН для экологически чистых продуктов называется "Eco Garantie".
Назовите ЕГО двумя словами.

Ответ:
Бельгийский стандарт.

Комментарий:
"Стандард" - это другая бельгийская команда.

Источник:
   1. Матч "Локомотив" - "Андерлехт", 29.09.2011 г.
   2. http://www.naturale-shop.ru/tags/eco-garantie/

Автор:
Иван Плаксин, в редакции Арсения Глазовского (оба - Ярославль)

Вопрос 8:
Рэймонд СмАллиан написал в 1978 году книгу. Напишите "Как же называется
эта книга?".

Ответ:
"Как же называется эта книга?".

Комментарий:
Своеобразный вопрос-шутка на школу.

Источник:
http://golovolomka.hobby.ru/books/smullian/name/content.shtml

Автор:
Арсений Глазовский (Ярославль)

Вопрос 9:
Одна из наружных реклам "Макдональдса" представляет собой висящий в
воздухе огромный кофейник, из которого в стоящий на асфальте стакан
льется струйка кофе. Ответьте, что стало основой этой рекламной
конструкции, зная, что вы могли неоднократно видеть это, когда
добирались на марафон.

Ответ:
Фонарь.

Зачет:
Фонарный столб и т.п. по смыслу.

Комментарий:
Изогнутый фонарный столб напомнил создателям рекламы струю кофе,
льющуюся в стакан, а сама лампочка - кофейник.

Источник:
http://www.adme.ru/tvorchestvo-reklama/gigantskaya-reklama-347805/

Автор:
Наталья Масленникова (Ярославль)

Вопрос 10:
На сайте fishki.net [фишки точка нет] есть шуточная инструкция,
рассказывающая о том, как сделать из НЕЕ собаку. В рекламном слогане
одной транспортной компании говорится, что ОНА "задушит жабу". Назовите
ЕЕ.

Ответ:
Такса.

Комментарий:
Инструкция, аналогичная изготовлению собак из воздушных шариков, только
вместо шарика - такса. Рекламный слоган такси звучит так: "Наша такса
задушит вашу жабу".

Источник:
   1. http://ru.fishki.net/picsw/092011/13/pics/pics_006.jpg
   2. http://www.taxi-taxa.spb.ru/

Автор:
Арсений Глазовский (Ярославль)

Вопрос 11:
В одной юмористической передаче к первой половине известной фразы было
придумано окончание: "законодательная, исполнительная и судебная".
Напишите первые четыре слова этой фразы, получившей известность
благодаря Марку Твену.

Ответ:
"Есть три вида лжи...".

Источник:
   1. "Бункер News", эфир от 10.11.2012 г.
   2. http://ru.wikipedia.org/wiki/Ложь,_наглая_ложь_и_статистика

Автор:
Арсений Глазовский (Ярославль)

Вопрос 12:
Вторая и менее известная часть этого знаменитого изречения - "здоровье в
воде". Назовите отечественного поэта, который закончил одно из своих
самых известных стихотворений первой частью изречения.

Ответ:
Александр Блок.

Комментарий:
Полностью выражение звучит на латыни так: "In vino veritas, in aqua
sanitas". (Истина в вине, здоровье в воде). Стихотворение Блока
"Незнакомка" заканчивается словами "Я знаю, истина в вине".

Источник:
   1. http://ru.wikipedia.org/wiki/In_vino_veritas
   2. http://slova.org.ru/blok/neznakomka/

Автор:
Юлия Кулакова (Ярославль)

Вопрос 13:
В день рождения Владимира Путина получили широкое распространение
сообщения, включающие выражение "спасибопутинузаэто". Зачастую поводы
для "благодарности" были достаточно абсурдны. В одном из сообщений к
выражению просто добавили два символа. Какие?

Ответ:
'о.

Комментарий:
Полное сообщение - Спасибопутинузаэто'о, автор намекает на камерунского
нападающего футбольного клуба "Анжи" Самюэля Это'О.

Источник:
http://ura.ru/content/svrd/07-10-2011/news/1052135169.html

Автор:
Арсений Глазовский (Ярославль)

Вопрос 14:
Внимание, в вопросе есть замена.
   Когда одного студента спросили, что такое ИКС, он ответил: ИКС - это
такое слово, которое пишем вместо того, правописание которого не знаем.
У слова "ИКС" нет ИКСОВ. Какое слово греческого происхождения мы
заменили на ИКС в этом вопросе?

Ответ:
Синоним.

Источник:
http://nature.web.ru/db/msg.html?mid=1154178

Автор:
Наталья Масленникова (Ярославль)

Вопрос 15:
На асфальте рядом с рекламным щитом в форме НЕГО авторы рекламы
разместили муляжи, изображающие половинки птиц. "ОНО" - роман 1963 года.
Назовите ЕГО двумя словами.

Ответ:
Лезвие бритвы.

Комментарий:
Авторы хотели показать, что рекламный щит в форме лезвия бритвы
настолько острый, что разрезает даже садящихся на него птиц. "Лезвие
бритвы" - роман советского писателя Ивана Ефремова.

Источник:
   1. http://www.adme.ru/tvorchestvo-reklama/gigantskaya-reklama-347805/
   2. http://ru.wikipedia.org/wiki/Лезвие_бритвы_(роман,_1963)

Автор:
Наталья Масленникова (Ярославль)

Вопрос 16:
Когда Гарри Поттер пытается освоить Оглушающее заклинание, недовольный
Рон по сути предлагает ДЕЛАТЬ ЭТО. ДЕЛАТЬ ЭТО предлагал своему другу и
герой советского фильма. Что мы заменили словами "ДЕЛАТЬ ЭТО"?

Ответ:
Тренироваться на кошках.

Комментарий:
Гарри тренирует заклинание на Роне, поэтому тот и недоволен.
Справедливости ради, Рон предлагает тренироваться только на одной кошке
- Миссис Норрис, но особой роли для взятия вопроса это не играет.

Источник:
   1. Дж. Роулинг. Гарри Поттер и Кубок Огня (в переводе Марины
Литвиновой).
   2. Фильм "Операция "Ы" и другие приключения Шурика".

Автор:
Анна Смирнова (Ярославль)

Вопрос 17:
(pic: 20120992.jpg)
   На англоязычном сайте brainlesstales.com выкладываются комиксы, в
которых не слишком остроумно обыгрываются англоязычные слова, фразы и
названия. Восстановите фразу персонажа слева двумя или четырьмя словами.

Ответ:
I phone, you tube.

Зачет:
iPhone, YouTube.

Источник:
http://www.brainlesstales.com/2011-01-27/i-phone-you-tube

Автор:
Юрий Селин (Ярославль)

Вопрос 18:
(pic: 20120993.jpg)
   Перед вами фотография автомобильного номера, прикрученного к машине.
Догадавшись, в чем его особенность, назовите отличительный признак
водителя данной машины.

Ответ:
Блондин[ка].

Зачет:
По смыслу.

Комментарий:
Если перевернуть номер, можно увидеть текст "I'm blond".

Источник:
(pic: 20120994.jpg)

Автор:
Юрий Селин (Ярославль)

Вопрос 19:
   <раздатка>
   Костя согласился финансировать этот проект и все остальные проекты в
моей жизни тоже!
   &nbsp;
   Не так всё было... Совсем не так!
   </раздатка>
   "Костя согласился финансировать этот проект и все остальные проекты в
моей жизни тоже!" в романе Оксаны Робски и "Не так всё было... Совсем не
так!" в повести Стругацких являются ИМИ. У ярославской церкви Иоанна
Предтечи ИХ пятнадцать. Назовите ИХ одним словом.

Ответ:
Главы.

Источник:
   1. О. Робски. Casual.
   2. А. и Б. Стругацкие. Отягощенные злом, или 40 лет спустя.
   3. http://www.360yaroslavl.ru/rus/yaroslavl/cerkov-ioanna-predtechi

Автор:
Юлия Кулакова, в редакции Арсения Глазовского (оба - Ярославль)

Вопрос 20:
Согласно одному анекдоту, в серию "Собрание сочинений ЕГО" входят книги
"Как завести друзей" и "Российские полупроводники". Назовите ЕГО имя и
фамилию.

Ответ:
Иван Сусанин.

Источник:
http://www.anekdot.ru/id/217623/

Автор:
Наталья Масленникова (Ярославль)

Вопрос 21:
Благодаря проекту журнала "Bulletin of Atomic Scientists" [бУлитин оф
этОмик сАйентистс], можно сказать, что в фильме "Хранители" мир
находится почти в состоянии ядерной войны. В предыдущем предложении мы
заменили три слова одним. Через минуту напишите эти три слова.

Ответ:
Без пяти минут.

Комментарий:
Речь о проекте "Часы Судного дня". Периодически на обложке журнала
публикуется изображение часов, показывающих без нескольких минут
полночь. Время, оставшееся до полуночи, символизирует напряженность
международной обстановки и прогресс в развитии ядерного вооружения. Сама
полночь символизирует момент ядерного катаклизма. В фильме часы
показывают без пяти минут полночь.

Источник:
   1. http://ru.wikipedia.org/wiki/Хранители_(фильм)
   2. http://ru.wikipedia.org/wiki/Часы_Судного_дня

Автор:
Арсений Глазовский (Ярославль)

Вопрос 22:
На одной современной карикатуре изображен волк, собирающийся разрушить
домик поросят. Также на карикатуре есть ОНИ. Назовите ИХ двумя словами
по-русски или по-английски.

Ответ:
Angry birds.

Зачет:
Злые птицы.

Комментарий:
Пожалуй, самое известное "средство" разрушения конструкций с поросятами.

Источник:
(pic: 20120995.jpg)

Автор:
Иван Плаксин, в редакции Арсения Глазовского (оба - Ярославль)

Вопрос 23:
Персонаж фильма "Крестный отец" узнаёт, что в деревне совсем нет мужчин
и что в этом виновата ОНА. А что является ее "символом" согласно
произведению 2006 года?

Ответ:
V.

Комментарий:
ОНА - вендетта. Фильм 2006 года - "V - значит вендетта".

Источник:
   1. Фильм "Крестный отец".
   2. http://ru.wikipedia.org/wiki/V_%E2%80%94_значит_вендетта_(фильм)

Автор:
Арсений Глазовский (Ярославль)

Вопрос 24:
Героиня фильма "Зубная фея" оказывается в месте, которое находится между
Раем и Адом, и видит там лифт, название которого представляет собой
англоязычный неологизм. Напишите этот неологизм.

Ответ:
Hellevator.

Комментарий:
Лифт, как несложно догадаться, едет вниз.

Источник:
Фильм "Зубная фея" (1997).

Автор:
Анна Смирнова (Ярославль)

Тур:
9 тур. "Немчиновка" (Москва)

Вопрос 1:
Шарада.
   Объявлен в розыск:
   Классный кашевар, порой не очень мил,
   умеет точно распознать: кто, что, куда носил.
   Коль не опухла голова,
   слог первый вам даю сперва:
   поднять что можно без рывка,
   но выносить не след.
   Второй вам даст одна река,
   чья рифма будет вам легка.

Ответ:
Сор-ока.

Вопрос 2:
Журналист телеканала "НТВ+", рассуждая о предстоящем старте клуба
"Валенсия" в чемпионате Испании и Лиге чемпионов, заметил, что
"Валенсии", потерявшей в межсезонье своих лидеров Сильву и Вилью, будет
трудно противостоять футбольным гигантам. Какое слово мы заменили словом
"гигантам"?

Ответ:
Голиафам.

Комментарий:
Без Давида Вильи и Давида Сильвы с Голиафами справиться некому.

Источник:
Программа "Футбольный клуб".

Вопрос 3:
На берегу реки сидят юноша с девушкой и маленькое существо и боятся
одного и того же. Кого?

Ответ:
Аиста.

Комментарий:
Лягушка тоже боится.

Источник:
Анекдот.

Вопрос 4:
Британский адмирал Джон Арбетнот Фишер известен как человек, совершивший
революцию в области военно-морского кораблестроения. В 1909 году он
получил титул барона и поместил на своем гербе девиз "Fear God and...".
Воспроизведите два последних слова девиза.

Ответ:
"... dread nought".

Зачет:
В русской транскрипции.

Комментарий:
Под руководством Фишера был построен линкор нового типа "Дредноут".
Dread - ужас, nought - ничто, т.е. неустрашимый.

Источник:
http://www.vokrugsveta.ru/vs/article/6528/

Вопрос 5:
Шарада.
   Объявлен в розыск: слово из морей,
   большое как король зверей.
   Вот две подсказки для того,
   кто отгадать рискнет его:
   1. Она полезна, но порой совсем не лезет в рот,
   ее на завтрак подают, а после бутерброд.
   2. А после завтрака вдвоем идем на променад,
   нас галереи с тобой ждут и в них они стоят.
   Когда стоят, когда висят,
   нам молотки про них стучат.

Ответ:
Каша-лот.

Вопрос 6:
Внимание, в вопросе есть замены.
   Вспоминая о 60-х годах прошлого века, ХОЛОДНЕНЬКИЙ пишет, что мир с
тех пор перевернулся - тогда ПЕРВЫЙ всегда был ХОЛОДНЫМ, а ВТОРОЙ -
ГОРЯЧИМ. ВТОРЫХ сейчас несколько, поэтому назовите нынешнего ПЕРВОГО.

Ответ:
[Барак] Обама.

Комментарий:
Все президенты США были белыми, а чемпионы мира по боксу в тяжелом весе
- черными. Сейчас всё наоборот. Александр Беленький - известный
журналист, пишущий о боксе.

Источник:
http://sport.rambler.ru/news/boxing/569467780.html

Вопрос 7:
Тамильские дети обучаются счету с помощью особой песни. Что,
называющееся по-тамильски "sa ri ga ma pa da ni" [са-ри-га-ма-па-да-ни],
используется для запоминания числа 7?

Ответ:
Ноты.

Источник:
http://en.wikipedia.org/wiki/Tamil_numerals

Вопрос 8:
В 1995 году спецподразделениями МВД РФ была проведена масштабная
операция под кодовым названием "Закат". Назовите максимально точно
основную цель этой операции.

Ответ:
Ликвидация "солнцевской" преступной группировки.

Зачет:
По упоминанию "солнцевских".

Вопрос 9:
Шарада.
   В розыск объявлен: забавный зверек.
   Для всех, кто знаком - симпатичен,
   к тому же еще весьма символичен.
   Довольно большой, двуцветный окрас.
   Надеюсь, уже всё понятно сейчас.
   Теперь же подсказки пошли по слогам:
   Слог первый почти не встречается нам.
   Он взрослый мужчина не нашей страны,
   с которой частенько ссорились мы.
   Второй слог вариантов не оставляет,
   совсем однозначно на всё отвечает.

Ответ:
Пан-да.

Вопрос 10:
[Ведущему: отточия не озвучивать.]
   Актриса Мусина-Пушкина писала Чехову: "Сегодня ... я ... встретила
Вас ..., едущего на извозчике, причем Вы смотрели прямо на меня, но
почему-то не удостоили поклона". Причиной такого поведения писателя была
случившаяся с ним незадолго до этого неприятность. О какой неприятности
идет речь?

Ответ:
Сломалось пенсне.

Комментарий:
Он ее просто не узнал.

Источник:
Дональд Рейфилд. Жизнь Антона Чехова. - М.: Б.Г.С.-Пресс, 2007.

Вопрос 11:
Все вы ИХ, несомненно видели, и знаете - ОНИ очень странные! То, что ОНИ
делают стоя, стоя кроме них никто делать толком не умеет, даже человек.
Еще в одном положении у НИХ пребывает совсем не та половина, о которой
принято думать. А глаза и хвост у них - почти такие же, как у
разновидности ящериц. Так кто же ОНИ?

Ответ:
Морские коньки.

Комментарий:
Плавают стоя; беременными становятся самцы; независимые глаза и
хвост-прицепка, как у некоторых хамелеонов.

Вопрос 12:
Журналист Евгений Дзичковский пишет, что на матче Лиги чемпионов в
Лужниках присутствовало около 500 болельщиков "Интера". Правда, он не
знает, сколько среди них было доморощенных российских [пропуск], а
сколько всамделишных [пропуск]. Напишите оба пропущенных слова,
отличающихся одной буквой.

Ответ:
"... интеристов...", "... интуристов".

Источник:
http://www.sport-express.ru/newspaper/2010-04-07/3_1/

Вопрос 13:
Шарада:
   Объявлена в розыск: проселочная дорога,
   что вдалеке и петляет немного.
   Еще слово местность порой означает,
   и юноши девушек там ожидают.
   Две части у слова, подсказок - две
   и обе держу у себя в голове.
   Первая часть интересна уж тем,
   что можно ее выставлять на обмен.
   Хочешь - не хочешь сделать приятно:
   Такой же предмет получишь обратно.
   Подсказку вторую в команде получишь,
   если улыбки скрыть не приучишь!

Ответ:
Око-лица.

Вопрос 14:
Недолюбливавший эту женщину Корней Чуковский утверждал, что на самом
деле ее муж произнес: "Эх, стерва!". Назовите фамилию этой женщины.

Ответ:
Книппер-Чехова.

Комментарий:
""Их штербе", - сказал Чехов, умирая. Это очень волнует праздный русский
ум: почему по-немецки? Отрезвляя гипотезы, я утверждал, что потому, что
доктор рядом был немец, и он сообщил ему свое мнение как врач врачу.
Недавно я услышал даже, что он сказал чисто по-русски: "Эх, стерва!" -
имея в виду то ли жизнь, то ли жену" (Битов).

Источник:
http://magazines.russ.ru/zvezda/2011/8/zh22.html

Вопрос 15:
Автор вопроса увидел фотографию забавного и весьма практичного
изобретения: плитка шоколада, поделенная на дольки разного размера, на
каждой из которых написано число в пределах нескольких десятков, которое
должно сопровождаться четырьмя буквами. Напишите эти буквы.

Ответ:
ккал.

Зачет:
kcal.

Источник:
http://24mix.ru/4938 (на фото, к сожалению, ошибка - должно быть kcal
или, на худой конец, Cal).

Вопрос 16:
Выдающийся борец сумо йокодзуна Асасёрю был вынужден завершить карьеру
после драки в ночном клубе. Заметка об этом на сайте japan-sumo.ru была
озаглавлена так: "Синего дракона сгубил...". Закончите название статьи.

Ответ:
"... зеленый змий".

Источник:
http://www.japan-sumo.ru/asaintai.html

Вопрос 17:
Шарада.
   Объявлен в розыск: объект эфемерный,
   но без него неизбежны потери.
   Кто же его потеряет напрасно,
   будет того положенье несчастно.
   Вас не хотел ввергать в азарт,
   но первый слог коснется карт.
   Уж если с ним судьба свела,
   Он ляжет во главе стола,
   А если ляжет с краю вдруг,
   не разыграйтесь, милый друг!
   Второй слог, букву получив,
   живет в сети. Не покормив,
   вы не узнаете, что он
   умен, и ловок и силен!
   Он в ваш залезет разговор, всем принеся урон,
   и часто, сдачу получив, всё лезет на рожон.

Ответ:
Кон-троль.

Вопрос 18:
В логотипе одного из тематических спутниковых телеканалов две буквы
изображены в виде стилизованных серпа и молота. Напишите название этого
канала.

Ответ:
"Ностальгия".

Комментарий:
Канал посвящен советским временам, на серп и молот похожи соседние буквы
"С" и "Т".

Источник:
http://www.nostalgiatv.ru/

Вопрос 19:
"Магазин на диване, ресторан на диване, работа на диване, дом на диване,
дискотека на диване... Вся жизнь на диване! Такая уж она у нас...".
Закончите анекдот одним словом.

Ответ:
"... клопов".

Источник:
http://www.anekdot.ru/id/442527/

Вопрос 20:
Однажды, будучи за границей, Джеффри Херст вручил подарок - майку с
надписью "Большое спасибо". На каком языке была сделана эта надпись?

Ответ:
На азербайджанском.

Комментарий:
Автор победного гола финала ЧМ-1966 подарил футболку сыну Тофика
Бахрамова - арбитра, засчитавшего англичанам победный гол.

Вопрос 21:
Говоря о чем-то непонятном, русские употребляют словосочетание
"китайская грамота", англичане упоминают греческий и т.д. Аналогичная
идиома есть и в эсперанто. Какой язык в ней фигурирует?

Ответ:
Волапюк.

Комментарий:
Как же не поддеть конкурентов.

Вопрос 22:
Скульптор Бальдаччини в свое время крайне негативно отзывался о работе
своего коллеги Поля Бельмондо. Это привело к скандалу, случившемуся в
1989 году. Назовите имя Бальдаччини.

Ответ:
Сезар.

Комментарий:
Сын Поля, актер Жан-Поль Бельмондо отказался от кинопремии "Сезар",
названной в честь Бальдаччини.

Вопрос 23:
Этот эпизод стал известным благодаря служившим в России голландцам
Стрейсу и Фабрициусу, причем первый описывает пьяную выходку, а второй -
языческое жертвоприношение. По мотивам этих событий было написано
известное стихотворение Дмитрия Садовникова. Процитируйте первую строчку
этого стихотворения.

Ответ:
"Из-за острова на стрежень...".

Источник:
http://www.razdory-museum.ru/c_razin-1.html

Вопрос 24:
Прослушайте эпиграмму Анатолия Белкина:
   В том и сила организма,
   Что ничто не пропадет:
   Из отходов маньеризма
   Варится [пропуск].
   Мы не спрашиваем, кому посвящена эпиграмма. Заполните пропуск.

Ответ:
"Бахыт-компот".

Источник:
А. Белкин. Эпиграмматика.

Тур:
10 тур. "Перманентный фейспалм" (Санкт-Петербург)

Вопрос 1:
Прослушайте часть стихотворения Алексея Зё:
   "Ну Вы представляете, Фридрих, -
   Смущенно сказал Мануил, -
   Мой стол, зарычав и запрыгав,
   Меня искусал и избил!"
   А стоя у Канта в прихожей
   И глядя на мебельный бег,
   Герр Ницше смеялся: "Похоже,
   Что данная...".
   Закончите стихотворение, добавив частицу в известный философский
термин.

Ответ:
"... вещь-не-в-себе".

Комментарий:
Этим вопросом команда "Перманентный фэйспалм" приветствует вас и заодно
предупреждает о том, что последует далее.

Источник:
http://www.stihi.ru/2005/12/23-2147/

Автор:
Ольга Звягинцева (Санкт-Петербург)

Вопрос 2:
(pic: 20120996.jpg)
   На этой фотографии изображен фрагмент трамвайных путей в
Новосибирске, который находится в ужасном состоянии. С ним и в прямом, и
в переносном смысле СДЕЛАЛИ ЭТО. Ответьте двумя словами, что именно.

Ответ:
Положили болт.

Зачет:
Забили болт.

Комментарий:
Вместо того чтобы заварить стык, трамвайщики просто положили туда болт.
Естественно, это, мягко говоря, ненормально.

Источник:
http://www.transphoto.ru/photo/178759/

Автор:
Евгений Капитульский (Рига - Санкт-Петербург)

Вопрос 3:
Статья Википедии про это "маломощное оружие для метания" содержит по
большей части физические и климатические данные. Назовите необходимый
для изготовления этого оружия материал.

Ответ:
Снег.

Комментарий:
Снежки.

Источник:
http://ru.wikipedia.org/wiki/Снежки

Автор:
Юрий Петров (Санкт-Петербург)

Вопрос 4:
По случаю матча "Суонси" - "Арсенал" Сергей Бондаренко процитировал
название советского фильма 1980 года. Видимо, просьба была адресована
"Арсеналу". Назовите этот фильм.

Ответ:
"Не стреляйте в белых лебедей".

Комментарий:
Широко известно, что "Арсенал" называют "ганнерами". Менее известно (но
зато выводится из названия команды), что "Суонси" называют "лебедями",
да к тому же они чаще всего играют в белом.

Источник:
   1. http://www.sports.ru/profile/4260361/status/136768/
   2. http://ru.wikipedia.org/wiki/Не_стреляйте_в_белых_лебедей_(фильм)
   3. http://en.wikipedia.org/wiki/Arsenal_F.C.
   4. http://en.wikipedia.org/wiki/Swansea_City_A.F.C.

Автор:
Евгений Капитульский (Рига - Санкт-Петербург)

Вопрос 5:
Иногда в начале необходимо преодолеть ТАКУЮ АЛЬФУ. Однако при этом важно
не пропустить ТАКУЮ ОМЕГУ, которую мы, впрочем, привыкли называть
соответствующим английским словом. ОМЕГА по определению является
множеством АЛЬФ. Назовите ТАКУЮ АЛЬФУ.

Ответ:
Мертвая точка.

Комментарий:
ТАКАЯ АЛЬФА - мертвая точка, с которой иногда так трудно сдвинуться.
ТАКАЯ ОМЕГА - мертвая линия, она же дедлайн. Линия, как известно,
состоит из точек.

Автор:
Алина Малиновская (Санкт-Петербург)

Вопрос 6:
В XIX веке один ученый пытался доказать, что венгерское письмо является
иероглифическим. Для примера он использовал венгерское слово
"k&uacute;t" [кут]. Он утверждал, что буква "k" [ка] является
изображением человека, а буква "t" [тэ] - некой птицы. Догадавшись,
какой объект, по его мнению, символизировала буква "u", переведите слово
"k&uacute;t" на русский язык.

Ответ:
Колодец.

Комментарий:
Буква "t" - это изображение колодезного журавля, а буква "u"
символизирует яму. Всё вместе - колодец.

Источник:
   1. В. Томсен. История языковедения до конца XIX века. - М.: Учпедгиз,
1938. - С. 31.
   2. http://hungarian_russian.academic.ru/6728/

Автор:
Евгений Капитульский (Рига - Санкт-Петербург)

Вопрос 7:
   <раздатка>
   Отныне желающие проехать между Москвой и Питером на поезде в середине
дня должны будут выложить в кошелек РЖД уже не 520 рублей, а 2044 рубля
в первую неделю курсирования нового поезда, 2460 рублей до 15 апреля
2010 года и 2670 рублей - после 15 апреля. Утренний и вечерний "Сапсаны"
обойдутся еще дороже.
   </раздатка>
   Статья в ЖЖ-сообществе ru_travel, отрывок из которой вы видите перед
собой, называется "ОН". Другой ОН звучит так: "24 сентября, Ясная
Поляна". Назовите ЕГО двумя словами.

Ответ:
Конец "Юности".

Комментарий:
"Юность" - не только произведение Льва Николаевича Толстого, но и очень
известный сидячий поезд Санкт-Петербург - Москва эконом-класса, на
некоторое время отмененный с пуском весьма сомнительного проекта
"Сапсан".

Источник:
   1. http://ru-travel.livejournal.com/12962606.html
   2. http://az.lib.ru/t/tolstoj_lew_nikolaewich/text_0030.shtml

Автор:
Георгий Федосов (Нижний Новгород)

Вопрос 8:
ОНИ бывают очень разными: одни различаются формой и доставили Гарри
некоторые проблемы, другие измеряются в битах, третьи - в миллиметрах
или дюймах, четвертые часто дают начало, а пятые можно найти в сводах
зданий. А какие обычно используются только в начале?

Ответ:
Скрипичные.

Зачет:
Басовые, музыкальные.

Источник:
   1. http://ru.wikipedia.org/wiki/Замковый_ключ
   2. http://ru.wikipedia.org/wiki/Ключ_(криптография)
   3. http://ru.wikipedia.org/wiki/Гаечный_ключ
   4. http://dic.academic.ru/dic.nsf/ogegova/277839/
   5. Фильм "Гарри Поттер и философский камень".

Автор:
Райтис Нугуманов, по идее Алисы Петровец (оба - Рига)

Вопрос 9:
На одном сайте есть специальный блог, в котором каждый день публикуются
однотипные фотографии знаменитостей и сотрудников сайта. Название этого
блога состоит из двух слов и совпадает с названием города. Напишите его.

Ответ:
Великие луки.

Комментарий:
Там хипстерские фотографии. А хипстеры сами знаете что делают с луками.

Источник:
http://www.sports.ru/tribuna/blogs/thegreatestlooks/

Автор:
Евгений Капитульский (Рига - Санкт-Петербург)

Вопрос 10:
Пирожки - это четверостишия без рифмы. Прослушайте пирожок ЖЖ-юзера
road_man:
   сорокалетний буратино
   с надеждой едет на кавказ
   он одинок ему сказали
   там могут выдолбить семью
   Какое слово мы заменили на "выдолбить"?

Ответ:
Вырезать.

Источник:
http://pirozhki-ru.livejournal.com/1188442.html

Автор:
Ольга Звягинцева (Санкт-Петербург)

Вопрос 11:
   <раздатка>
   FROM ______ ______
   </раздатка>
   Знакомому автора вопроса в самолете авиакомпании "Сибирь" выдали
брошюру, приглашавшую совершить определенный внутрироссийский рейс. В
заголовке этой брошюры мы пропустили два фрагмента, которые отличаются
друг от друга только наличием пробела. Восстановите их в правильном
порядке.

Ответ:
TOMSK, TO MSK.

Комментарий:
Рейс был Томск - Москва. А также обратно, но это роли не играет.

Источник:
ЛОАВ.

Автор:
Евгений Капитульский (Рига - Санкт-Петербург)

Вопрос 12:
Внимание, в вопросе есть замена.
   Одна актриса взяла псевдоним потому, что не хотела быть СЯКОЙ дочкой.
Ее настоящая фамилия - ТАКАЯ. Назовите эту фамилию.

Ответ:
Мамина.

Комментарий:
Ее отец - режиссер Юрий Мамин, она не хотела быть папиной дочкой. А сама
она известна как Катерина Ксеньева.

Источник:
   1. http://ru.wikipedia.org/wiki/Ксеньева,_Катерина_Юрьевна
   2. Ее интервью газете "Метро".

Автор:
Евгений Капитульский (Рига - Санкт-Петербург)

Вопрос 13:
Герои одной английской комедии 1990 года, скрывающиеся от правосудия,
запоминают порядок выполнения некоторого действия следующим образом: лоб
- пупок - кошелек - часы. Что это было за действие?

Ответ:
Крестное знамение.

Зачет:
По смыслу.

Комментарий:
Герои переоделись монашками.

Источник:
Фильм "Монахини в бегах".

Автор:
Алина Малиновская (Санкт-Петербург)

Вопрос 14:
Статья "Пять минут перед боем" рассказывает о том, что впервые было
написано Редьярдом Киплингом в 1932 году по заказу короля. Какое слово
иностранного происхождения мы убрали из названия статьи?

Ответ:
Курантов.

Комментарий:
Статья рассказывает о новогоднем обращении главы государства. Первую
такую речь в истории в 1932 году написал Киплинг для Георга V.

Источник:
http://www.rg.ru/2011/12/29/obrashenie.html

Автор:
Евгений Капитульский (Рига - Санкт-Петербург), при участии неизвестного
водителя попутной машины

Вопрос 15:
Автору пришлось добавить в прошлый вопрос слова "иностранного
происхождения" потому, что тестер не смог СДЕЛАТЬ ЭТО. На футболиста
Мирко Вучинича недавно напали грабители и СДЕЛАЛИ ЭТО. Какие два слова
мы заменили на "СДЕЛАТЬ ЭТО"?

Ответ:
Отобрать часы.

Источник:
   1. Лог скайпа автора предыдущего вопроса и тестера.
   2. http://www.allsport-news.com/news.php?readmore=22472

Автор:
Евгений Капитульский (Рига - Санкт-Петербург)

Вопрос 16:
Блиц.
   1. Отгадайте латышскую загадку: "Малюсенький бочоночек, да пиво двух
видов".
   2. Отгадайте латышскую загадку: "Матушка малюсенька, да платье в
одних заплатках".
   3. Отгадайте латышскую загадку: "Хозяин без книги поет".

Ответ:
   1. Яйцо.
   2. Курица.
   3. Петух.

Комментарий:
Начав "ab ovo", логично продолжить курицей и петухом. :-)

Источник:
   1. Автор вопроса - латыш в N-м поколении :-)
   2. Сборники загадок на латышском языке.

Автор:
Райтис Нугуманов (Рига)

Вопрос 17:
   <раздатка>
   Очевидно, ______ и был одним из тех светлых мечтателей, замыслы
которых прочнее иной брони. В 1929 году, в пору, когда стройка только
начиналась, как твердо надо было верить в намеченную и, казалось,
невыполнимую цель, <...>
   &nbsp;
   ПРОСТО ТАК НИКТО НЕ БУДЕТ
   С ТАКИМ УПОРСТВОМ ЛЕЗТЬ В ДУШУ
   СТОП ДОВОЛЬНО ОСТАНОВИСЬ И ПОДУМАЙ
   ХВАТИТ ГНАТЬ ЕРУНДУ МАЯКОВСКИЙ ______
   ИМ ПЛЕВАТЬ ОНИ НОСОМ КВЕРХУ
   ИЗ ДЕРЬМА В НЕБО ЧИСТОЕ ЛЕЗУТ
   </раздатка>
   Заполните любой из пропусков шестибуквенным словом.

Ответ:
Хренов.

Комментарий:
Речь в первом отрывке идет об истории стихотворения "Рассказ Хренова о
Кузнецкстрое и людях Кузнецка".

Источник:
   1. http://kuzbasshistory.narod.ru/book/Pam_Kr/06_1.htm
   2. http://www.anekdot.ru/id/-9977344/

Автор:
Георгий Федосов (Нижний Новгород)

Вопрос 18:
"Вторая"; английский барон, получивший за научные заслуги титул лорда;
распространенная по всему свету маленькая бабочка; свеча... Составив из
трех недостающих элементов списка распространенное выражение, ответьте,
что оно означает в хорошем смысле.

Ответ:
Контент (Креатив) Гениален, Автор Молодец.

Зачет:
Похвала автору (произведения, текста, вопроса) (и пр. по смыслу), а
также любая достаточно распространенная (googlable) расшивровка "КГ/АМ",
не содержащая ругани. Незачет: Аббревиатура кг/ам, а также любая
расшифровка с бранью или матом.

Комментарий:
Перечислены секунда, кельвин, моль, кандела. Кроме них, основными
единицами СИ являются килограмм, ампер и метр.

Источник:
   1. http://ru.wikipedia.org/wiki/Основные_единицы_СИ
   2. http://www.teenslang.su/id/8695

Автор:
Андрей Усачёв (Рига)

Вопрос 19:
Некоторое время назад интернет-портал mail.ru писал, что "Мой круг"
является ТАКОЙ сетью. На этом основании некий шутник приравнял сеть "Мой
круг" к ВОПД РНЕ, обществу "Память" и "Славянскому союзу". Какие два
слова мы заменили на ТАКУЮ?

Ответ:
Национальная социальная.

Комментарий:
Эти три организации являются национал-социалистическими.

Источник:
   1. http://ru.wikipedia.org/w/index.php?title=Национал-социализм&diff=14436820&oldid=14415524
   2. http://www.funlove.ru/mail.html
   3. http://ru.wikipedia.org/wiki/Русское_национальное_единство_(2000)
   4. http://ru.wikipedia.org/wiki/Национально-патриотический_фронт_%C2%ABПамять%C2%BB
   5. http://ru.wikipedia.org/wiki/Славянский_союз

Автор:
Евгений Капитульский (Рига - Санкт-Петербург)

Вопрос 20:
В первом лице единственного числа ОНИ встречаются в артиллерии и
биологии. Во втором лице множественного ОНА встречается в небольших
водоемах и по их берегам. Назовите и ИХ, и ЕЕ.

Ответ:
Ядра, выдра.

Комментарий:
Я-дра, ВЫ-дра.

Автор:
Юрий Петров (Санкт-Петербург)

Вопрос 21:
Сайт компьютерной помощи, специализирующийся в том числе и на продукции
компании "Apple", называется именем собственным, получившим известность
в 1925 году. Напишите это шестибуквенное название абсолютно точно.

Ответ:
iБолит.

Зачет:
iBolit.

Комментарий:
(pic: 20120997.jpg)

Источник:
   1. http://www.ibolit.info/
   2. http://ru.wikipedia.org/wiki/Айболит

Автор:
Евгений Капитульский (Рига - Санкт-Петербург)

Вопрос 22:
Однажды любитель городских транспортных систем Георгий Федосов
договорился провести со своей подругой утро в парке, но, не найдя ее на
месте встречи, позвонил ей на сотовый. Заспанный голос подруги на том
конце провода ответил, что она проспала и всё еще находится в кровати.
На это Георгий пошутил, что время встречи уже давно прошло и его подруга
должна бы быть уже ТАКАЯ, а не СЯКАЯ. "ТАКАЯ" и "СЯКАЯ" - названия двух
"соседок", появившихся в конце 1972 года. Назовите ТАКУЮ и СЯКУЮ.

Ответ:
Беговая, Полежаевская.

Комментарий:
Утром в парке люди, как правило, встречаются, чтобы побегать. Бывает,
конечно, и иначе. Но не в тот раз.

Источник:
   1. ЛОАВ.
   2. http://ru.wikipedia.org/wiki/Беговая_(станция_метро)
   3. http://ru.wikipedia.org/wiki/Полежаевская_(станция_метро)

Автор:
Георгий Федосов (Нижний Новгород)

Вопрос 23:
(pic: 20120998.jpg)
   В эти выходные в Испании проходит двадцать первый ОН. Назовите ЕГО
двумя словами, каждое из которых мы закрыли на раздаточном материале.

Ответ:
Тур Примеры.

Комментарий:
Высшая лига чемпионата Испании по футболу называется Примера. В эти
выходные в Испании как раз проходит ее 21-й тур. На раздаточном
материале мы скрыли слово "примеры" и сокращение "тур.", обозначающее
турецкий язык. Якутский язык - тюркский, как и киргизский, чувашский,
татарский и, естественно, турецкий. Что, кстати, и подтверждает данная
таблица.

Источник:
   1. http://ru.wikipedia.org/wiki/Якутский_язык
   2. http://www.sports.ru/tags/1364772.html?type=calendar

Автор:
Евгений Капитульский (Рига - Санкт-Петербург)

Вопрос 24:
Английский стенд-ап комик Джаспер Керрот рассказывает: "Что такое
триллион? Это очень-очень много. Вот, представьте себе, если бы Иисус
тратил по миллиону в день, начиная прямо со своего рождения, то к
сегодняшнему дню он успел бы потратить только три четверти триллиона. Но
Иисус, конечно же, не тратит, ...". Закончите шутку двумя словами на
английском или на русском.

Ответ:
Jesus saves.

Зачет:
Иисус хранит, Иисус сохраняет.

Источник:
The One Jasper Carrott (BBC, 9 January 2012).

Автор:
Райтис Нугуманов (Рига)

Тур:
11 тур. "Трудновыговариваемые" (Долгопрудный)

Вопрос 1:
На гербе этой автономной области изображены две собаки. А на карте мира,
на которую автор вопроса наткнулся в Интернете, она изображена в виде
шарика с двумя крылышками. Мы не спрашиваем, какого они цвета. Ответьте,
какой стране принадлежит эта автономная область.

Ответ:
Испания.

Комментарий:
Islas Canarias - буквально "Собачьи острова". Крылья были желтого,
точнее, канареечного цвета.

Источник:
ЛОАВ.

Автор:
Максим Дегтярёв

Вопрос 2:
ИХ можно увидеть на фотографии времен Великой Отечественной войны рядом
с героем Советского Союза летчиком Речкаловым. В начале Первой мировой
войны ИХ можно было увидеть где-нибудь на Филиппинах или в зоне
Панамского канала там, где сейчас на две больше. Назовите ИХ абсолютно
точно.

Ответ:
Сорок восемь звезд.

Комментарий:
(pic: 20120999.jpg)
   Речкалов недаром стал героем Советского Союза - на фотографии он
позирует на фоне сорока восьми звезд на его самолете, символизирующих
сорок восемь сбитых самолетов противника. На момент 1914 года Филиппины
и зона Панамского канала принадлежали США, где в 1912 году приняли флаг
с 48 звездами.

Источник:
   1. http://airaces.narod.ru/all1/rechkal.htm
   2. http://ru.wikipedia.org/wiki/История_Филиппин
   3. http://ru.wikipedia.org/wiki/Панамский_канал
   4. http://ru.wikipedia.org/wiki/Флаг_Соединённых_Штатов_Америки

Автор:
Максим Дегтярёв

Вопрос 3:
Человек, уронивший на себя хьортетакк и облившийся ахобланко, может
сказать о себе то же, что и герой известного стихотворения. Назовите его
автора.

Ответ:
Игорь Северянин.

Комментарий:
"Весь я в чем-то норвежском! Весь я в чем-то испанском!". :-) Хьортетакк
и ахобланко - блюда, соответственно, норвежской и испанской кухни.

Источник:
   1. http://ru.wikipedia.org/wiki/Смультринг
   2. http://ru.wikipedia.org/wiki/Ахобланко
   3. http://ru.wikipedia.org/wiki/Ананасы_в_шампанском

Автор:
Максим Дегтярёв

Вопрос 4:
Небольшие естественные спутники газовых планет, которые не позволяют
крайним частицам кольца уходить от него в стороны, называют
спутниками-петухами. Мы не сомневаемся в вашем высоком классе, назовите
две буквы, которые мы заменили в этом вопросе на одну.

Ответ:
ас.

Комментарий:
Не петухами, а пастухами. Они как бы "пасут" частицы, за что и получили
свое название.

Источник:
http://ru.wikipedia.org/wiki/Спутник-пастух

Автор:
Илья Козырев

Вопрос 5:
В мультфильме "Мэтр и призрачный свет", созданном по мотивам мультфильма
"Тачки", действие которого происходит в мире, населенном одними только
автомобилями, главный герой сталкивается с маленькой машинкой,
отличающейся от всех остальных не только размерами. Она, в отличие от
других, по земле не ездит, избрав другой способ перемещения. Назовите
марку этого автомобиля.

Ответ:
Фольксваген.

Комментарий:
Речь идет об автомобиле Фольксваген Жук, который изображен в мультфильме
насекомым, обыгрывая свое название.

Источник:
Мультфильм "Мэтр и призрачный свет".

Автор:
Анна Савинова

Вопрос 6:
После показательного обезвреживания условного противника генерал
поинтересовался формой военнослужащих и устойчивостью ткани на разрыв.
Намек был понят. Назовите действие, последовавшее за этой фразой.

Ответ:
Награждение.

Комментарий:
Генерал имел в виду "дырочки" под новые погоны.

Автор:
Андрей Рыжников

Вопрос 7:
Одна из средневековых легенд рассказывает о том, что разбойник купил
АЛЬФУ, после чего абсолютно спокойно украл не только те деньги, которые
сам за нее заплатил, но и деньги, заплаченные за АЛЬФУ другими людьми.
Что мы заменили на АЛЬФУ?

Ответ:
Индульгенция.

Комментарий:
Если быть точным, он купил индульгенцию на отпущение будущей кражи и
совершил эту кражу через несколько минут, но уже без греха.

Автор:
Андрей Рыжников

Вопрос 8:
Вот цитата из описания КАДМИЯ: "... спиральных углублений, выдавленных в
поликарбонате. Каждое углубление имеет примерно 100 нм в глубину и 500
нм в ширину. Длина углубления варьируется от 850 нм до 3,5 мкм...". Что
мы заменили на КАДМИЙ?

Ответ:
CD.

Зачет:
Компакт-диск.

Комментарий:
Кадмий в таблице Менделеева именуется Cd.

Источник:
   1. http://ru.wikipedia.org/wiki/CD-ROM
   2. http://ru.wikipedia.org/wiki/Кадмий

Автор:
Андрей Рыжников

Вопрос 9:
Напишите название, которое было заимствовано из научно-технического
журнала и должно было означать технологический прогресс, если известно,
что даже при самых благоприятных условиях на покрытие такого расстояния
уйдет не менее трех минут, хотя, в свете последних исследований,
возможно и меньше.

Ответ:
"30 Seconds to Mars".

Зачет:
"30 секунд до Марса".

Комментарий:
Даже при перемещении со скоростью света в момент противостояния на
покрытие расстояния между Землей и Марсом уйдет не меньше трех минут.

Источник:
http://ru.wikipedia.org/wiki/Thirty_Seconds_to_Mars

Автор:
Андрей Рыжников

Вопрос 10:
Многие побывали ТАМ. А согласно одному источнику, ТАМ находятся, в том
числе, Тесей и Филоктет. Ответьте четырьмя словами, где - ТАМ.

Ответ:
Между Сциллой и Харибдой.

Комментарий:
Согласно словарю мифов, статьи о Тесее и Филоктете находятся после
статьи о Сцилле, но до статьи о Харибде.

Автор:
Андрей Рыжников

Вопрос 11:
Согласно одному справочнику, рекомендуется выкладывать не ИКС, а БЕТА,
потому что в слове БЕТА меньше закруглений. Что мы заменили на ИКС и что
- на БЕТА?

Ответ:
SOS, HELP.

Комментарий:
Согласно справочнику по выживанию, слово "HELP" легче выкладывать
прямыми предметами (деревьями, досками и др.).

Автор:
Андрей Рыжников

Вопрос 12:
[Ведущему: прочитать цитату медленно, под запись.]
   В описании ЭТОГО, помимо различных числовых формул, можно встретить
цитату: "Мы порхали и блистали, / Но застряли в перевале: / Не признали
наши крали / Авторалли". Назовите ЭТО.

Ответ:
Число e.

Комментарий:
Длины слов кодируют первые 12 знаков числа e.

Источник:
http://ru.wikipedia.org/wiki/e_(число)

Автор:
Андрей Рыжников

Вопрос 13:
С 1893 года имена обладателей переходящего хоккейного Кубка Стэнли
гравируются на поверхности самого кубка. Можно сказать, что в 1991 году
Кубок Стэнли СДЕЛАЛ ЭТО. Очевидно, канадские сфинксы никогда НЕ ДЕЛАЮТ
ЭТОГО. Что также НЕ ДЕЛАЕТ ЭТОГО в известной песне из советского фильма
1968 года?

Ответ:
Солнечный зайчик.

Комментарий:
К 1991 году на обручах Кубка не осталось места для имен, их сняли и
заменили чистыми - кубок, можно сказать, сбросил кожу, или полинял.
Канадский сфинкс - бесшерстная порода кошек - никогда не линяет. В песне
к фильму "Еще раз про любовь" героиня Татьяны Дорониной поет: "А весной
линяют разные звери, не линяет только солнечный зайчик".

Источник:
   1. http://ru.wikipedia.org/wiki/Кубок_Стэнли
   2. http://ru.wikipedia.org/wiki/Сфинкс_(кошка)
   3. http://www.songkino.ru/songs/eshe_ras.html
   4. http://ru.wikipedia.org/wiki/Ещё_раз_про_любовь

Автор:
Дан Баранов

Вопрос 14:
(pic: 20121000.jpg)
   Примечательно, что впервые ИКС автор вопроса прочитал на электронном
носителе. Назовите ИКС.

Ответ:
"451 градус по Фаренгейту".

Комментарий:
Представленные числа - та же самая температура в других шкалах; также
это температура самовоспламенения бумаги - "подгоревшая" раздатка; в
романе говорится о запрете бумажных книг.

Автор:
Андрей Рыжников

Вопрос 15:
Автор вопроса назвал одиннадцать процентов словосочетанием "РАЗРЫВНАЯ
ПУЛЯ". Различают следующие типы РАЗРЫВНЫХ ПУЛЬ: ПУЛЯ одиночными ударами,
триолями, квартолями и секстолями. Напишите два слова, которые мы
заменили словами "РАЗРЫВНАЯ ПУЛЯ".

Ответ:
Барабанная дробь.

Комментарий:
Число 11 иногда называют "барабанные палочки", стало быть, 11/100 -
барабанная дробь.

Источник:
   1. http://ru.wikipedia.org/wiki/Барабанный_рудимент
   2. Личные наблюдения автора вопроса.

Автор:
Максим Дегтярёв

Вопрос 16:
Дуплет.
   Роб Пайк во время работы над операционной системой "Inferno" читал
"Божественную комедию" Данте, что повлияло на некоторые названия,
придуманные им.
   1. В честь чего был окрещен разработанный для этой операционной
системы язык программирования, под который легко могут подогнуться
специалисты, знающие язык Си?
   2. В честь чего был назван протокол связи, обеспечивающий
взаимодействие с локальными и удаленными ресурсами, одним из назначений
сообщений которого является "перемещение в иерархии файлов"?

Ответ:
   1. Лимб.
   2. Стикс.

Комментарий:
Слово "окрещен" было своеобразной подсказкой. В лимбе, или первом круге
ада, у Данте располагались в том числе некрещеные младенцы. Также
подсказкой было слово "подогнуться". Суть известного танца лимбо
заключается именно в том, чтобы подогнуться и пройти под палкой.

Источник:
   1. http://ru.wikipedia.org/wiki/Inferno_(операционная_система)
   2. http://www.osp.ru/pcworld/1997/10/158096/

Автор:
Илья Козырев

Вопрос 17:
Война - мероприятие очень затратное, это подтвердит любой император.
Однако эти войны даже спасли от банкротства целую кинокомпанию. Мы не
просим вас назвать эти войны. Ответьте абсолютно точно, когда и где они
происходили, по-русски или по-английски.

Ответ:
Давным-давно в далекой-предалекой галактике.

Зачет:
A long time ago in a galaxy far, far away.

Комментарий:
Первый из фильмов "Star Wars. Эпизод IV: Новая надежда" имел огромный
кассовый успех, что фактически спасло компанию "20th Century Fox" от
угрожавшего ей тогда банкротства.

Источник:
http://en.wikipedia.org/wiki/Star_Wars_(film)

Автор:
Татьяна Берсенёва

Вопрос 18:
Согласно информации с форума игры "World of Tanks", советский
трехбашенный танк Т-135 удалось построить лишь в единичном экземпляре,
да и тот пришлось оставить на Сталинградском тракторном заводе. Почему в
сентябре 1942 года, когда бои шли непосредственно на территории завода,
немцам не удалось захватить этот танк?

Ответ:
Этого танка никогда не существовало.

Зачет:
По смыслу (например, Т-135 - мистификация).

Комментарий:
Тема про трехбашенные танки КВ-6 и Т-135 оказалась шуткой.

Источник:
   1. http://forum.worldoftanks.ru/index.php?/topic/149861-%D1%82-135/
   2. http://ru.wikipedia.org/wiki/Волгоградский_тракторный_завод

Автор:
Максим Дегтярёв

Вопрос 19:
[Ведущему: В обоих словах "Один" ударение на второй слог.]
   Прослушайте список:
   Вид дракона в полете - восемьдесят четыре.
   Многословный - шестьдесят четыре.
   Звук грома - пятьдесят два.
   Один - один.
   Мы не спрашиваем, какой количественный показатель обозначают числа.
Ответьте, чем являются все элементы предложенного списка.

Ответ:
[Китайские] иероглифы.

Комментарий:
Даны три самых сложных китайских иероглифа и один из самых простых;
числа - количество черт.

Источник:
http://ru.wikipedia.org/wiki/Китайское_письмо

Автор:
Максим Дегтярёв

Вопрос 20:
По словам знакомого автора вопроса, "Аватар" смог обойти "Титаник" лишь
благодаря ЕЙ. Назовите ЕЕ словом латинского происхождения.

Ответ:
Инфляция.

Источник:
   1. ЛОАВ.
   2. http://ru.wikipedia.org/wiki/Список_самых_кассовых_фильмов
   3. http://ru.wikipedia.org/wiki/Инфляция

Автор:
Никита Евстратов

Вопрос 21:
На Форуме альтернативной истории, или ФАИ, участники создают
альтернативные миры и дают им имена. Каким коротким значимым словом
называется мир, в котором династия сасанидов-огнепоклонников захватила
власть в Японии?

Ответ:
МЯСО.

Зачет:
Мясо.

Комментарий:
Мир Японских Сасанидов-Огнепоклонников. :-) Упоминание аббревиатуры ФАИ
намекает на то, что ответ - тоже аббревиатура.

Источник:
http://fai.org.ru/forum/index.php/topic/10510-мир-японских-сасанидов-огнепоклонников-мясо/

Автор:
Максим Дегтярёв

Вопрос 22:
Внимание, список: Испания - 1 ИКС, Эквадор - 4 ИКСА, Мексика - 1 ИКС.
Согласно одной китайской легенде, ИКС по доброте душевной провез крысу
на своей спине, поэтому оказался лишь вторым. Назовите семибуквенным
словом того, кто убивает ИКСОВ.

Ответ:
Матадор.

Комментарий:
ИКСЫ - это быки. Аллюзия на игру "Быки-коровы". Загадано было слово
"матадор". По китайской легенде крыса переплыла реку, забравшись на
спину быка, а потом соскочила со спины и получила право быть первой в
восточном календаре. В списке приведены некоторые страны, в которых
проводится коррида.

Источник:
http://ru.wikipedia.org/wiki/Китайский_календарь

Автор:
Илья Козырев

Вопрос 23:
Дуплет.
   На первый взгляд может показаться, что для ответа на второй вопрос
вам достаточно будет переставить восемнадцатую. Ну, или шестнадцатую.
   1. Первоначально он имел голос и манеры американского продавца
подержанных машин, но Энтони Дэниелс, который его играл, придал ему
характерное поведение и британский акцент. Проигнорируйте дефис и
назовите его четырьмя символами.
   2. Если ввести эти четыре буквы в поиск русскоязычной Википедии, вас
перенаправит на статью, в конце которой несколько раз упоминаются
разрушительные погодные явления. Предком того, что называют этой
аббревиатурой, считают многозарядную корейскую повозку Хвачха. Напишите
эту аббревиатуру.

Ответ:
   1. C3PO.
   2. РСЗО.

Комментарий:
C-3PO [си-три-пи-о] - андроид из вселенной "Звездных Войн". В статье
"Реактивная система залпового огня" перечисляются русские РСЗО: "Град",
"Ураган", "Смерч", "Торнадо". Если взглянуть на правильные ответы,
сперва действительно может показаться, что они отличаются положением
буквы "Р", восемнадцатой в русском алфавите и шестнадцатой в английском.

Источник:
   1. http://ru.wikipedia.org/wiki/C-3PO
   2. http://ru.wikipedia.org/wiki/Реактивная_система_залпового_огня

Автор:
Максим Дегтярёв

Вопрос 24:
12 августа 2012 года в Омске пройдет традиционный ОН. ОН отличается
одной буквой от известного всем нам мероприятия. Догадавшись, что мы
заменили на НЕГО, назовите город, в котором проходит это мероприятие.

Ответ:
Пущино.

Комментарий:
Омский марафон начнется вместе с олимпийским.

Источник:
http://www.omsk.aif.ru/society/society_transport/564982

Автор:
Никита Евстратов

Тур:
12 тур. "Прометей" (Ивантеевка)

Вопрос 1:
Большой разведывательный корабль "Урал" - самый большой в мире
корабль-разведчик, единственный корабль проекта 1941 "Титан". После
вступления на службу в 1989 году корабль преследовали неудачи, несколько
человек из экипажа погибло. На данный момент он находится на утилизации.
Причин неудач было много, однако среди команды бытовало утверждение, что
виной тому связь с названием разработавшего его КБ. А как называлось это
КБ?

Ответ:
"Айсберг".

Комментарий:
Внутри команды корабля бытовало предположение, что на судьбу "Урала"
повлияла связь между названием проекта ("Титан") и его разработчиком (КБ
"Айсберг").

Источник:
http://ru.wikipedia.org/wiki/ССВ-33_%C2%ABУрал%C2%BB

Автор:
???

Вопрос 2:
В России сейчас нет аристократов, поэтому последнее издание ТАКОГО ИКСА
датируется 1912 годом. Зато сейчас в нашей стране есть двадцать ИКСОВ.
Первоначально ИКС мог обозначать и ствол дерева, и колодку,
прикрепляемую к ногам преступника, и кассовые книги римлян. Впервые
ИКСОМ назвал издание императорских конституций император Грегориан. Что
мы заменили на "ТАКОЙ ИКС"?

Ответ:
Дуэльный кодекс.

Источник:
   1. http://ru.wikisource.org/wiki/Дуэльный_кодекс_В._Дурасова
   2. http://ru.wikipedia.org/wiki/Кодекс

Автор:
Александр Ложечко

Вопрос 3:
На одной шуточной фотографии предлагается ОНА с отверстием 1x1 см,
предлагающая дать шанс... Кому?

Ответ:
Мухе.

Комментарий:
ОНА - мухобойка.

Источник:
http://pikabu.ru/story/dayte_mukhe_shans_467769

Автор:
Константин Сахаров

Вопрос 4:
В статье "Российской газеты", посвященной долговому кризису Еврозоны,
приводятся слова президента Европейского центрального банка Марио Драги,
который оценивает экономическую ситуацию крайне негативно, а также
говорит о недопустимости катастрофы, которая может последовать за крахом
евро. Заголовок статьи состоит из трех слов, сочетающих в себе известный
призыв, ставший названием информационно-успокоительной программы,
выходившей на экраны с 2004 по 2008 год, и устойчивое выражение.
Воспроизведите этот заголовок.

Ответ:
"Тушите Старый Свет".

Источник:
   1. http://www.rg.ru/2011/12/21/evtr.html
   2. http://ru.wikipedia.org/wiki/Тушите_свет

Автор:
???

Вопрос 5:
Терри Пратчетт в книге "Кот без прикрас" советует в крайнем случае,
чтобы наказать кота, обратиться к помощи мафии. Однако сетует, что
мафиози будет невозможно СДЕЛАТЬ ЭТО в корзину, где кот спит. Ответьте
тремя словами, что такое "СДЕЛАТЬ ЭТО".

Ответ:
Подбросить лошадиную голову.

Зачет:
Подкинуть лошадиную голову.

Источник:
Терри Пратчетт. Кот без прикрас.

Автор:
???

Вопрос 6:
Первый опыт олимпиад среди НИХ приходится на 2001 год, хотя логичнее
было бы начать в 1996 или 2008 году. В программу игр входило пять
дисциплин: скалолазание, эквилибристика, прыжки в длину, поднятие
тяжестей и бег. Кто является участником этих олимпиад?

Ответ:
Крысы.

Комментарий:
Слово "опыт" будто бы намекает на крыс.

Источник:
   1. http://animalworld.com.ua/news/Udivitel-nyje-fakty-o-krysah
   2. http://ru.wikipedia.org/wiki/Китайский_календарь

Автор:
???

Вопрос 7:
В одном московском университете есть коридор, где до недавнего времени
висели портреты основателей научных школ. Их объединяла одна
особенность, отражавшая состояние тех, кого они изображали. А как
назвали этот коридор студенты?

Ответ:
Кладбище.

Комментарий:
Люди на портретах были уже мертвы, везде были даты рождения и смерти.

Автор:
Николай Абрамов

Вопрос 8:
(pic: 20121001.jpg)
   Блиц.
   В блоге Константина Белика в разделе "Злобные рисунки" автор вопроса
обнаружила картинки, которые мог бы нарисовать человек, буквально
понимающий всё, что ему говорят.
   1. Первая картинка могла бы послужить иллюстрацией к начальной строке
знаменитого стихотворения Маяковского. Напишите ее.
   2. Во второй картинке зашифрован северный город-герой. Назовите его.
   3. Третья картинка показывает, как можно применить пирроксан. Как
именно?

Ответ:
   1. "Крошка-сын к отцу пришел".
   2. Ленинград.
   3. Внутримышечно.

Источник:
(pic: 20121002.jpg)

Автор:
Юлия Назина

Вопрос 9:
Проводя ночь в замке дяди Владимира, Бендеру внезапно попадается на
глаза число, которое, будучи отраженном в зеркале, вызвало у него
приступ гексафобии. В предыдущем предложении мы пропустили одно слово.
Напишите его.

Ответ:
Зверя.

Зачет:
Дьявола.

Источник:
http://ru.wikipedia.org/wiki/Футурама

Автор:
Владимир Зебрев

Вопрос 10:
Первый ИКС - фильм режиссера Элиссы Даун 2008 года. Второй ИКС
присутствует в повести братьев Стругацких 1972 года. Третий ИКС -
таинственный объект в Тверской области. Четвертый ИКС присутствует в
названии российской телепередачи, выходящей на экраны с 1994 года.
Правильно выбрав два из перечисленных ИКСОВ, назовите государственный
орган, на гербе которого они присутствуют.

Ответ:
Центральная избирательная комиссия Российской Федерации.

Комментарий:
На гербе Центризбиркома в лапах орла находятся белый (серый) и черный
шары. Упомянуты: фильм "Черный шар", золотой шар из повести "Пикник на
обочине", бетонный (серый или белый) шар из Тверской области, имеющий
туманное происхождение, телепередача Виталия Вульфа "Мой серебряный
шар".

Источник:
   1. http://ru.wikipedia.org/wiki/Чёрный_шар
   2. http://ru.wikipedia.org/wiki/Пикник_на_обочине
   3. http://www.the-village.ru/village/food/restaurants/66071-bolshoy-belyiy-shar
   4. http://ru.wikipedia.org/wiki/Мой_серебряный_шар
   5. http://ru.wikipedia.org/wiki/Центральная_избирательная_комиссия_Российской_Федерации

Автор:
Александр Ложечко

Вопрос 11:
После смерти северокорейского лидера Ким Чен Ира одна из газет
опубликовала статью, посвященную изменениям, которые затронут правящую
элиту. Заголовок этой статьи на одну букву короче названия
российско-белорусского фильма 1995 года. Воспроизведите этот заголовок.

Ответ:
"Ын за отца".

Комментарий:
Новым лидером КНДР стал сын покойного Ким Чен Ира Ким Чен Ын.

Источник:
   1. http://www.mk.ru/politics/article/2011/12/19/654247-yin-za-ottsa.html
   2. http://ru.wikipedia.org/wiki/Сын_за_отца

Автор:
???

Вопрос 12:
Если говорить об ИКСАХ, созданных человеком, то самый старый появился в
1320 году в Шотландии. Самым же молодым мог стать ИКС, внезапно вышедший
в 2007 году, если бы не семь точек в названии. В 2006 году на трубе
около котельной старой Таллинской электростанции была повешена табличка,
посвященная одному из ИКСОВ. Назовите ИКС.

Ответ:
Сталкер.

Комментарий:
В Шотландии есть замок Сталкер, табличка под Таллином посвящена
одноименному фильму Андрея Тарковского, а о компьютерной игре и так все
знают.

Источник:
   1. http://ru.wikipedia.org/wiki/Замок_Сталкер
   2. http://ru.wikipedia.org/wiki/S.T.A.L.K.E.R.
   3. http://ru.wikipedia.org/wiki/Сталкер_(фильм)

Автор:
???

Вопрос 13:
Политолог Глеб Павловский называет Россию ПРОПУСКОМ, который время от
времени исчезает. Второе из слов пропуска - "политический". Напишите
остальные два.

Ответ:
Неопознанный объект.

Источник:
http://www.vz.ru/politics/2006/11/22/58213.html

Автор:
Александр Ложечко

Вопрос 14:
Начиная с 2001 года к МКС начали доставляться многоцелевые модули
материально-технического снабжения, разработанные в Италии. Всего было
создано три таких устройства, каждое из которых получило свое имя.
Несмотря на несоответствие в количестве, на логотипе был изображен ИКС.
Назовите ИКС.

Ответ:
Черепашка-ниндзя.

Зачет:
Рафаэль.

Источник:
http://ru.wikipedia.org/wiki/Многоцелевой_модуль_снабжения

Автор:
Николай Абрамов

Вопрос 15:
На улицах Стамбула можно встретить большое разнообразие ИКСОВ. В истории
ИКСОВ упоминается "цветной" американец Гаррет Морган, который впервые
запатентовал ИКС оригинальной конструкции. Что же за ИКСЫ стоят в
изобилии на улицах Стамбула?

Ответ:
Светофоры.

Комментарий:
В блоге Артемия Лебедева "Пришел, увидел" есть фотографии с
запечатленными улицами, где стоит множество светофоров, а также они
изображены в виде дорожного знака на трассе. Гаррет Морган не только
запатентовал светофор, но и впервые в патенте указал как техническую
конструкцию, так и назначение светофоров.

Источник:
   1. http://tema.ru/travel/istanbul.2010.12/
   2. http://ru.wikipedia.org/wiki/Светофор

Автор:
???

Вопрос 16:
(pic: 20121003.jpg)
   На раздаточном материале изображен, пожалуй, самый необычный способ
делать ЭТО. Интернет-пользователь mariana_aga собрала целую коллекцию
возможных вариантов ЭТОГО, куда, в числе прочего, вошли и крест, и
шестиконечная звезда, и солнце, и треугольник, и, разумеется, разное
количество полос в различных комбинациях. Все вы сегодня с этим уже
сталкивались. Изобразите на бланке ответа один из предметов, который мы
от вас скрыли.

Ответ:
Батон хлеба с тремя надрезами.

Зачет:
По смыслу.

Комментарий:
(pic: 20121004.jpg)
   ЭТО - надрезы на хлебе.

Источник:
   1. http://www.rnns.ru/71633-xudozhnik-igor-varchenko.html
   2. http://www.yablor.ru/blogs/nadrezi-na-korke/1182944

Автор:
Алла Болотова, Александр Ложечко

Вопрос 17:
В марте 1962 года министр среднего машиностроения Ефим Славский получил
отчет физиков-ядерщиков Юрия Бабаева и Юрия Трутнева "О необходимости
развертывания работ по изучению возможностей использования атомных и
термоядерных взрывов в технических и научных целях". В частности, там
говорилось об отставании СССР в области исследований промышленного
применения ядерных взрывов от США, где с 1957 года действовала
программа, названная "ИКС". Мы не просим назвать ИКС. Напишите известное
выражение, отсылкой к которому являлось название этой программы.

Ответ:
Перековать мечи на орала.

Комментарий:
Американская программа была названа "Лемех" - отсылка к библейскому
выражению "они перековали мечи на орала". Результатом же советских
экспериментов стало появление озера Чаган.

Источник:
   1. http://ru.wikipedia.org/wiki/Проект_%C2%ABЧаган%C2%BB
   2. http://ru.wikipedia.org/wiki/Операция_%C2%ABПлаушер%C2%BB

Автор:
Николай Абрамов

Вопрос 18:
В интернет-сообществе домохозяек на один вопрос ответили так: "Вы,
конечно, можете купить специальное средство, но воспользуйтесь лучше
народным, например, пепси или спрайтом". Воспроизведите этот вопрос,
если он состоит из слов, начинающихся на одну и ту же букву.

Ответ:
Чем чистить чайник?

Источник:
http://forum.say7.info/topic10355.html

Автор:
Алла Болотова

Вопрос 19:
Званием ИКС было удостоено несколько человек за способность выполнять
свою работу в неблагоприятных для этого условиях. В 1988 году ИКСА
сыграл известный голливудский актер. Назовите этого актера.

Ответ:
Дастин Хоффман.

Комментарий:
ИКС - это человек дождя.

Источник:
   1. http://ru.wikipedia.org/wiki/Сенна,_Айртон
   2. http://ru.wikipedia.org/wiki/Человек_дождя

Автор:
Николай Абрамов

Вопрос 20:
"Папаша, закурить не найдется?" - знакомая многим фраза из
"Бриллиантовой руки", которую произносит здоровенный детина. На роль
этого детины Гайдай утвердил Плешакова сразу после того, как тот показал
ИКС. На жалобу девушки, что у нее ИКС, и вопрос, что делать, один
интернет-пользователь ответил: "Смени пол". Какие два слова мы заменили
на ИКС?

Ответ:
Волосатая грудь.

Источник:
   1. http://shkolazhizni.ru/archive/0/n-50173/
   2. http://www.psylive.ru/forum/default.aspx?act=read&id=3552

Автор:
Юлия Назина

Вопрос 21:
Название станции между Миром машин и Матрицей - "Mobil Ave". Кинопоиск в
связи с этим упоминает два места, объясняя происхождение названия
станции. Напишите оба места.

Ответ:
Ад и рай.

Комментарий:
"Mobil" - анаграмма слова "Limbo", что в переводе означает чистилище,
место между адом и раем.

Источник:
   1. http://www.kinopoisk.ru/level/1/film/316/
   2. http://ru.wikipedia.org/wiki/Лимб_(религия)

Автор:
???

Вопрос 22:
Упитанные стриптизерши иногда хватают лишку. Однажды чешский спортсмен
хватил лишку - сломал шест во время попытки на высоте 5,76. Что мы
заменили на "хватил лишку"?

Ответ:
Перегнул палку.

Источник:
   1. http://www.anekdot.ru/id/544220/
   2. http://desporter.com.ua/posts/239

Автор:
???

Вопрос 23:
В день премьеры мультфильма "Шрек-2" в Италии дистрибьютор полностью
выкупил место для рекламы в газете "La Gazzetta dello Sport", и газета
СДЕЛАЛА ЭТО. ЭТО СДЕЛАЕТ гонщик Джиро д'Италия, если потеряет лидерство
в генеральной классификации, но сохранит его в горной. Что такое
"СДЕЛАТЬ ЭТО"?

Ответ:
Поменять розовый [цвет] на зеленый.

Комментарий:
Розовая майка лидера Джиро д'Италия имеет такой цвет как раз из-за
газеты, основавшей гонку. "Горный король" надевает майку зеленого цвета.

Источник:
   1. http://www.kinopoisk.ru/level/1/film/5273/
   2. http://ru.wikipedia.org/wiki/Джиро_д%E2%80%99Италия

Автор:
???

Вопрос 24:
В вопросе есть замена.
   Этому преподавателю навигации во время своего путешествия пришлось
исполнить роли Нептуна и гавайского аборигена, а при последнем рывке
использовать силу вырывающихся газов. Ответьте одним словом, что мы
заменили на "силу вырывающихся газов".

Ответ:
Шампанское.

Источник:
Мультфильм "Приключения капитана Врунгеля".

Автор:
???

Тур:
13 тур. "Чаатал-Хуюк" (Коломна)

Вопрос 1:
Название сайта, предоставляющего социальную услугу, всего лишь одной
буквой отличается от исповедующего одну из религий. Напишите название
этого сайта.

Ответ:
Будист.

Комментарий:
Надеемся, что этот вопрос, открывающий наш пакет, вас разбудит.

Источник:
http://budist.ru/

Автор:
Екатерина Соколова (Москва)

Вопрос 2:
Перед этим событием ПЕРВЫЙ объявил, что оно является борьбой за "большое
черное дело". ВТОРОЙ лишь возразил, что он "чернее" ПЕРВОГО в два раза.
Назовите ПЕРВОГО и ВТОРОГО в правильном порядке.

Ответ:
Мохаммед Али, Джордж Форман.

Зачет:
Кассиус Клей, Джордж Форман.

Комментарий:
Это событие - их знаменитый боксерский поединок, который часто
рассматривается как один из самых великих боксерских поединков XX века.

Источник:
http://ru.wikipedia.org/wiki/Грохот_в_джунглях

Автор:
Алексей Подлаткин (Воскресенск)

Вопрос 3:
ИГРЕК - отличная вьетнамская фамилия. ИКС - китайская фамилия, а также
название фильма 1998 года. Согласно шутке, российские математики
вычислили ИГРЕК. Они прибавляют его к ИКС и ходят пьяные от счастья. Мы
не просим вас назвать ИКС абсолютно точно. Напишите ИКСИГРЕК.

Ответ:
Пиво.

Комментарий:
Абсолютно точно число "пи" никто не назовет.

Источник:
   1. http://ru.wikipedia.org/wiki/Во
   2. http://ru.wikipedia.org/wiki/Пи
   3. http://www.anekdot.ru/id/-10042369/

Автор:
Екатерина Соколова (Москва)

Вопрос 4:
31 августа 2011 года один знакомый автора вопроса, находясь в центре
Москвы, предположил в удаляющемся человеке довольно известного политика,
однако автор вопроса не согласился. Оказалось, что это действительно был
он. Знакомый прокомментировал данную ситуацию строчкой из песни, в
которой на слух изменилось только ударение в одном слове. Стоит также
отметить, что политик был одет в обычные брюки. Назовите фамилию этого
политика.

Ответ:
МилОв.

Комментарий:
"А я МилОва узнаю, да по походке...".

Источник:
ЛОАВ.

Автор:
Даниил Курганов (Реутов)

Вопрос 5:
Один знакомый автора вопроса предположил, что если бы ОН все-таки
утонул, то к его названию стоило бы добавить "хре", однако сделать это
было бы некому. Назовите ЕГО двумя словами.

Ответ:
Ноев ковчег.

Комментарий:
Если бы он утонул, то стал бы Хреноевым ковчегом.

Источник:
ЛОАВ.

Автор:
Даниил Курганов (Реутов)

Вопрос 6:
19 июня 2011 года ЕЕ платье улетело на аукционе "Profile in History" за
4,6 миллиона долларов, что для данного аукционного дома является нормой.
Назовите ЕЕ.

Ответ:
Мэрилин Монро.

Зачет:
Норма Джин Бейкер.

Источник:
http://ru.wikipedia.org/wiki/Мэрилин_Монро

Автор:
Даниил Курганов (Реутов)

Вопрос 7:
Габриэль БоннО де МаблИ, характеризуя купцов, называл их людьми без
АЛЬФЫ. "АЛЬФА" - российская хоккейная команда, получившая это название в
1957 году. Назовите АЛЬФУ.

Ответ:
Родина.

Комментарий:
Хоккей бывает не только с шайбой. Кировская "Родина" - известная команда
по хоккею с мячом.

Источник:
   1. http://www.istmira.com/istoriografiya-srednix-vekov/1963-istoriografiya-prosveshheniya-vo-francii-monteske.html
   2. http://ru.wikipedia.org/wiki/Родина_(хоккейный_клуб)

Автор:
Алексей Подлаткин (Воскресенск)

Вопрос 8:
Стихи А.С. Пушкина:
   Когда я в комнате моей
   Пишу, читаю без лампады,
   И ясны спящие громады
   Пустынных улиц, и светла
   Адмиралтейская игла <...>.
   В этом отрывке интернет-пользователь, соблюдая рифму, заменил одно
слово, сделав стихотворение очень современным. Напишите замененное слово
русскими или английскими буквами.

Ответ:
Айпад.

Зачет:
iPad.

Комментарий:
   Когда я в комнате моей
   Пишу, читаю без айпада,
   И ясны спящие громады
   Пустынных улиц, и светла
   Адмиралтейская игла <...>.

Источник:
http://leseg.tumblr.com/post/12603560195

Автор:
Екатерина Соколова (Москва)

Вопрос 9:
Внимание, в вопросе есть замена.
   ИКС был застрелен 4 февраля 1936 года в Давосе еврейским студентом
Давидом Франкфуртером. Более известного тезку ИКСА первоначально
планировали назвать "Адольф Гитлер". Гибель ИКСА - центральная часть
сюжета полудокументального романа немецкого писателя Гюнтера Грасса
"Траектория краба". Назовите "убийцу" тезки ИКСА.

Ответ:
[Александр] Маринеско.

Комментарий:
ИКС - это Вильгельм Густлофф, а его тезка - знаменитый германский
пассажирский лайнер, который первоначально должен был называться "Адольф
Гитлер", но после убийства Густлоффа по инициативе самого Гитлера новый
лайнер было решено назвать в честь убитого. Гибель корабля,
торпедированного 30 января 1945 года советской подводной лодкой С-13 под
командованием А.И. Маринеско, считается крупнейшей катастрофой в морской
истории.

Источник:
   1. http://ru.wikipedia.org/wiki/Густлофф,_Вильгельм
   2. http://ru.wikipedia.org/wiki/Вильгельм_Густлофф_(судно)

Автор:
Алексей Подлаткин (Воскресенск)

Вопрос 10:
В песне группы "Ундервуд" микробы в святой воде ДЕЛАЮТ ЭТО. По одной из
версий, одного архитектора прозвали так, потому что он строго ДЕЛАЛ ЭТО.
Назовите самую известную его работу.

Ответ:
Собор Василия Блаженного.

Зачет:
Храм Василия Блаженного, Храм Покрова на рву.

Комментарий:
ДЕЛАТЬ ЭТО - держать пост. Постник и Барма - зодчие, создавшие собор
Василия Блаженного.

Источник:
   1. "Ундервуд", песня "Это судьба".
   2. http://www.moscow-hotels-russia.com/rus/vasil_blagen.htm
   3. http://ru.wikipedia.org/wiki/Постник_Яковлев

Автор:
Екатерина Соколова (Москва)

Вопрос 11:
По словам знакомого автора вопроса, этого известного вам уроженца
немецкого ШвЕрина с небольшой натяжкой можно назвать ИКСОМ. В известном
советском мультфильме один персонаж называет ИКСОМ знаменитого уроженца
поместья Хаггуд. Мы не спрашиваем вас, что мы заменили на ИКС. Назовите
этого персонажа.

Ответ:
[Почтальон] Печкин.

Комментарий:
Уроженец немецкого Шверина - упоминавшийся ранее Вильгельм Густлофф, а
ИКС - это "человек и пароход". В мультфильме "Зима в Простоквашино"
Печкин называет "человеком и пароходом" Крузенштерна, который является
уроженцем поместья Хаггуд.

Источник:
   1. http://ru.wikipedia.org/wiki/Густлофф,_Вильгельм
   2. http://ru.wikipedia.org/wiki/Крузенштерн,_Иван_Фёдорович
   3. Мультфильм "Зима в Простоквашино" (1984).

Автор:
Алексей Подлаткин (Воскресенск)

Вопрос 12:
В статье Википедии, посвященной ИМ, приводится такое предложение: "Mы
Dаем Cоветы Lишь Xорошо Vоспитанным Iндивидуумам". В этой же статье
размещено фото с Красной площади. Ответьте максимально точно, что именно
находится на этом фото.

Ответ:
Кремлевские куранты.

Зачет:
По упоминанию Спасской башни и часов.

Комментарий:
Приведенное предложение - мнемоническое правило для римских цифр.

Источник:
http://ru.wikipedia.org/wiki/Римские_цифры

Автор:
Екатерина Соколова (Москва)

Вопрос 13:
Во Франции можно официально жениться или выйти замуж за НЕГО, хотя закон
в этом случае не дает права наследования за супругом. В 1972 году
появился детектив, в котором всех интересовал некий вопрос, связанный с
ним. Назовите ЕГО.

Ответ:
Покойник.

Комментарий:
Имеется в виду детектив Хмелевской "Что сказал покойник".

Источник:
   1. Французские законы.
   2. И. Хмелевская. Что сказал покойник.

Автор:
Даниил Курганов (Реутов)

Вопрос 14:
Внимание, в вопросе есть замена.
   В 1985 году ИКС был объявлен Книгой рекордов Гиннесса "наиболее
продаваемым альбомом за всю историю". 1 октября 1975 года состоялся
знаменитый ИКС в столичном городе. Происхождение названия этого ИКСА
объясняется тем, что один из его участников упомянул ГОРИЛЛУ. Мы не
спрашиваем, что мы заменили на ИКС. Назовите этот столичный город.

Ответ:
Манила.

Комментарий:
ИКС - это "Триллер", знаменитый альбом Майкла Джексона. 1 октября 1975
года состоялся знаменитый "Триллер в Маниле" - боксерский поединок между
Мохаммедом Али и Джо Фрейзером. Перед поединком Али, в свойственной себе
манере давая различные интервью СМИ, попытался "уколоть" соперника,
срифмовав три слова - "триллер", "Манила" и "горилла".

Источник:
   1. http://ru.wikipedia.org/wiki/Thriller
   2. http://ru.wikipedia.org/wiki/Триллер_в_Маниле

Автор:
Алексей Подлаткин (Воскресенск)

Вопрос 15:
(pic: 20121005.jpg)
   Есть мнение, что ОН сейчас влияет на умы молодежи даже больше, чем
Микки-Маус. Видимо, именно в этом и заключается тонкая шутка автора
оригинала розданного вам изображения. На этом изображении мы не только
закрыли его лицо, но и кое-что слегка изменили. Назовите ЕГО.

Ответ:
Доктор Хаус.

Зачет:
Хью Лори.

Комментарий:
Такой вот каламбур.

Источник:
(pic: 20121006.jpg)

Автор:
Даниил Курганов (Реутов)

Вопрос 16:
Под статьей о новом выявлении ЭТОЙ болезни на сайте newsland.ru в одном
из комментариев читатель скаламбурил, добавив четыре буквы к названию
болезни. Догадавшись, что это за болезнь, назовите область, которая была
местом обнаружения болезни и поводом для каламбура.

Ответ:
Новосибирская [область].

Комментарий:
Новосибирская язва - вот такой черный юмор.

Источник:
http://www.newsland.ru/news/detail/id/811941/

Автор:
Екатерина Соколова (Москва)

Вопрос 17:
Внимание, в вопросе есть замена.
   Один известный политик начал одну из своих предвыборных речей с
фразы: "Дамы и господа, не знаю, понимаете ли вы, что в меня только что
стреляли; но СЛОНА так просто не убьешь". Догадавшись, о ком идет речь,
назовите слово, которое мы заменили словом "СЛОН".

Ответ:
Лось.

Комментарий:
Речь идет о Теодоре Рузвельте, который, как известно, сначала состоял в
республиканской партии, а с ее расколом перешел в созданную
прогрессивную партию. Как известно, символом республиканской партии
является слон, а символом прогрессивной партии после покушения стал
лось.

Источник:
   1. http://ru.wikipedia.org/wiki/Рузвельт,_Теодор
   2. http://ru.wikipedia.org/wiki/Прогрессивная_партия_(США,_1912)
   3. http://ru.wikipedia.org/wiki/Республиканская_партия_(США)

Автор:
Алексей Подлаткин (Воскресенск)

Вопрос 18:
Некоторые из НИХ предпочитают использовать ложки с отпиленными ручками.
О смерти одного из НИХ рассказывается в произведении 1970 года. Назовите
это произведение.

Ответ:
"Отель "У Погибшего Альпиниста"".

Комментарий:
В рюкзаке каждый грамм на счету, да и места такая ложка занимает чуточку
меньше.

Источник:
   1. http://www.skitalets.ru/books/trdruzby/
   2. А. и Б. Стругацкие. Отель "У Погибшего Альпиниста".

Автор:
Екатерина Соколова (Москва)

Вопрос 19:
Внимание, в вопросе есть замена.
   Евгений МилькОвский утверждает, что один русский поэт когда-то сказал
фразу: "ИКСЫ есть, но ИКСА нет". Тема ИКСОВ сейчас очень актуальна, а в
статье Википедии, посвященной ИКСУ, присутствует известная картина 1878
года. Назовите ИКС.

Ответ:
Выбор.

Комментарий:
Имеется в виду картина Виктора Васнецова "Витязь на распутье". В течение
минуты вам тоже пришлось сделать выбор.

Источник:
   1. Женя Мильковский, песня "Снится".
   2. http://ru.wikipedia.org/wiki/Выбор

Автор:
Алексей Подлаткин (Воскресенск)

Вопрос 20:
В песне Евгения МилькОвского о проблемах современного общества говорится
о том, что лирический герой сможет спокойно уснуть только тогда, когда
ИКС отключат за неуплату. ИКС относится к типу G2V [джи ту ви]. Также
под псевдонимом ИКС известна некая Ольга Николаева. Какое слово мы
заменили на ИКС?

Ответ:
Солнце.

Комментарий:
По спектральной классификации Солнце относится к типу G2V ("желтый
карлик"). Ольга Николаева под псевдонимом "Солнце" известна как
участница реалити-шоу "Дом-2".

Источник:
   1. Женя Мильковский, песня "Снится".
   2. http://ru.wikipedia.org/wiki/Солнце
   3. http://www.kino-teatr.ru/kino/acter/w/star/48839/bio

Автор:
Алексей Подлаткин (Воскресенск)

Вопрос 21:
В вопросе есть замены.
   Кур Си разработал учение о всеобщей любви, а религиозная форма этого
учения так и называлась курсизм. Можно сказать, что на протяжении
нескольких столетий он был очень даже в курсе. Назовите художника, в
фамилии которого можно встретить КурСи.

Ответ:
[Амедео] Модильяни.

Комментарий:
Мо Ди (Мо-цзы) - великий китайский философ. Замены: Кур Си = Мо Ди, в
курсе = в моде.

Источник:
http://ru.wikipedia.org/wiki/Мо-цзы

Автор:
Даниил Курганов (Реутов)

Вопрос 22:
Обычно ОН весит от 1 до 2 килограммов. Приобрел в последнее время
популярность, как мечта "победителя". Был ОН и героем финской сказки.
Также холодный ОН упоминается в песне Высоцкого "Инструкция перед
поездкой за рубеж". Назовите ЕГО.

Ответ:
Язь.

Комментарий:
Виктор (от лат. "победитель") Гончаренко и его дикий крик "Язь!!!" почти
всем, конечно, известен. Ну а Николай - герой песни Высоцкого - "съел
холодного язя и инструкцию прослушал, что там - можно, что - нельзя".

Источник:
   1. http://ru.wikipedia.org/wiki/Язь
   2. http://s-skazka.org.ua/index.php?id=finland&sk=44
   3. В.С. Высоцкий, песня "Инструкция перед поездкой за рубеж".

Автор:
Александр Ермаков (Сапожок)

Вопрос 23:
В рассказе о знаменитом потомке захолустных помещиков фигурирует
альпийский ОН, где нашел смерть ИКС. Однако в 1980 году роль альпийского
ЕГО сыграл абхазский. Мы не спрашиваем вас о НЕМ. Назовите ИКСА.

Ответ:
Мориарти.

Зачет:
Профессор Мориарти, Джеймс Мориарти.

Комментарий:
ОН - это водопад. В рассказе "Последнее дело Холмса" фигурирует
Рейхенбахский водопад, который находится в Швейцарии. Враг Холмса -
Мориарти гибнет в водопаде. В "роли" Рейхенбахского водопада в
знаменитом советском фильме "Приключения Шерлока Холмса и доктора
Ватсона" "снялся" Гегский водопад в Абхазской АССР. А в рассказе "Случай
с переводчиком" Холмс рассказывает: "Предки мои были захолустными
помещиками...".

Источник:
   1. http://ru.wikipedia.org/wiki/Рейхенбахский_водопад
   2. http://www.221b.ru/geoPart4.htm
   3. http://ru.wikipedia.org/wiki/Шерлок_Холмс

Автор:
Александр Ермаков (Сапожок)

Вопрос 24:
Если вам повезет найти ЕЕ в фундаменте, то установить возраст дома не
составит труда. В НИИЧАВО ОНА была ТАКОЙ. Что мы заменили словом
"ТАКАЯ"?

Ответ:
Неразменная.

Зачет:
Пятикопеечная.

Комментарий:
ОНА - это монета. Существует обычай закладывать в фундамент монетку при
строительстве. "В институте чародейства и волшебства Привалов обнаружил
чудесные свойства неразменного пятака".

Источник:
   1. http://www.moy-dom.by/?p=331
   2. А. и Б. Стругацкие. Понедельник начинается в субботу.

Автор:
Екатерина Соколова (Москва)

Тур:
14 тур. "Биркиркара" (Москва)

Вопрос 1:
В ответ на предложение профессора Фарнсворта воссоздать условия,
предшествовавшие Большому Взрыву, и высказанное им сожаление о том, что
он уже слишком стар, чтоб дожить до конца этого эксперимента, Туранга
Лила ответила: "Я удивлена, что вы дожили до ИКСА". В ИКСЕ есть
"синхрофазотрон". Какие три слова мы заменили ИКСОМ?

Ответ:
Конец этого предложения.

Комментарий:
В конце предложения "В конце этого предложения есть "синхрофазотрон""
действительно есть "синхрофазотрон". А мы решили начать свой тур с
конца.

Источник:
http://www.theinfosphere.org/Transcript:Reincarnation

Вопрос 2:
В произведении Ивана Козлова Бог, рассказывая Ионе про расширение
Вселенной, упоминает ИКС, ставший препятствием для дальнейшего роста. У
одной из жертв религиозного маньяка в фильме "Семь" ИКС треснул от
расширения. Назовите ИКС.

Ответ:
Желудок.

Комментарий:
В фильме "Семь" отрицательный персонаж иллюстрировал смертные грехи.
Иллюстрируя чревоугодие, он заставлял жертву есть спагетти, пока не
треснули стенки желудка. В стихотворении Ивана Козлова Бог говорит: "У
нас вот вселенная, расширяясь, на днях наткнулась на что-то И... в
общем, наши спецы полагают, что это стенки желудка".

Источник:
   1. http://ittarma.livejournal.com/184751.html
   2. Фильм "Семь" (1995), реж. Дэвид Финчер.

Вопрос 3:
Согласно авторам предисловия к книге Ильи Пригожина, при определенных
условиях энтропия становится ЕЮ. А что является ЕЮ, согласно статье,
написанной в 1848 году уроженцем Безансона?

Ответ:
Анархия.

Комментарий:
Согласно Пригожину, порядок может появляться из хаоса, а Прудон писал о
том, что анархия - мать порядка.

Источник:
   1. http://lib.babr.ru/index.php?book=355
   2. http://rus.anarchopedia.org/Пьер_Жозеф_Прудон
   3. http://en.wikipedia.org/wiki/Pierre-Joseph_Proudhon

Вопрос 4:
Кембриджский профессор математики с гордостью, но не без юмора пишет,
что несколько веков назад это кресло занимал сам Исаак Ньютон, хотя в то
время оно и не имело... Чего?

Ответ:
Электронного управления.

Зачет:
По смыслу.

Комментарий:
Этот профессор - Стивен Хокинг.

Источник:
С. Хокинг. Мир в ореховой скорлупке. - СПб.: Амфора, 2009. - С. 40.

Вопрос 5:
Ларри Мартин однажды сказал: "Леннон был душой "Битлз", Харрисон -
духом, Пол - сердцем, а Ринго - ИКСОМ". В честь героев первой пятилетки
в Евпатории был открыт санаторий под названием "ИКС". Назовите ИКС.

Ответ:
Ударник.

Источник:
   1. http://www.onlineathens.com/stories/120901/new_1209010057.shtml
   2. http://www.udarnik.info/

Вопрос 6:
Сергей Жариков сказал о НЕМ так: "Все смотрят на него - и думают: что же
нам теперь, вместе с этой вот свиньей строить рыночную экономику?!".
Назовите ЕГО.

Ответ:
Константин Боровой.

Комментарий:
"Даже фамилия происходит от слова "боров"".

Источник:
http://www.laertsky.com/sk/sk_003.htm

Вопрос 7:
Автор вопроса был свидетелем того, как его друг пытался научить своего
маленького сына первому слову, повторяя его громко и как бы радостно. В
имеющемся у автора вопроса словаре Ожегова данное слово отсутствует, а
если бы имелось, то находилось бы на 16-й странице. Напишите это слово.

Ответ:
Абырвалг.

Комментарий:
Цитата из "Собачьего сердца": "Отчетливо лает "Абыр", повторяя это слово
громко и как бы радостно".

Источник:
   1. ЛОАВ.
   2. Словарь С.И. Ожегова.

Вопрос 8:
Поморьскою говОрею ОНО называется "багрець". Назовите ЕГО двумя словами,
начинающимися на одну и ту жу букву.

Ответ:
Северное сияние.

Источник:
Поморьская говОря. Краткий поморско-русский словарь. - Архангельск,
2005.

Автор:
Дмитрий Карякин

Вопрос 9:
Длинноволосого Ву-мурта можно узнать по мокрой левой поле кафтана.
Ответьте двумя словами: кто такой этот Ву-мурт?

Ответ:
Удмуртский водяной.

Комментарий:
"Ву" - вода, "мурт" - человек. Ву-мурта прогоняют, стуча палками и
топорами по льду.

Источник:
http://unicorn.e-nk.ru/doku.php/articles:history:mifologija_finno-ugrov

Вопрос 10:
Глагол "петь" в языке ток-писин совпадает по звучанию с названием
учреждения, построенного в 1825 году по проекту Элама Линдса. Назовите
это учреждение.

Ответ:
Синг-Синг.

Источник:
   1. А. Леонтьев. Путешествие по карте языков мира. - М.: Издательский
дом Мещерякова, 2008. - С. 288.
   2. http://ru.wikipedia.org/wiki/Синг-Синг

Вопрос 11:
Сербское проклятие в переводе звучит так: "Дай Бог, чтобы ИКС показали
по CNN" [си-эн-эн]. Консультантом программы "ИКС" является президент
Академии фэншуй Александр Анищенко. Назовите ИКС двумя словами.

Ответ:
Твой дом.

Источник:
   1. http://www.reddit.com/r/AskReddit/comments/efee7/what_are_your_favorite_culturally_untranslateable/
   2. http://www.tdktv.ru/tv/tvoydom/

Вопрос 12:
В энциклопедии "Мiфалогiя беларуса&#1118;" эта статья находится после
статьи "Ястраб". В ней упоминается, в частности, "цыганский дождь", т.е.
дождь, который идет при солнечной погоде. Напишите название этой статьи
на белорусском языке.

Ответ:
Я&#1118;рэй.

Зачет:
Я&#1118;рей, Яурэй, Яурей.

Источник:
Мiфалогiя беларуса&#1118;. - Минск, 2011.

Вопрос 13:
Отвечая на вопрос "Сколько же мы с тобой не виделись?", герой пьесы
"Деревянный гость" предложил отпилить палец. То, что он упоминает далее,
знакомый автора вопроса сравнил с печатями в студенческом билете.
Назовите ЭТО двумя словами.

Ответ:
Годичные кольца.

Зачет:
Годовые кольца.

Комментарий:
Герой пьесы - Буратино.

Источник:
http://prekratite.narod.ru/OSP/RTR/buratin.htm

Вопрос 14:
В ЕГО шее 31 позвонок. Статья в декабрьском журнале "The New Yorker"
называлась "ОН в красных башмачках". Назовите ЕГО.

Ответ:
Черный лебедь.

Комментарий:
Настоящая "лебединая шея". Фильм "Черный лебедь" о сошедшей с ума
балерине журнал сравнивает с другим известным фильмом о балете "Красные
башмачки".

Источник:
   1. http://live.1001chudo.ru/australia_1189.html
   2. http://www.newyorker.com/online/blogs/newsdesk/2010/12/black-swan-in-red-shoes.html

Вопрос 15:
В римейке фильма 70-х годов одну из ролей исполняет девушка в наушниках.
Назовите героиню, которую она играет.

Ответ:
[Принцесса] Лея [Органа].

Источник:
Star Wars Uncut: Director's Cut
(http://www.youtube.com/watch?v=7ezeYJUz-84).

Вопрос 16:
Джулиан Барнс пишет, что скульптор Илья Слоним лепил бюст Шостаковича,
однако результат не удовлетворил председателя Союза композиторов РСФСР.
"Нам нужен оптимистичный Шостакович", - сказал скульптору аппаратчик. В
следующем предложении книги можно найти слово греческого происхождения.
Напишите это слово.

Ответ:
Оксюморон.

Комментарий:
"Шостакович любил повторять этот оксюморон".

Источник:
Дж. Барнс. Нечего бояться. - М.: Эксмо, 2011. - С. 200-201.

Вопрос 17:
На карте национальных стереотипов работы Яна Цветкова Россия названа
"Параноидальной нефтяной империей", Швейцария - попросту "Банком", а
одно небольшое европейское государство - УКАЗАТЕЛЬНЫМ. Догадавшись,
какое слово мы заменили словом "УКАЗАТЕЛЬНЫЙ", назовите это государство.

Ответ:
Македония.

Комментарий:
Безымянным. Обыгрывается известный запрет называть Македонию Македонией.

Источник:
http://www.alphadesigner.com/project-mapping-stereotypes.html

Вопрос 18:
Правый приток реки Кулу в Магаданской области носит название ПЕРВЫЙ. По
мнению Ольги Иконниковой, в 1930-е годы ПЕРВЫЙ вовсе не был привычным
спутником тех, кто дал реке это название, поскольку ее берега изобилуют
лесами. Какое слово мы заменили словом "ПЕРВЫЙ"?

Ответ:
Примус.

Комментарий:
При наличии дров тащить с собой примус и горючее для него геологическим
партиям не было необходимости.
   Расширенный комментарий: "В.В. Леонтьев называет этот предмет
необходимым снаряжением геологов. На наш взгляд, это не совсем отвечает
действительности. Обратим внимание на то, что название появилось в 1935
году на глазомерной карте геолога К.Д. Соколова. Опытные
геологи-полевики утверждают, что в то время примус вряд ли мог входить в
необходимое снаряжение, так как, во-первых, требовал запаса горючего, а
во-вторых, сам был достаточно тяжел. Нужно учесть, что в маршруте
приходилось (да и сейчас приходится) нести рюкзак с образцами пород,
геологический молоток, зачастую ружье, бинокль и многое другое,
действительно необходимое. Кроме того, территория, где протекает этот
ручей, изобилует лесами, а следовательно, и топливом для костра.
Возможно, где-то неподалеку был стан, на котором (как редкость!) имелся
примус, напоминавший о налаженном быте "материка", что и стало причиной
названия ручья".

Источник:
http://academnet.neisri.ru/academnet/smu/philolog/russian/ikonnikova/geologia.html

Вопрос 19:
Бактериальные маты, как правило, состоят из трех слоев:
бактерии-сульфаторедукторы, до которых свет практически не доходит,
пурпурные бактерии, а также фотосинтезирующие цианобактерии. Принято
считать, что в ходе эволюции клетки, имеющие ядро, появились на границе
первого и второго из перечисленных слоев. Рассказывая об этом в книге,
изданной в феврале 2010 года, Александр Марков упоминает государство.
Назовите это государство.

Ответ:
Малави.

Комментарий:
Флаг Малави до 29 июля 2010 года состоял из трех полос: черной, красной
и зеленой и изображения восходящего Солнца на границе первой и второй
полос. Если его перевернуть, то как раз получится схема бактериального
мата.

Источник:
   1. А. Марков. Рождение сложности. Эволюционная биология сегодня.
Неожиданные открытия и новые вопросы. - М.: Астрель, 2010. - Вторая
цветная вкладка.
   2. http://en.wikipedia.org/wiki/Flag_of_Malawi

Вопрос 20:
Прослушайте стихотворение Романа Лейбова о первом и последнем ИКСЕ:
   СТРОЧКА ПРОПУЩЕНА
   был мультимедИен:
   СТРОЧКА ПРОПУЩЕНА
   летел, сияя и урчА.
   Мы не просим вас назвать два пропущенных слова. Назовите ИКСА двумя
словами.

Ответ:
Монгольский космонавт.

Комментарий:
Космонавта из Монголии звали ЖугдэрдэмидИйн ГуррагчА. Стихотворение
звучит так:
   Жугдэрдэмидийн
   был мультимедиен:
   Гуррагча
   летел, сияя и урча.

Источник:
http://lj.rossia.org/users/r_l/3297117.html

Вопрос 21:
Продолжим тему.
   Дуплет.
   1. В "Сказке о царе Салтане" А.С. Пушкина ЕЕ пение упоминается пять
раз. Назовите ЕЕ.
   2. В "Сказке о царе Салтане" А.С. Пушкина ЕЕ пение упоминается один
раз. Назовите ЕЕ.

Ответ:
   1. Белка.
   2. Стрела.

Зачет:
   1. Белка.
   2. Стрелка.

Комментарий:
А больше в этой сказке никто не поет. Это был еще один вопрос про
космонавтов.

Источник:
А.С. Пушкин. Сказка о царе Салтане.

Вопрос 22:
Внимание, цитата! "Маруан Шамах родился и вырос во Франции. Но, проведя
единственный матч за юношескую сборную России, Маруан принял решение
играть за команду своей исторической родины - сборную Марокко". В этой
цитате из еженедельника "Футбол" мы заменили два слова одним. Напишите
замененные нами два слова.

Ответ:
Этой страны.

Комментарий:
Данные слова были заменены словом "России".

Источник:
"Футбол", 2010, N 44. - С. 40.

Вопрос 23:
Михаил Боярский вспоминает, что однажды пошел в баню вместе со
спартаковцами, но общение оказалось для него не слишком приятным. Далее
Боярский говорит, что больше с НИМИ в баню он не ходок. Назовите ИХ.

Ответ:
Баскетболисты.

Комментарий:
Разумеется, знаменитый артист пошел в баню не с футболистами московского
"Спартака", а с баскетболистами "Спартака" питерского. На уровне лица
Боярского располагались совсем иные части тел этих великанов, что
вызывало у него некоторый психологический дискомфорт.

Источник:
http://www.sports.ru/tribuna/blogs/radiozenit/61298.html

Вопрос 24:
Согласно одной кулинарной книге, в ИХ состав входят, помимо прочих
ингредиентов, непременно и свежая, и квашеная капуста, а также не менее
трех сортов мяса. По мнению редакторов пакета, ИХ у вас во время
отыгрыша этого тура быть не должно. Назовите ИХ двумя словами.

Ответ:
Сложные щ[щ]и.

Источник:
В. Юрьев. Кухня западных славян. - Луганск, 2010.

Тур:
15 тур. "Сборная Биологики" (Санкт-Петербург)

Вопрос 1:
[Ведущему: кавычки голосом не выделять!]
   Известно, что монголы брали в армию солдат из покоренных народов, а
также использовали наемников. Таким образом, одновременно на службе
монголов могли находиться ПЕРВЫЕ, ВТОРЫЕ и русские. Однако это не
помогло монголам победить ТРЕТЬИХ. Через несколько столетий русские
использовали "ПЕРВОГО" и "ВТОРОГО" для борьбы с ТРЕТЬИМИ, но, как
выяснилось позже, безуспешно. Мы не просим вас назвать ПЕРВОГО, ВТОРОГО
и ТРЕТЬЕГО. Назовите ПЕРВОГО и ВТОРОГО.

Ответ:
Варяг, кореец.

Источник:
   1. Собственные наблюдения.
   2. http://www.koreanspace.ru/index.php?showtopic=2123
   3. http://minnakiri.sengoku.ru/docs/mongol.htm
   4. http://ru.wikipedia.org/wiki/Армия_Монгольской_империи
   5. http://ru.wikipedia.org/wiki/Монгольские_вторжения_в_Корею
   6. http://sv-rasseniya.narod.ru/xronologiya/3-arxeologicheskie-svidetelstva-drevnej-istorii.html/28.html

Автор:
Александр Маничев (Санкт-Петербург)

Вопрос 2:
Автор вопроса утверждает, что часто бывает в Санкт-Петербурге на
единственной в городе площади трех вокзалов. Самый длинный мост в мире
построен в Новосибирске в 1986 году и имеет длину 2145 м. Какие одни и
те же пять букв мы пропустили в каждом предложении?

Ответ:
Метро.

Комментарий:
Сенная площадь - единственный трехстанционный узел Санкт-Петербургского
метрополитена. Метровокзалом часто называют станцию метро.

Источник:
   1. Собственные наблюдения.
   2. http://metro.yandex.ru/spb
   3. http://dic.academic.ru/dic.nsf/rus_orthography/43012/
   4. http://ru.wikipedia.org/wiki/Метромост
   5. http://gelio.newsib.ru/post/646/

Автор:
Александр Маничев (Санкт-Петербург)

Вопрос 3:
В начале французского мюзикла "Ромео и Джульетта" звучат слова "У каждой
АЛЬФЫ есть своя АЛЬФА". В книге Джеймста Стоддарта "Обманный дом"
описывается странная библиотека, в которой книги, посвященные АЛЬФЕ,
стоят в разделе "Политика", так как по сути описывают лишь несовершенные
представления, окрашенные предрассудками. Назовите АЛЬФУ.

Ответ:
История.

Источник:
   1. Мюзикл "Ромео и Джульетта".
   2. http://parolesfr.kiev.ua/text2.php?id=334&id_singers=16 (фраза в
оригинале - "Toutes les histoires ont leur histoire")
   3. Дж. Стоддарт. Обманный дом. http://lib.rus.ec/b/80509/read

Автор:
???

Вопрос 4:
Дуплет.
   1. Однажды автор вопроса заметил, что благодаря развитию высоких
технологий он стал вести разгульную жизнь. Какую букву мы пропустили в
предыдущем предложении?
   2. Однажды автор вопроса заметил, что люди, которые пишут
компьютерные вирусы, с умом пользуются благами цивилизации. Какая буква
в предыдущем предложении лишняя?

Ответ:
   1. Г.
   2. Л.

Комментарий:
   1. Произошло развитие Интернета, в частности произошло появление
Google.
   2. Ошибки в исходном коде программы могут вызвать желание
воспользоваться данной уязвимостью.

Источник:
Собственные наблюдения.

Автор:
Александр Маничев (Санкт-Петербург)

Вопрос 5:
Один из пользователей сервиса "Ответы" на портале mail.ru ошибочно
назвал ТАКОЙ вокзал Москвы СЯКИМ. С ТАКОГО вокзала можно уехать в бывшие
СЯКИЕ земли. Слова "ТАКОЙ" и "СЯКОЙ" в некоторой степени созвучны. А
причиной ошибки, видимо, послужило произведение конца XIX века. Какие
слова мы заменили на "ТАКОЙ" и "СЯКОЙ"?

Ответ:
Павелецкий и половецкий.

Комментарий:
"Половецкие пляски", видимо, послужили причиной того, что Павелецкий
вокзал был назван Половецким. С Павелецкого вокзала можно уехать на юг
России и Украины - когда-то места обитания половцев.

Источник:
   1. http://otvet.mail.ru/question/1950320
   2. http://ru.wikipedia.org/wiki/Павелецкий_вокзал
   3. http://ru.wikipedia.org/wiki/Половецкая_степь

Автор:
Александр Маничев (Санкт-Петербург)

Вопрос 6:
Автор вопроса назвал абонемент на посещение домашних матчей ФК "Спартак"
(Москва) сложным словом, обозначающим устройство, ставшее популярным в
2000-х годах, заменив в данном слове одну букву. Как назвал автор
вопроса данный абонемент?

Ответ:
Спарт-карта.

Источник:
http://ru.wikipedia.org/wiki/Смарт-карта

Автор:
Александр Маничев (Санкт-Петербург)

Вопрос 7:
Прослушайте цитату из книги Максима Могилевского, посвященную известному
советскому фильму: "Пурш, должно быть, и в самом деле поразился. Вроде
бы человек в снегу, но снегом не пахнет". Актер, сыгравший в этом
фильме, сказал об описанной выше сцене так: "С этой сцены и началась вся
наша отечественная порноиндустрия". Мы не просим назвать вас того, кто
сыграл роль Пурша. Назовите фильм.

Ответ:
"Полосатый рейс".

Комментарий:
Пурш - имя тигра. Упомянута сцена с голым мужчиной (Леонов) в ванной,
покрытым пеной.

Источник:
М. Могилевский. "Полосатый рейс". - М.: Дрофа, 2003.

Автор:
Игорь Титов (Санкт-Петербург)

Вопрос 8:
Индийский мыслитель Шанкара по воле богов должен был прожить только
восемь лет. Но в возрасте восьми лет он уговорил свою мать разрешить ему
СДЕЛАТЬ ЭТО. После чего прожил еще восемь лет. СДЕЛАТЬ ЭТО хотел один из
братьев Карамазовых. Какие два слова мы заменили словами "СДЕЛАТЬ ЭТО"?

Ответ:
Стать монахом.

Комментарий:
Когда Шанкара стал монахом, он начал новую жизнь, и отсчет восьми лет
начался заново (кстати, потом он ухитрился дожить до тридцати двух).
Алексей Карамазов хотел уйти в монастырь.

Источник:
   1. http://ru.wikipedia.org/wiki/Шанкара
   2. Ф.М. Достоевский. Братья Карамазовы.
http://az.lib.ru/d/dostoewskij_f_m/text_0100.shtml

Автор:
Дарья Амирханова (Санкт-Петербург)

Вопрос 9:
Внимание, в вопросе есть замена.
   До недавнего времени этот Наместник мог ПОСТАВИТЬ В УГОЛ целый город
или страну. И неоднократно пользовался этим правом. Вы хорошо знакомы с
произведениями одного человека, который был ПОСТАВЛЕН В УГОЛ в начале XX
века. Какие три слова мы заменили словами "ПОСТАВИТЬ В УГОЛ"?

Ответ:
Отлучить от церкви.

Комментарий:
До II Ватиканского собора Папа Римский (он же Наместник Христа)
действительно мог отлучить от церкви целую страну. А в 1901 году от
церкви был отлучен Лев Николаевич Толстой.

Источник:
   1. http://ru.wikipedia.org/wiki/Интердикт
   2. http://ru.wikipedia.org/wiki/Отлучение_от_церкви
   3. http://ru.wikipedia.org/wiki/Толстой,_Лев_Николаевич

Автор:
Дарья Амирханова (Санкт-Петербург)

Вопрос 10:
Внимание, в вопросе есть замена.
   В книге Михаила Успенского "Время Оно" бисер является способом
попасть в мифическое Время Оно, когда "и вода была мокрее, и валенки
теплее, и старики моложе". Бисер - название представителей одного из
направлений литературы. Какое слово мы заменили словом "бисер"?

Ответ:
Классики.

Комментарий:
Замена произведена с намеком на существование романов "Игра в бисер" Г.
Гессе и "Игра в классики" Х. Кортасара.

Источник:
М. Успенский. Время Оно. http://lib.rus.ec/b/302419/read

Автор:
Дарья Амирханова (Санкт-Петербург)

Вопрос 11:
Когда до премьеры оперы Бизе "Искатели жемчуга", широко объявленной на
весь Париж, оставалось всего две недели, ни авторы либретто, ни сам
композитор всё еще никак не могли решить, чем же закончить оперу.
Экзотическая опера требовала яркой развязки. Положение спас директор
театра, который в отчаянии произнес фразу, подсказавшую либреттистам
идею финала. Назовите эту фразу.

Ответ:
Гори оно всё огнем!

Автор:
Дарья Амирханова (Санкт-Петербург)

Вопрос 12:
В одной статье, посвященной попыткам закрыть телеканал "2x2" в 2008 году
было замечено, что борьба идет не за ПЕРВУЮ, а за ВТОРУЮ. Слова "ПЕРВАЯ"
и "ВТОРАЯ" отличаются одной буквой. Назовите ПЕРВУЮ и ВТОРУЮ.

Ответ:
Чистота, частота.

Источник:
http://www.melkow.ru/piblication/v-borbe-za-chastotu-ili-chistotu-veshchaniya

Автор:
Александр Салита (Санкт-Петербург)

Вопрос 13:
В сообществе ЖЖ ru_psychology обсуждалась боязнь ИХ. Было высказано
предположение, что ОНИ напоминают глаза, например, хищника. В одном из
шуточных комментариев приведены детские слова "ЗАПУГИВАТЬ" и
"ОТПУГИВАТЬ". Что с НИМИ делают при запугивании и отпугивании?

Ответ:
Расстегивают и застегивают.

Комментарий:
Речь идет о боязни пуговиц.

Источник:
http://ru-psychology.livejournal.com/2024199.html

Автор:
Екатерина Губарева (Санкт-Петербург)

Вопрос 14:
В вопросе есть замены.
   В одном из эпизодов произведения Питера Уоттса "Ложная слепота" герои
ведут напряженное обсуждение обстановки. Цитата: "РУКИ Шпинделя
СОЕДИНИЛИСЬ вместе - точно гусеницы поцеловались". Какие слова мы
заменили на "РУКИ" и "СОЕДИНИЛИСЬ"?

Ответ:
Брови, сползлись.

Зачет:
По слову "брови".

Комментарий:
"Брови Шпинделя сползлись вместе - точно гусеницы поцеловались".

Источник:
П. Уоттс. Ложная слепота.

Автор:
Екатерина Губарева (Санкт-Петербург)

Вопрос 15:
В одном из выпусков "Шоу Уральских пельменей" была разыграна сценка из
жизни животных:
   - Пора бы тебе, сынок, найти девушку.
   - Да я уже нашел. Только ее [ПРОПУСК] не выпускают.
   - Родители?
   - Нет, крепежно-присосочный хрящ.
   Догадавшись, кто ее герои, заполните пропуск в диалоге
существительным с предлогом.

Ответ:
"... из домика...".

Источник:
http://www.youtube.com/watch?v=3BirxmDO6YI

Автор:
Екатерина Губарева (Санкт-Петербург)

Вопрос 16:
Внимание, в вопросе есть замена.
   В одном из выпусков "Шоу Уральских пельменей" прозвучала пародийная
песня о жизни ученых. Цитата: "Астрофизик орал из дупла, что он В
КАРАГАНДЕ". Какие три слова мы заменили словами "В КАРАГАНДЕ"?

Ответ:
В черной дыре.

Источник:
http://www.youtube.com/watch?v=hrOqzJPnFZE

Автор:
Екатерина Губарева (Санкт-Петербург)

Вопрос 17:
Внимание, в вопросе есть замены.
   Персонажа мультфильма "Принцесса и лягушка" в русской озвучке зовут
Человек-Недотрога. Янковский в статье 2008 года, посвященной 50-летию со
дня смерти Грина, рассказывает, что тот подарил ему сборник своих
произведений, в числе которых есть "Недотрога", подписанный так: "Грин,
Человек, Недотрога". Какие два слова мы заменили в предыдущем
предложении?

Ответ:
Шварц, тень.

Комментарий:
"Недотрога" - незаконченный роман Грина.

Источник:
http://www.pravmir.ru/shvarc-chelovek-ten-k-50-letiyu-so-dnya-smerti/

Автор:
Елизавета Дмитриева (Санкт-Петербург), Максим Воронецкий (Гродно)

Вопрос 18:
В популярной онлайн-игре "Танки онлайн" АЛЬФА - один из видов оружия,
которое может не только уничтожать врагов, но и лечить союзников. АЛЬФА
почиталась в Лютеции, а в одном из мифов, действие которого происходит
на ее исторической родине, она восстановила жизнь своему мужу. Мы не
просим вас назвать АЛЬФУ. Назовите имя ее мужа.

Ответ:
Осирис.

Комментарий:
АЛЬФА - богиня Изида (Исида). По одному из мифов, своим дуновением она
вернула жизнь Осирису.

Источник:
   1. http://ru.tankiwiki.com/Изида
   2. http://ru.wikipedia.org/wiki/Исида

Автор:
Игорь Титов (Санкт-Петербург)

Вопрос 19:
Один из героев романа Генриха Бёлля говорит, что в 1942-1943 годах в
Германии "иногда люди умирали [пропуск]". Заполните пропуск двумя
словами, начинающимися на одну букву.

Ответ:
"... своей смертью".

Комментарий:
Действительно, редкая смерть для того времени.

Источник:
Г. Бёлль. Групповой портрет с дамой. - М.: Терра - Terra, 1997. - С.
120.

Автор:
Дарья Амирханова (Санкт-Петербург)

Вопрос 20:
Статья русской Википедии "Дело ИКСА" находится в разделе "Антисемитизм".
Автор вопроса авторитетно утверждает, что ИКС не является ингредиентом
алкогольного коктейля "Молоко бешеной коровы". Назовите ИКС.

Ответ:
Бейлис.

Комментарий:
Дело Бейлиса - громкий судебный процесс в 1911 году по обвинению еврея
Менахема Менделя Бейлиса в убийстве 12-летнего мальчика. Бейлис -
сливочный ликер, вкус которого пытаются имитировать коктейлем "Молоко
бешеной коровы" - смесью водки и сгущенного молока.

Источник:
   1. http://ru.wikipedia.org/wiki/Дело_Бейлиса
   2. http://www.rolewiki.org/Молоко_бешеной_коровы

Автор:
Дарья Амирханова (Санкт-Петербург)

Вопрос 21:
   <раздатка>
   ОНА бывает при поражениях коры левого полушария головного мозга, при
этом больной не может сделать то, что вы только что сделали. ОНА
отличается двумя буквами от распространенного мужского имени и совпадает
с очень редким женским именем. Назовите ЕЕ.
   </раздатка>
   ОНА бывает при поражениях коры левого полушария головного мозга, при
этом больной не может сделать то, что вы только что сделали. ОНА
отличается двумя буквами от распространенного мужского имени и совпадает
с очень редким женским именем. Назовите ЕЕ.

Ответ:
Алексия.

Источник:
   1. http://ru.wikipedia.org/wiki/Алексия
   2. http://www.kid.ru/forum/index.php?showtopic=43363

Автор:
Екатерина Губарева (Санкт-Петербург)

Вопрос 22:
(pic: 20121007.jpg)
   Перед вами изображения проекта "Devour". Вы видите не экзотические
планеты, а то, с чем каждый из вас сталкивается на своей кухне. Мы не
просим вас ответить, что это. Назовите автора литературного
произведения, увидевшего свет в 1902 году, название которого точно
отвечает нам, где мы можем увидеть подобные изображения.

Ответ:
Горький.

Комментарий:
Произведение - "На дне", а на фотографиях - днища старых кастрюль.

Источник:
   1. http://www.kulturologia.ru/blogs/230411/14367/
   2. http://ru.wikipedia.org/wiki/На_дне

Автор:
Игорь Титов (Санкт-Петербург)

Вопрос 23:
(pic: 20121008.jpg)
   Платье, которое вы видите на раздатке, называется "robe &agrave; la
polonaise" [роб а ля полонез]. Интересующиеся политикой француженки
стали носить такое платье в 1773 году. Назовите событие, благодаря
которому оно появилось.

Ответ:
Раздел Польши.

Автор:
Елизавета Дмитриева (Санкт-Петербург)

Вопрос 24:
(pic: 20121009.jpg)
   Перед вами логотип московского спортивного клуба. А каким именно
спортом занимаются в этом клубе?

Ответ:
Кёрлинг.

Комментарий:
"Мишени" для кёрлинга придали форму МКАДа, а ее центр обозначает
местонахождение клуба.

Источник:
http://www.artlebedev.ru/everything/curling-club/

Автор:
???

Тур:
16 тур. "Шрёдинбаг" (сборная)

Редактор:
Евгений Кононенко (Москва)

Вопрос 1:
Прослушайте фрагмент стихотворения Иосифа Бродского:
   Достань стаканы
   и выпьем водки за улан, стоящих
   на первом месте в списке мертвецов,
   как в классном списке.
   Какая дата двадцатого века стала заглавием этого стихотворения?

Ответ:
1 сентября 1939 года. Незачет: 1 сентября.

Комментарий:
Начало Второй мировой войны. Уланы - легкая польская кавалерия. Война
началась с нападения Германии на Польшу. 1 сентября - день начала
занятий в школе, поэтому Бродский и использует такое сравнение.

Источник:
http://www.stihi-rus.ru/1/br/1.htm

Автор:
Андрей Лутенко (Кишинев)

Вопрос 2:
По утверждениям солиста, в 1985 году многие родители, желая показать,
что их любовь всё так же сильна, давали своим детям одно и то же имя,
состоящее из трех латинских букв. Напишите это имя.

Ответ:
Sly.

Комментарий:
Клаус Майне, вокалист группы "Scorpions", говорит, что песня "Still
Loving You" спровоцировала бэби-бум во Франции, в результате чего тысячи
людей назвали своих детей аббревиатурой названия этой песни.

Источник:
http://www.the-scorpions.com/phpbb3/viewtopic.php?f=2&t=28184

Автор:
Сергей Якоб (Кишинев)

Вопрос 3:
В статье на сайте радио "Эхо Москвы" говорилось о том, что президент
Абхазии Александр АнквАб пережил пять покушений, но всё же рискнул
выставить свою кандидатуру на выборах и победил. Один из комментаторов
отметил, что из "АнквАба" может получиться... Что?

Ответ:
Ва-банк.

Зачет:
Вабанк.

Комментарий:
Слова "Анкваб" и "ва-банк" - анаграммы, поэтому президенту Абхазии не
привыкать к рискованным шагам.

Источник:
http://www.echo.msk.ru/blog/cknot/807511-echo/

Автор:
Григорий Алхазов (Кишинев)

Вопрос 4:
Капитан космического корабля из романа Станислава Лема "Фиаско" оказался
в трудной ситуации. Не зная, как поступить, он воспользовался ЕЮ, что
оказалось возможным лишь после включения электромагнита. Назовите ЕЕ.

Ответ:
Монета.

Зачет:
Монетка.

Комментарий:
Герой доверил судьбу монетке, но подбросить ее в условиях невесомости
было невозможно.

Источник:
http://lib.rus.ec/b/32412/read#t10

Автор:
Полина Мелека (Кишинев)

Вопрос 5:
В фильме "Время" показано общество, в котором время является платежной
единицей. В бедных временных секторах люди борются за существование, и
лишь единицам удается выбраться в более благополучные сектора.
Бизнесмен, владеющий банком времени, выбрал в качестве кода для своего
сейфа дату 12 февраля 1809 года, потому что в этот день родился... Кто?

Ответ:
Чарльз Дарвин.

Комментарий:
Такой вот искусственный отбор.

Источник:
Фильм "Время".

Автор:
Григорий Алхазов (Кишинев)

Вопрос 6:
Название одного из разделов статьи в журнале "Вокруг света" о
достопримечательностях Филиппин состоит из трех английских слов. На слух
этот заголовок лишь незначительно отличается от названия американского
фильма 1994 года, хотя по смыслу является прямо противоположным.
Воспроизведите название этого раздела.

Ответ:
"Natural Born Healers".

Комментарий:
Фильм 1994 года "Прирожденные убийцы" в оригинале называется "Natural
Born Killers". Хилеры - филиппинские целители.

Источник:
   1. http://www.vokrugsveta.ru/vs/article/7594/
   2. http://en.wikipedia.org/wiki/Natural_Born_Killers

Автор:
Григорий Алхазов (Кишинев)

Вопрос 7:
Сидя за столиками одесского ресторана "Богемский", даже непьющие люди
оказываются ТАМ. Какие два слова мы заменили словом "ТАМ"?

Ответ:
Под Мухой.

Зачет:
Под мухой.

Комментарий:
Над каждым столиком висит репродукция Альфонса Мухи. Богемия - часть
Чехии, а Муха - чех, так что его картины в ресторане с названием
"Богемский" вполне уместны.

Источник:
ЛОАВ.

Автор:
Григорий Алхазов (Кишинев)

Вопрос 8:
Всего таких музеев семь, они располагаются в Санкт-Петербурге, Москве,
Омске, Новокузнецке, Старой Руссе, Семипалатинске, а также в селе
Даровое Московской области. Автор вопроса назвал их обычным для музеев
словом, выделив в нем первые пять букв. Напишите эти пять букв.

Ответ:
ДОСТО.

Комментарий:
Это музеи, посвященные Достоевскому. Автор вопроса назвал их
ДОСТОпримечательностями.

Источник:
   1. http://ru.wikipedia.org/wiki/Музеи_Фёдора_Михайловича_Достоевского
   2. ЛОАВ.

Автор:
Григорий Алхазов (Кишинев)

Вопрос 9:
Название статьи о пользе некоего растения для зрения всего на одну букву
короче названия фильма - лауреата Каннского кинофестиваля 2007 года.
Напишите название статьи.

Ответ:
"Мои черничные очи".

Комментарий:
В 2007 году лауреатом Каннского кинофестиваля стал фильм Вонга Карвая
"Мои черничные ночи". Польза черники для зрения широко известна.

Источник:
   1. http://www.kachestvo.ru/zdorovie/bad/moi-chernichnye-ochi.html
   2. http://ru.wikipedia.org/wiki/Мои_черничные_ночи

Автор:
Григорий Алхазов (Кишинев)

Вопрос 10:
Очевидцы рассказывают о загадочном летающем объекте, который
неоднократно видели в Вязниковском районе Владимирской области. Этот
объект получил название "Огненный ИКС". Мы не спрашиваем вас, какое
слово мы заменили на ИКС. Ответьте, кому посвящен скит, построенный в
этих краях в 1365 году.

Ответ:
Святому Георгию.

Зачет:
Георгию Победоносцу, Георгию.

Комментарий:
ИКС - змей. Считается, что огненный змей появлялся здесь еще в
незапамятные времена, поэтому Егорьевский скит неспроста посвящен
Георгию Победоносцу.

Источник:
http://my.vyazniki.ru/forum6/thread295.html

Автор:
Григорий Алхазов (Кишинев)

Вопрос 11:
В мультфильме "Кот в сапогах" Шалтай-Болтай уговаривает кота
поучаствовать в авантюре, утверждая, что кот получит столько золота, что
ему хватит... Закончите фразу Шалтая-Болтая четырьмя словами.

Ответ:
"... на все девять жизней".

Комментарий:
Золота хватит на всю жизнь, а у кошек жизней, как известно, девять.

Источник:
Мультфильм "Кот в сапогах".

Автор:
Григорий Алхазов (Кишинев)

Вопрос 12:
Дмитрий Долгих отмечает, что футболист Карлос Тевес перебрался в
Манчестер без семьи, а английский язык так и не выучил, поэтому близких
людей у Тевеса в Манчестере нет. Название статьи об этом состоит из трех
слов и лишь одной буквой отличается от названия романа, написанного в
2001 году. Напишите название этой статьи.

Ответ:
"Одиночество в Сити".

Комментарий:
Тевес играет за "Манчестер Сити". Роман Януша Вишневского называется
"Одиночество в Сети".

Источник:
   1. http://www.sports.ru/tribuna/blogs/dolgih/248192.html
   2. http://ru.wikipedia.org/wiki/Одиночество_в_Сети

Автор:
Григорий Алхазов (Кишинев)

Вопрос 13:
Цитата из книги "Третий рейх под знаменем оккультизма": "Оккультизм [ТРИ
СЛОВА ПРОПУЩЕНЫ] и посвящении, поэтому для сохранения секретности
создавались ... ордена и организации". Ньюкасл был [ТРИ СЛОВА ПРОПУЩЕНЫ]
под названием Понс Элиус. Пропуски неотличимы друг от друга на слух.
Заполните любой из них.

Ответ:
"... основан на тайне...".

Зачет:
"... основан на Тайне...".

Комментарий:
Ньюкасл стоит на реке Тайн.

Источник:
   1. С.В. Зубков. Третий рейх под знаменем оккультизма.
http://lib.rus.ec/b/269665/read
   2. http://ru.wikipedia.org/wiki/Ньюкасл-апон-Тайн

Автор:
Тимур Кафиатуллин (Нижний Новгород)

Вопрос 14:
Дуплет.
   1. Автор вопроса, посмотрев в конце ноября фильм "Аноним",
выдвигающий версию авторства пьес Шекспира, назвал его двумя словами.
Напишите эти два слова.
   2. После просмотра фильма "Аноним" автор вопроса решил узнать мнение
своего друга об этом фильме. Воспроизведите вопрос, на который
предлагалось ответить другу.

Ответ:
   1. Зимняя сказка.
   2. "Как вам это понравилось?".

Зачет:
   2. "Как вам это понравится?".

Комментарий:
Оба ответа - названия произведений Вильяма нашего Шекспира.

Источник:
ЛОАВ.

Автор:
Тимур Кафиатуллин (Нижний Новгород)

Вопрос 15:
Казанский зооботсад носит название "Наш ИКС", хотя, например, мангровый
варан там всего один. Назовите ИКС двумя словами.

Ответ:
Ноев ковчег.

Источник:
   1. http://fotki.yandex.ru/users/kotly-pavel/album/55343/
   2. http://www.kazzoobotsad.ru/

Автор:
Тимур Кафиатуллин (Нижний Новгород)

Вопрос 16:
После скандала с регулировкой снаряда остряки Интернета вложили в уста
гимнастки Светланы Хоркиной крылатую фразу, обращенную к организаторам
олимпиады в Сиднее. Процитируйте эту фразу.

Ответ:
"За козла ответите!".

Комментарий:
Как известно, организаторы соревнований неверно отрегулировали высоту
гимнастического "козла", из-за чего пострадали результаты Светланы
Хоркиной и нескольких других спортсменок.

Источник:
http://www.anekdot.ru/id/-10095243/

Автор:
???

Вопрос 17:
[Ведущему: выделить долгую гласную "И" в названии.]
   В мультфильме "Bee Movie" [биии муви] известного человека обвиняют в
незаконном использовании псевдонима. Назовите этот псевдоним.

Ответ:
Стинг.

Комментарий:
Мультфильм "обыгрывает" тему пчел. Sting - жало.

Источник:
http://en.wikipedia.org/wiki/Bee_Movie

Автор:
Тимур Кафиатуллин (Нижний Новгород)

Вопрос 18:
Согласно хронике, после арабского завоевания в 997 году одного из
средиземноморских островов некий купец заключил с новыми хозяевами
острова сделку, после чего ему понадобились 900 верблюдов. Что же он
купил у арабов?

Ответ:
Обломки Колосса Родосского.

Комментарий:
Говорят, бронзовый был...

Источник:
http://ru.wikipedia.org/wiki/Колосс_Родосский

Автор:
???

Вопрос 19:
Тренер Мохаммеда Али Анджело Данди решил проверить, насколько страшным
кажется зрителям будущий соперник его боксера - чемпион мира, бывший
уголовник Санни Листон. Предположения оправдались. Какой вопрос задавал
болельщикам тренер в преддверии боя?

Ответ:
Какого он роста?

Зачет:
По смыслу.

Комментарий:
"У страха глаза велики". Кстати, оцененный болельщиками рост в среднем
составил 205 см, хотя реальный рост был 186 см.

Источник:
А. Беленький. Большие чемпионы. - М.: АСТ, 2004. - С. 163.

Автор:
???

Вопрос 20:
Слово "кодо" в переводе с японского означает "стук сердца". Возможно,
поэтому в игре "Warcraft III" [варкрафт три] ящерицы-"кодо" несут на
себе ЕГО. В НЕМ есть "щи". Назовите ЕГО.

Ответ:
Барабанщик.

Комментарий:
В слове "барабанщик" есть слог "щи". "Кодо" - известный японский
музыкальный коллектив, знаменитое шоу барабанов.

Источник:
   1. http://en.wikipedia.org/wiki/Kodo_(taiko_group)
   2. http://classic.battle.net/war3/orc/units/kodobeast.shtml

Автор:
Тимур Кафиатуллин (Нижний Новгород)

Вопрос 21:
Говорят, что Джон Кеннеди запретил советникам использовать при написании
его инаугурационной речи это слово. Лучшие умы Америки работали над
несколькими вариантами речи, но всё же четыре раза это короткое слово в
речи прозвучало. Напишите это слово.

Ответ:
Я.

Зачет:
I.

Комментарий:
Кеннеди не хотел акцентировать внимание на себе.

Источник:
Д. Рэндалл. Универсальный журналист.

Автор:
???

Вопрос 22:
В журнале "Ринг" помещена редкая фотография, датируемая 1925 годом:
чемпион мира по боксу в легком весе Бенни Леонард находится в кадре
рядом со своей мамой на фоне белой стены. В обеих руках у него
одинаковые предметы. Подпись под фото гласит: "Мама таки добилась
своего. Бенни Леонард демонстративно...". Закончите подпись под фото
четырьмя словами.

Ответ:
"... вешает перчатки на гвоздь".

Комментарий:
Таки какая ж мама боксера не мечтает, чтобы ее сына перестали бить в
лицо?

Источник:
"Ринг", октябрь 2003 г. - С. 63.

Автор:
???

Вопрос 23:
В середине XX века Джеймс Рестон заметил, что в прежние времена людей,
сообщавших плохие новости, частенько отправляли на виселицу, а сейчас
эти люди надеются на ИКС. Между прочим, на ИКС могут надеяться также
некоторые художники и композиторы. Назовите ИКС двумя словами,
начинающимися на одну букву.

Ответ:
Пулитцеровская премия.

Комментарий:
Пулитцеровская премия - награда в области не только журналистики, но и
литературы, музыки и театра, среди ее номинаций есть карикатура и
музыкальные произведения.

Источник:
   1. Дж. Брайант, С. Томпсон. Основы воздействия СМИ. - М.:
Издательский дом "Вильямс", 2004. - С. 257.
   2. http://ru.wikipedia.org/wiki/Пулитцеровская_премия

Автор:
???

Вопрос 24:
ДЕЛАЛИ ЭТО, в частности, баскетболист Майкл Джордан, теннисист Александр
Волков, Фродо Бэггинс, пес Шарик из Простоквашино. Леонид Утёсов в песне
"У Черного моря" признавался, что тоже ДЕЛАЛ ЭТО. Обошлось. А 5 сентября
1919 года судьба не была благосклонна к известному герою, и он СДЕЛАЛ
ЭТО. Назовите этого героя.

Ответ:
[Василий Иванович] Чапаев.

Комментарий:
ДЕЛАТЬ ЭТО - тонуть. Все перечисленные персонажи в свое время были
спасены в воде, кроме Чапаева. Утёсов пел: "Есть море, в котором я плыл
и тонул, и на берег вытащен к счастью".

Источник:
   1. http://ru.wikipedia.org/wiki/У_Чёрного_моря
   2. http://ru.wikipedia.org/wiki/Чапаев,_Василий_Иванович

Автор:
???

Тур:
17 тур. "Тапир во время чумы" (сборная)

Вопрос 1:
В коротком произведении Сергей Федин пишет, что "раки ели...". Где и
кого?

Ответ:
В иле Икара.

Комментарий:
Это палиндром: "А раки ели в иле Икара".

Источник:
http://slovomir.narod.ru/slovarevo/ant/21.html

Автор:
Сергей Спешков

Вопрос 2:
В одном детективе убийцей оказывается довольно крупная женщина. Герой,
который долго за ней охотился, глядя на нее, вспоминает заглавного героя
другого произведения. Назовите этого заглавного героя.

Ответ:
Моби Дик.

Источник:
   1. Ф. Варгас. Мертвые, вставайте!
http://www.flibusta.net/b/95990/read
   2. http://ru.wikipedia.org/wiki/Моби_Дик

Автор:
Иван Семушин

Вопрос 3:
Прозвище "АЛЬФА" носило сразу несколько адмиралов XVII-XVIII веков. А
индейский вождь из племени шайенов получил прозвище АЛЬФА по той
причине, что был очень выносливым. Ответьте двумя словами, что мы
заменили АЛЬФОЙ.

Ответ:
Деревянная нога.

Источник:
   1. http://en.wikipedia.org/wiki/Pegleg
   2. http://en.wikipedia.org/wiki/Wooden_Leg

Автор:
Андрей Сметанин

Вопрос 4:
Слоган из рекламы интернет-магазина игрушек: "Позови свою игрушку!".
Какое слово мы заменили?

Ответ:
Кликни.

Источник:
ЛНА.

Автор:
Нина Семушина

Вопрос 5:
Когда в книге Ромена Гари описывается, как юного фокусника обучают
ловким трюкам, автор упоминает роман, вышедший в тридцатые годы XIX
века. Назовите этот роман.

Ответ:
"Приключения Оливера Твиста".

Комментарий:
Тренировка была по сути сходна с тренировкой ремесла карманника. Оливер
Твист некоторое время жил среди воров-карманников.

Источник:
   1. Ромен Гари. Чародеи. http://www.flibusta.net/b/257326/read
   2. http://ru.wikipedia.org/wiki/Приключения_Оливера_Твиста

Автор:
Иван Семушин

Вопрос 6:
В списке величайших актеров по версии Американского института
киноискусств второе место занимает Кэри Грант. Андрей Шумаков назвал
Гранта тремя словами. Напишите эти слова.

Ответ:
Первый после Богарта.

Комментарий:
А на первом месте Хэмфри Богарт.

Источник:
http://en.wikipedia.org/wiki/AFI%27s_100_Years...100_Stars

Автор:
Сергей Спешков

Вопрос 7:
Согласно законодательству 1737 года, у конных повозок ЭТО было
запрещено. Однако это правило трактовалось весьма вольно и фактически не
соблюдалось. Назовите ЭТО.

Ответ:
Дышло.

Комментарий:
Закон (в данном случае - указ) - что дышло.

Источник:
http://www.kommersant.ru/doc/1528818

Автор:
Алексей Гилёв

Вопрос 8:
В немецком концлагере Штутгоф узникам, пойманным при попытке бегства, на
грудь и спину пришивали ЕЕ. Назовите ЕЕ.

Ответ:
Мишень.

Комментарий:
Таким образом, уменьшались шансы при попытке следующего бегства.

Источник:
http://lib.ru/MEMUARY/KONCLAGER/les.txt_Piece100.05

Автор:
Андрей Сметанин

Вопрос 9:
По-русски ПЕРВЫЙ и ВТОРОЙ называются похожими словами греческого
происхождения. В китайском языке название ПЕРВОГО происходит от названия
висячего продолговатого колокола, а название ВТОРОГО - от слова,
означающего "побег бамбука". Назовите ПЕРВЫЙ и ВТОРОЙ.

Ответ:
Сталактит, сталагмит.

Источник:
http://www.cjvlang.com/Hpotter/wordplay/stalac.html

Автор:
Андрей Сметанин

Вопрос 10:
Опытный ИКС из рассказа Ивана Зорина мог с пяти шагов отличить рубашку
от платья. Какое слово мы заменили на ИКС?

Ответ:
Шулер.

Зачет:
Картежник.

Комментарий:
Отличить рубашку короля от платья дамы.

Источник:
Иван Зорин. Крыло пересмешника.

Автор:
Иван Семушин

Вопрос 11:
Почти четыре десятилетия назад во Флориде был утвержден официальный
минерал штата. Примечательно, что в самой Флориде он не встречается.
Назовите этот минерал двумя словами.

Ответ:
Лунный камень.

Комментарий:
В 1969 году с мыса Канаверал во Флориде стартовала первая экспедиция на
Луну, в честь этого в 1970 году минералом штата был избран лунный
камень, кстати, не встречающийся на Луне.

Источник:
http://dhr.dos.state.fl.us/facts/symbols/symbol.cfm?page=2&id=17

Автор:
Андрей Сметанин

Вопрос 12:
Эту сельскохозяйственную культуру в Центральную Америку в 1516 году
привез Томас де Берланга. Примерно в то же время эти территории стали
сырьевым придатком Испании. Что это за сельскохозяйственная культура?

Ответ:
Бананы.

Комментарий:
Бананы появились в "банановых республиках" только в XVI веке.

Источник:
   1. http://ru.wikipedia.org/wiki/Банан
   2. http://ru.wikipedia.org/wiki/Испанская_империя

Автор:
Андрей Сметанин

Вопрос 13:
Рабле приводит поговорку "кабан, Минерву поучающий". В комментариях к
книге написано, что эта поговорка по смыслу аналогична русской
поговорке. Какая часть тела в ней упоминается?

Ответ:
Рыло.

Комментарий:
Написано, что эта поговорка эквивалентна поговорке "со свиным рылом в
калашный ряд", которая также известна в варианте с "суконным рылом".

Источник:
Ф. Рабле. Гаргантюа и Пантагрюэль (из серии "Библиотека всемирной
литературы").

Автор:
Иван Семушин

Вопрос 14:
В одном интервью Михаила Ефремова спросили, как он любит проводить
свободное время. Михаил привел фразу, которую впервые услышал от Андрея
Кобзона. Воспроизведите эту фразу из пяти слов.

Ответ:
"Дети гениев отдыхают на природе".

Комментарий:
Инверсия популярного выражения "Природа отдыхает на детях гениев". Отцы
Ефремова и Кобзона - не то чтобы гении, но довольно известные творческие
люди.

Источник:
http://www.argumenti.ru/culture/n22/32517

Автор:
Сергей Спешков

Вопрос 15:
Название "ИКС" буквально означает "лишенный природных свойств".
Возможно, именно поэтому Венедикт Ерофеев дал ИКСУ соответствующую
характеристику. Назовите ИКС.

Ответ:
Денатурат.

Комментарий:
В числе жидкостей, мало приспособленных для питья и, тем не менее,
употребляемых, Ерофеев описывает и денатурат. "Но ведь денатурат, будучи
только объектом вдохновения, сам этого вдохновения начисто лишен".

Источник:
   1. http://www.narcom.ru/ideas/common/17.html
   2. Венедикт Ерофеев. Москва - Петушки.
http://www.flibusta.net/b/142473/read

Автор:
Сергей Спешков

Вопрос 16:
Получив титул барона, богатый персонаж Джулиана Барнса нанял
специалиста, чтобы тот вырастил ему ЭТО. Назовите ЭТО двумя словами.

Ответ:
Генеалогическое дерево.

Зачет:
Родословное дерево.

Источник:
Дж. Барнс. Англия, Англия. http://lib.rus.ec/b/139248/read

Автор:
Алексей Гилёв

Вопрос 17:
Поклонники сериала "Шерлок" до сих пор пытаются понять, каким образом
Холмс выжил при падении с крыши. Авторы одного видео попытались
проанализировать работу создателей сериала над соответствующим
фрагментом. Это видео называется "ИКС Шерлока". Какие два слова мы
заменили на ИКС?

Ответ:
Разбор полетов.

Источник:
http://www.youtube.com/watch?v=YEqDydCPe-w

Автор:
Нина Семушина

Вопрос 18:
В образном репортаже спортивного журналиста Акселя Вартаняна ИКС
сохраняет спокойствие и уверен, что Лев Яшин даст ему досмотреть матч до
конца. Ответьте одним словом, что мы заменили ИКСОМ.

Ответ:
Ноль.

Зачет:
Нуль.

Комментарий:
Ноль на табло стадиона верит, что Яшин не пропустит. И Яшин не
пропустил.

Источник:
http://www.sport-express.ru/newspaper/2011-06-02/5_1/

Автор:
Алексей Гилёв

Вопрос 19:
В прямом эфире новозеландского телевидения показали ЕЕ, шесть лет
скрывавшуюся от людей. Что с НЕЙ сделали в прямом эфире?

Ответ:
Остригли.

Зачет:
Побрили, сняли шерсть.

Комментарий:
Когда поймали овцу, на ней было огромное количество шерсти.

Источник:
http://ru.wikipedia.org/wiki/Шрек_(овца)

Автор:
Андрей Сметанин

Вопрос 20:
Герой Мишеля Турнье считает, что главная человеческая добродетель -
продажность и что ИКС в буквальном смысле давно бы наступил, если бы
нами руководили продажные правители. Какие два слова мы заменили на ИКС?

Ответ:
Золотой век.

Источник:
М. Турнье. Пятница, или Тихоокеанский лимб.
http://www.flibusta.net/b/56965/read

Автор:
Иван Семушин

Вопрос 21:
Актер из Папуа-Новой Гвинеи Вильям Такаку - это ШЕСТНАДЦАТИЛЕТНИЙ
КАПИТАН. Какие четыре слова мы заменили на "ШЕСТНАДЦАТИЛЕТНИЙ КАПИТАН"?

Ответ:
Человек, который был Пятницей.

Комментарий:
Такаку снимался в экранизации "Робинзона Крузо".

Источник:
http://en.wikipedia.org/wiki/William_Takaku

Автор:
Сергей Спешков

Вопрос 22:
В одной советской газете 1920-х годов это слово зарифмовали со словом
"оппозиционер". В газете говорилось, что и то, и другое не нужно.
Напишите это слово.

Ответ:
Ер.

Источник:
"Живая театральная газета", ПГУ, 1929 г.

Автор:
Андрей Сметанин

Вопрос 23:
Согласно Гюго, после ареста Эсмеральды место в сердце Феба тут же заняла
Флёр-де-Лис. Какое появившееся в четвертом веке до нашей эры выражение
Гюго при этом упоминает?

Ответ:
Природа не терпит пустоты.

Источник:
   1. В. Гюго. Собор Парижской богоматери.
http://www.flibusta.net/b/151369/read
   2. http://www.bibliotekar.ru/encSlov/15/217.htm

Автор:
Иван Семушин

Вопрос 24:
В вопросе есть замена.
   В 1845 году на похоронах Эндрю Джексона СЕКРЕТАРЬ бывшего президента
был удален с церемонии за богохульство. Что мы заменили словом
"СЕКРЕТАРЬ"?

Ответ:
Попугай.

Комментарий:
Заменили одну птицу другой.

Источник:
http://www.historybyzim.com/2011/08/andrew-jacksons-parrot/

Автор:
Андрей Сметанин

Тур:
18 тур. "Джаггернаут Джигурды" (Тверь)

Вопрос 1:
В средневековом романе рассказывается, как Тристан, излеченный от
ранения, идет в баню, а Изольда обнаруживает среди его вещей меч,
которым был убит ее дядя Морхольт. Разъяренная, она хватает меч и
заносит его над беспомощным рыцарем. Французский писатель Мишель Турнье
находит параллель между эпизодом романа и некой исторической сценой.
Назовите участников этой исторической сцены.

Ответ:
Марат и Шарлотта Корде.

Комментарий:
"Вы спросите, какая связь? Образ такой же, мотив тоже политический. Но
прежде всего, обе сцены ярко иллюстрируют тему беспомощного состояния
человека, лежащего голым в горячей воде, перед стоящей одетой женщиной с
оружием, охваченной яростью мщения".

Источник:
М. Турнье. Тристан и Изольда. // М. Турнье. Полет вампира. - М.:
Стратегия, 2004.

Автор:
Софья Вискова

Вопрос 2:
В вопросе есть замена.
   "ЛАМЕХИ" - название романа Роберта Хайнлайна о семьях людей, которые
в результате генетических экспериментов старели очень медленно. Какие
два слова мы заменили на "ЛАМЕХИ"?

Ответ:
Дети Мафусаила.

Комментарий:
Как известно, Мафусаил прожил почти тысячу лет, и герои Хайнлайна тоже
были долгожителями. Ламех - единственное известное из имен детей
Мафусаила.

Источник:
   1. Р.Э. Хайнлайн. Дети Мафусаила.
   2. Быт. 5:25-27.

Автор:
Станислав Попов

Вопрос 3:
В вопросе есть замена.
   Герой норвежского писателя Эрленда Лу, сочиняя текст для
туристической брошюры, отмечает, что на одно озеро приходится 27,3 ИКСА.
Масса "ИКСА" по олимпийским нормативам не должна быть меньше 105
килограммов. Какое слово мы заменили на ИКС?

Ответ:
Финн.

Источник:
   1. Э. Лу. Лучшая страна в мире. - СПб.: Азбука-классика, 2004.
   2. http://www.chernomor.su/tech/klassy-yacht

Автор:
Софья Вискова, Станислав Попов

Вопрос 4:
(pic: 20121010.jpg)
   На розданной вам фотографии изображен довольно примитивный ИКС из
датского города Скаген. С целью сокрытия информации об аварии на "ИКСЕ"
челябинская газета написала: "Полярные сияния... можно будет наблюдать и
в дальнейшем на широтах Южного Урала". Какое слово мы заменили на ИКС?

Ответ:
Маяк.

Источник:
   1. Ян Пенберти. Маяки: 75 самых красивых маяков мира. - М.:
Арт-родник, 2009.
   2. http://ru.wikipedia.org/wiki/Кыштымская_авария

Автор:
Станислав Попов

Вопрос 5:
На острове Санторин почва содержит большое количество пемзы и
вулканической пыли, что замедляет процесс гниения. В местной поговорке
упоминается Санторин и вампиры. Какой топоним упоминается в русском
аналоге этой поговорки?

Ответ:
Тула.

Комментарий:
Тела умерших на Санторине хорошо сохраняются, поэтому поговорка
"посылать вампира на Санторин" означает то же, что и "возить уголь в
Ньюкасл" или "ездить в Тулу со своим самоваром".

Источник:
Призраки. - М.: Мой мир, 2004.

Автор:
Софья Вискова, Станислав Попов

Вопрос 6:
В кафедральном соборе Дубровника находится скульптурная композиция,
изображающая распятие. Поскольку Дубровник находится на Адриатическом
побережье, неудивительно, что в этой композиции ОНА сделана из
окаменевшей ЕЕ. Назовите ЕЕ.

Ответ:
Губка.

Источник:
Личный опыт автора вопроса.

Автор:
Станислав Попов

Вопрос 7:
Король Великобритании Эдуард VIII при крещении получил семь имен:
Эдуард, Альберт, Кристиан... Назовите любые три из оставшихся четырех
имен.

Ответ:
Георг, Андрей, Патрик, Давид.

Зачет:
Любые три из перечисленных, засчитываются и английские варианты имен.

Комментарий:
По четырем святым покровителям частей Великобритании.

Источник:
http://ru.wikipedia.org/wiki/Эдуард_VIII

Автор:
Софья Вискова

Вопрос 8:
В финальной партии чемпионата мира 1985 года ПЕРВЫЙ и ВТОРОЙ оставались
в игре даже после шести ударов. Назовите российскую организацию, на
гербе которой присутствуют ПЕРВЫЙ и ВТОРОЙ.

Ответ:
Центральная избирательная комиссия России.

Зачет:
Центризбирком.

Комментарий:
ПЕРВЫЙ и ВТОРОЙ - черный и белый шары. Чемпионат мира 1985 года по
снукеру считается самым напряженным матчем, т.к. судьба чемпионства
решалась в последнем черном шаре.

Источник:
   1. http://ru.wikipedia.org/wiki/Центральная_избирательная_комиссия_Российской_Федерации
   2. http://en.wikipedia.org/wiki/1985_World_Snooker_Championship_final

Автор:
Станислав Попов

Вопрос 9:
   <раздатка>
   Be My Love
   </раздатка>
   Французская ювелирная фирма Шоме выпустила серию колец из золота
разных оттенков. Благодаря необычной форме колец из нескольких тонких
можно легко составить широкое. В названии этой серии мы пропустили одну
букву. Восстановите оригинальное название.

Ответ:
"Bee My Love".

Источник:
Журнал "Elle", август 2011 г.

Автор:
Софья Вискова

Вопрос 10:
В одном интернет-комиксе утверждается, что в основу этой игры легли
исторические события Кровавого Воскресенья в России: выстроившаяся рота
царских солдат залпом убивала протестующих, идущих по улице в ходе
демонстрации. На самом деле идея игры родилась благодаря
научно-популярной книге "Полимино". О какой игре идет речь?

Ответ:
Тетрис.

Источник:
   1. http://www.leragecomics.com/index.php/rage-comics/tetris-2/
   2. http://ru.wikipedia.org/wiki/Тетрис

Автор:
Станислав Попов

Вопрос 11:
В кулинарной книге Ники Белоцерковской описываются такие блюда, как
буйабес и рататуй. Название этой книги - "Про еду. Про вино. ...".
Закончите название одним словом.

Ответ:
Прованс.

Источник:
Н. Белоцерковская. Про еду. Про вино. Прованс. - М.: Эксмо, 2011.

Автор:
Софья Вискова

Вопрос 12:
В фильме Вольфганга Петерсена "Подводная лодка" капитан просит
журналиста не фотографировать матросов, пока с ними не ПРОИЗОЙДЕТ ЭТО. В
1860-х годах мисс Энни Джонс стала зарабатывать по 150 долларов в
неделю, потому что с ней ПРОИЗОШЛО ЭТО. Какие два слова мы заменили на
"ПРОИЗОШЛО ЭТО"?

Ответ:
Выросла борода.

Источник:
http://en.wikipedia.org/wiki/Annie_Jones_(bearded_woman)

Автор:
Станислав Попов

Вопрос 13:
Умберто Эко в своей статье говорит, что Адам прожил 930 лет, а Ной -
всего 500. Причиной такой короткой по ветхозаветным меркам жизни Эко
считает ТО, что послужило причиной смерти героя отечественного
мультфильма. Назовите имя и фамилию этого героя.

Ответ:
Билли Бонс.

Комментарий:
Как известно, Ной пил, причем неслабо. :-) А Билли Бонс "умер от
чрезмерного употребления рома". Кстати, Эко ошибается или лукавит: Ной
прожил 950 лет.

Источник:
У. Эко. Картонки Минервы. - СПб.: Symposium, 2008.

Автор:
Станислав Попов

Вопрос 14:
История футболиста Карлоса Тевеса довольно печальна: после трансфера ему
пришлось долго жить в отрыве от семьи, он не смог выучить новый язык, а
после ряда скандалов от него отвернулись болельщики. Название статьи,
посвященной Тевесу на сайте sports.ru, состоит из трех слов и всего лишь
одной буквой отличается от заглавия европейского романа 2001 года.
Восстановите название статьи.

Ответ:
"Одиночество в Сити".

Источник:
   1. http://www.sports.ru/tribuna/blogs/dolgih/248192.html
   2. http://ru.wikipedia.org/wiki/Одиночество_в_Сети

Автор:
Александр Круглов, Станислав Попов

Вопрос 15:
(pic: 20121011.jpg)
   На левом фото, сделанном в 1890-х годах, вы видите АЛЬФУ вместе с
отпрыском. На правом фото - актриса Кэрол Энн Сьюзи, которая исполняет
роль АЛЬФЫ в сериале, но вы наверняка ее не узнали. Какие два слова Мы
заменили на АЛЬФУ?

Ответ:
Мама Говарда.

Комментарий:
Слева - Лавкрафта, справа - Воловитца.

Источник:
   1. http://samlib.ru/g/gawrjuchenkow_j_f/lovecraft.shtml
   2. http://www.kinopoisk.ru/level/4/people/50508/

Автор:
Станислав Попов

Вопрос 16:
Дуплет.
   1. В 1970-х годах во Франции была создана бомба, способная пробивать
бетонные сооружения. Назовите имя, которое получила эта бомба.
   2. Сайт Ufosecret утверждает, что для уничтожения подземных баз
пришельцев в Америке была разработана ракета, способная проникать через
несколько сотен метров скальной породы. Назовите имя, которое получила
эта ракета.

Ответ:
   1. Дюрандаль.
   2. Эскалибур.

Источник:
   1. http://ru.wikipedia.org/wiki/Дюрандаль
   2. http://www.ufosecret.ru/page_1411.html

Автор:
Софья Вискова, Станислав Попов

Вопрос 17:
В конце XIX века в США был спущен на воду крейсер "Везувий", оснащенный
ТАКИМИ пушками для стрельбы динамитными зарядами. Примерно в то же время
преподаватель Принстонского университета Чарльз Хинтон создал СЯКУЮ
пушку для метания бейсбольных мячей, но после нескольких серьезных травм
пушка была разобрана, а изобретатель уволен. Ответьте в правильном
порядке, какие слова мы заменили на "ТАКАЯ" и "СЯКАЯ".

Ответ:
Пневматическая, пороховая.

Зачет:
Пневматическая, огнестрельная.

Источник:
   1. http://en.wikipedia.org/wiki/USS_Vesuvius_(1888)
   2. T.Pratchett, I.Stewart, J.Cohen. Science of Discworld III:
Darwin's Watch. Ebury Press, 2006.

Автор:
Станислав Попов, Сергей Малкин

Вопрос 18:
Один креационист утверждал, что теория эволюции - всего лишь научный
курьез, поскольку Дарвин не СДЕЛАЛ ЭТО. В прошлом году один ученый
СДЕЛАЛ ЭТО через несколько дней после смерти. Какие слова мы заменили на
"СДЕЛАТЬ ЭТО"?

Ответ:
Получить Нобелевскую премию.

Источник:
   1. T.Pratchett, I.Stewart, J.Cohen. Science of Discworld III:
Darwin's Watch. Ebury Press, 2006.
   2. http://www.gazeta.ru/science/2011/10/03_a_3788298.shtml

Автор:
Станислав Попов

Вопрос 19:
В песне Леонида Утёсова "У Черного моря" две ПЕРВЫХ являются ВТОРЫМИ.
ВТОРОЙ для ПЕРВОЙ служит "реклама", а ПЕРВОЙ ВТОРОЙ является "фирма".
Что мы заменили словами "ПЕРВАЯ" и "ВТОРАЯ"?

Ответ:
Анаграмма, рифма.

Комментарий:
В первом куплете песни рифмуются слова "дорог" и "город".

Источник:
http://ru.wikipedia.org/wiki/У_Чёрного_моря_(песня)

Автор:
Станислав Попов

Вопрос 20:
Комикс "Индиана Джонс и ОН" повествует о секретной нацистской
организации, пытающейся с помощью философского камня вернуть к жизни
павших фашистов. ОН всего лишь одной буквой отличается от прозвища
человека, родившегося в Ошмянском уезде Виленской губернии. Какие два
слова мы заменили на "ОН"?

Ответ:
Железный Феникс.

Комментарий:
А в Ошмянском уезде родился Феликс Эдмундович Дзержинский.

Источник:
http://en.wikipedia.org/wiki/Indiana_Jones_and_the_Iron_Phoenix

Автор:
Софья Вискова, Станислав Попов

Вопрос 21:
   <раздатка>
   я знал однажды валентину
   химичку в школе номер семь
   у ней свободная ___________
   читалась в позах и глазах
   </раздатка>
   В розданном вам стихотворении-"пирожке" мы пропустили одно слово.
Восстановите его.

Ответ:
Валентность.

Источник:
http://www.perashki.ru/piro/22996/

Автор:
???

Вопрос 22:
На рекламном постере специализированного журнала изображен чернокожий
мужчина, невозмутимо сидящий за столом среди ку-клукс-клановцев.
Рекламный слоган гласит: "Стань королем ЭТОГО". "ЭТО" - название фильма
1976 года. Какое слово мы заменили на "ЭТО"?

Ответ:
Блеф.

Источник:
http://www.adme.ru/prints/stante-korolem-blefa-s-zhurnalom-live-poker-ddb-france-22790/

Автор:
Станислав Попов

Вопрос 23:
Олдос Хаксли, описывая мрачное полуподвальное помещение, говорит: "свет
<...>, попав в этот подземный каземат, бесследно растворялся - как капля
ПЕРВОГО, угодившая во ВТОРУЮ". Многим из вас известна история, как
ПЕРВОЕ использовалось в необычной ВТОРОЙ. А кем использовалась?

Ответ:
Лениным.

Комментарий:
ПЕРВОЕ - молоко, ВТОРАЯ - чернильница.

Источник:
О. Хаксли. Банкет в честь Тиллотсона.

Автор:
Станислав Попов

Вопрос 24:
Продюсер Ирвинг Тальберг отговаривал своих коллег покупать права на
создание фильма "Унесенные ветром", говоря, что "ни один фильм о НЕЙ
никогда не принес ни цента". Википедия к фильмам о НЕЙ относит "Банды
Нью-Йорка" и "Белое солнце пустыни". Какие два слова мы заменили на
"ОНА"?

Ответ:
Гражданская война.

Источник:
   1. http://en.wikipedia.org/wiki/Irving_Thalberg
   2. У. Эко. Картонки Минервы. - СПб.: Symposium, 2008.

Автор:
Софья Вискова

Тур:
19 тур. "Прочерк" (Москва)

Вопрос 1:
(pic: 20121012.jpg)
   Внимание, выражения, замененные нами на словосочетания "ДЕЛАТЬ ЭТО" и
"ДЕЛАТЬ ТО", отличаются знаками препинания.
   По мнению автора вопроса, заполненная клеточка в учебном расписании
ДЕЛАЕТ ЭТО. В течение ближайших нескольких часов ДЕЛАТЬ ТО будут около
трехсот человек. Ответьте, какое выражение из шести слов мы заменили
словами "ДЕЛАТЬ ЭТО" или "ДЕЛАТЬ ТО".

Ответ:
Отвечать на вопросы Что? Где? Когда?

Зачет:
С любой расстановкой знаков препинания.

Комментарий:
Обычно в клеточке указывают, какой предмет (Что?), в какой аудитории
(Где?) и в какое время (Когда?) будет проходить. Все вы, придя на турнир
по "Что? Где? Когда?", отвечаете на специфичные вопросы этой игры.

Источник:
   1. Расписание занятий в МИЭТ.
   2. ЛОАВ.

Автор:
Григорий Смыслов

Вопрос 2:
(pic: 20121013.jpg)
   Перед вами фрагмент меню, которое автор вопроса увидел в одном кафе.
В каком городе оно располагалось?

Ответ:
Кронштадт.

Комментарий:
Поэтому Гринфилдт и Ронненфилдт.

Источник:
ЛОАВ.

Автор:
Кирилл Борусяк

Вопрос 3:
В одной из серий "Симпсонов" Барт, используя одновременно 15 ИКСОВ,
разбил почти все стекла в домах. Автор вопроса в течение последних пяти
лет пользуется только ИКСОМ. Назовите ИКС словом греческого
происхождения.

Ответ:
Мегафон.

Комментарий:
Поставив последовательно 15 мегафонов, Барт вызвал акустический удар,
выбивший стекла почти во всех домах. Автор вопроса пользуется услугами
сотового оператора "Мегафон".

Источник:
   1. http://ru.wikipedia.org/wiki/The_Secret_War_of_Lisa_Simpson
   2. ЛОАВ.
   3. http://ru.wikipedia.org/wiki/Мегафон

Автор:
Игорь Тюнькин

Вопрос 4:
   <раздатка>
   Toki Epelanto
   </раздатка>
   Токипона - язык, придуманный канадкой Соней Элен Киса. Переведите
раздатку с токипонского на русский.

Ответ:
Эсперанто.

Комментарий:
Название одного искусственного языка на другом. Частица toki означает
"язык". К слову, само название языка переводится как "язык добра".

Источник:
http://ru.wikipedia.org/wiki/Токипона

Автор:
Григорий Смыслов

Вопрос 5:
Отличительной особенностью цикла появления цикад является невозможность
синхронизации с циклом хищников, что позволяет эффективно
восстанавливать популяцию. Причиной этого являются ОНИ. В стихотворении
Арти Кума ученые стремятся построить ИХ в бесконечный ряд. Назовите ИХ
двумя словами.

Ответ:
Простые числа.

Источник:
   1. http://www.tommcmahon.net/2009/11/cicadagenerated-prime-numbers.html
   2. http://www.stihi.ru/2005/06/25-485/

Автор:
Марьяна Зборовская

Вопрос 6:
В одной из песен Вероника Долина жалуется, что ее строка расщепляется.
Какой год упоминается в этой песне?

Ответ:
1986.

Комментарий:
Год чернобыльской аварии. Намек на расщепляющийся атом.

Источник:
В. Долина, "К цветам".

Автор:
Андрей Волыхов

Вопрос 7:
Перевод слова "ИКС" на английский язык совпадает с его ИКСОМ. Что мы
заменили на ИКС?

Ответ:
Транслит.

Комментарий:
Транслит - передача текста, записанного не латиницей, латинскими
буквами, а также цифрами и иными доступными на клавиатуре компьютера
знаками.

Источник:
   1. http://ru.wikipedia.org/wiki/Транслит
   2. http://en.wikipedia.org/wiki/Translit

Автор:
Григорий Смыслов

Вопрос 8:
На портале lib.rus.ec произведение Александра Дюма-отца об одном
средневековом правителе имеет объем лишь 80 килобайт, а о его сыне - в
несколько раз больше. Назовите обоих правителей.

Ответ:
Пипин Короткий и Карл Великий.

Комментарий:
Произведение про Пипина Короткого - короткое, а про Карла Великого -
намного больше.

Источник:
http://lib.rus.ec/a/2538

Автор:
Андрей Волыхов

Вопрос 9:
Если верить одному новостному сайту, команда разработчиков nigma.ru
[нИгма точка ру] после маркетингового исследования молодежной аудитории
решила создать сервис, позволяющий осуществить ЭТО. Назовите ЭТО тремя
словами, начинающимися на одни и те же две буквы.

Ответ:
Поиск по порно.

Комментарий:
Это - команда разработчиков одной из поисковых систем. Новость, кстати,
оказалась первоапрельской шуткой.

Источник:
http://habrahabr.ru/blogs/search_engines/22640/

Автор:
Григорий Смыслов

Вопрос 10:
После увиденного автором вопроса спектакля на сцену под аплодисменты
выходят актеры. Однако через некоторое время на сцене остается только
Леонид Броневой. Назовите его роль в этом спектакле.

Ответ:
Фирс.

Комментарий:
Прямо как в последнем действии "Вишневого сада", когда про Фирса все
забыли. Чехов определяет Фирса как "лакея, старика 87 лет". Родившийся в
1928 году Леонид Броневой как никто подходит к этой роли.

Источник:
   1. ЛОАВ: "Ленком", "Вишневый сад", 06.12.2011 г.
   2. http://www.lenkom.ru/afisha/vs/
   3. http://az.lib.ru/c/chehow_a_p/text_0150.shtml

Автор:
Александр Загороднев

Вопрос 11:
На логотипе микрогорода, который называется "В лесу", используются не
ОНИ-"лапки", а ОНИ-"елочки", стилизованные соответствующим образом.
Назовите ИХ одним словом.

Ответ:
Кавычки.

Комментарий:
Кавычки в русской типографике бывают "лапками" и "елочками". В данном
случае кавычки-"елочки" стилизованы под елки.

Источник:
http://www.microgorodvlesu.ru/

Автор:
Андрей Волыхов

Вопрос 12:
В русском переводе Алиса Льюиса Кэрролла, пытаясь вспомнить одно
произведение, упоминает жука, певицу и императрицу. Назовите это
произведение.

Ответ:
"Дом, который построил Джек".

Комментарий:
Алиса никак не могла вспомнить стихотворение "Дом, который построил
Джек" и спутала Джека с жуком, пшеницу - с певицей, а синицу - с
императрицей.

Источник:
http://merrysongster.narod.ru/thesis/alice.html

Автор:
Григорий Смыслов

Вопрос 13:
По мнению создателей мультсериала "Гриффины", монумент Обамы должен быть
выполнен в виде НЕГО. "ОН" был опубликован в 1956 году. Назовите ЕГО
двумя словами.

Ответ:
Черный обелиск.

Комментарий:
По аналогии с монументом Вашингтона, монумент Обамы должен быть в виде
обелиска. Про цвет - без комментариев. :-)

Источник:
   1. "Family Guy", s08e19, 13-я минута.
   2. http://ru.wikipedia.org/wiki/Чёрный_обелиск_(роман)

Автор:
Григорий Смыслов

Вопрос 14:
Внимание, в вопросе есть замены.
   На Кронштадтском международном экологическом фестивале искусств было
выставлено произведение под названием "Чайковский из чая". Какую фамилию
мы заменили на Чайковский?

Ответ:
Мусоргский.

Комментарий:
Портрет Мусоргского сделан из мусора, собранного на берегу Финского
залива. Экологично.

Источник:
http://www.etovidel.net/sights/city/saint-petersburg/id/_musorgskij_iz_musora_i_drugie_art-obekty_kronfesta

Автор:
Кирилл Борусяк

Вопрос 15:
В одной из передач Би-Би-Си рассказывается, как во время войны во
Вьетнаме солдат бросился на гранату. Ранения были столь серьезными, что
врачи сразу же были отпущены. В одном из слов предыдущего предложения мы
заменили три буквы. Восстановите исходное слово.

Ответ:
Грехи.

Комментарий:
Подумали, что после такого парень уже не жилец, и сразу исповедовали.

Источник:
BBC. Инстинкт человека.

Автор:
Игорь Тюнькин

Вопрос 16:
Экономист Константин Сонин назвал себя синоптиком, предрекая Китаю
падение темпов роста. Какие три буквы мы пропустили в этом вопросе?

Ответ:
ске.

Комментарий:
Синоскептик.

Источник:
http://ksonin.livejournal.com/400927.html

Автор:
Кирилл Борусяк

Вопрос 17:
Героиня одного сериала, вероятно, не мечтает об ЭТОМ, поскольку говорит
герою, что ей достаточно не заплатить по коммунальным счетам, чтобы ей
устроили ЭТО. Назовите ЭТО тремя словами.

Ответ:
Ужин при свечах.

Зачет:
Вечер при свечах.

Источник:
"The Big Bang Theory", s02e14, 11-я минута.

Автор:
Александр Загороднев

Вопрос 18:
На форумах, посвященных известным произведениям, частым объектом
обсуждения является пейринг - то есть составление пар среди персонажей.
Назовите персонажей, сторонники любовной линии между которыми называют
себя Harmony [Хармони].

Ответ:
Гарри [Поттер], Гермиона [Грейнджер].

Комментарий:
Название "Harmony" является комбинацией имен.

Источник:
http://watsonemma.ucoz.ru/blog/harmony_harry_hermione/2011-06-03-182

Автор:
Игорь Тюнькин

Вопрос 19:
В одном из произведений Азимова рассказывается о преступнике с Меркурия,
спрятавшем пленку от микрофильма на подоконнике, в результате чего та
испортилась. Детектив при решении дела сказал, что ошибка преступника
была в том, что ОНА на Земле все-таки умирает. Назовите ЕЕ коротким
словом.

Ответ:
Ночь.

Комментарий:
Преступник, живший на теневой стороне Меркурия, который, как
предполагалось во времена Азимова, почти все время обращен только одной
стороной к Солнцу, привык к вечной темноте за окном. Из-за солнечных
лучей пленка испортилась.

Источник:
А. Азимов. Ночь, которая умирает.

Автор:
Игорь Тюнькин

Вопрос 20:
Лирическая героиня песни Вероники Долиной говорит, что раньше она была
нежной. Кем она, по ее словам, стала, если дальше в песне фигурирует
известная банальная и ироничная рифма?

Ответ:
Снежной бабой.

Комментарий:
   Была я баба нежная,
   А стала баба снежная,
   Вот и вся любовь,
   Вот и нос - морковь...

Источник:
В. Долина, "Снежная баба".

Автор:
Андрей Волыхов

Вопрос 21:
Согласно "Энциклопедии всеобщих заблуждений", это растение обязательно
погибнет без постоянного присмотра. Возможно, именно поэтому это
растение дало название термину из двух слов. Какому?

Ответ:
Банановая республика.

Комментарий:
Как и бананы, "банановые республики" не выживут без покровительства
сильных хозяев.

Источник:
   1. Энциклопедия всеобщих заблуждений.
   2. http://ru.wikipedia.org/wiki/Банановая_республика

Автор:
Игорь Тюнькин

Вопрос 22:
В онлайн-игре "Grepolis" [греполис] наиболее простой способ
противостоять этому юниту - использовать колющее оружие. Назовите этого
юнита.

Ответ:
Циклоп.

Зачет:
Полифем.

Комментарий:
Название проекта может подсказать, что игра основана на античных мифах.
Аллюзия на встречу Одиссея с Полифемом.

Источник:
http://wiki.ru.grepolis.com/wiki/Циклоп

Автор:
Александр Загороднев

Вопрос 23:
В постановке Деза МакАннуфа оперы "Фауст" обретение Маргаритой прощения
от грехопадения показано при помощи НЕЕ. Другая "ОНА" получила
известность в начале 1970-х годов. Назовите ЕЕ.

Ответ:
Лестница в небо.

Зачет:
Stairway to heaven.

Комментарий:
Во второй части вопроса речь идет о песне "Led Zeppelin".

Источник:
   1. Metropolitan Opera, Gounod's Faust, 10.12.2011 г.
   2. http://www.metoperafamily.org/uploadedFiles/MetOpera/watch_and_listen/Live_in_HD/11-12_Season/Faust.Cast.Sheet.DATES.pdf
   3. http://en.wikipedia.org/wiki/Stairway_to_Heaven

Автор:
Александр Загороднев

Вопрос 24:
В одном из произведений Роберта Шекли женщин на большую часть времени
помещают в поле, не выпускающее их без разрешения мужа. Автор сравнивает
это поле с электромагнитной АЛЬФОЙ. Назовите АЛЬФУ словом персидского
происхождения.

Ответ:
Паранджа.

Зачет:
Чадра.

Комментарий:
Такое поле сравнивается с электромагнитной паранджой, из-за которой
посторонние не видят женщину.

Источник:
   1. http://lib.rus.ec/b/111215/read
   2. http://ru.wikipedia.org/wiki/Паранджа

Автор:
Игорь Тюнькин

Тур:
20 тур. "Команда жуликов и воров" (Ярославль)

Вопрос 1:
На эмблеме сообщества, посвященного итальянскому футболу, АЛЬФА
символически изображена в виде мяча. Однако обычно при описании АЛЬФЫ
упоминается не круг, а треугольник. Что мы заменили на АЛЬФУ?

Ответ:
Сицилия.

Комментарий:
На эмблеме Апеннинский полуостров изображен в виде обутой в бутсу ноги,
готовящейся пробить по Сицилии - футбольному мячу; по форме Сицилия
напоминает равнобедренный треугольник.

Источник:
   1. http://www.sports.ru/tribuna/blogs/calcio_renaissance/
   2. http://www.resorts-italy.ru/36.html

Вопрос 2:
Цитата из произведения Салмана Рушди, писателя индийского происхождения:
"Мир полон историй любви, и все любовники в каком-то смысле - ИКСЫ своих
предшественников". 21 августа 2009 года был проведен всемирный день
"ИКСА". Какое слово в этом вопросе олицетворено словом "ИКС"?

Ответ:
Аватар.

Комментарий:
21 августа 2009 года наступил так называемый "Всемирный день Аватара". В
сети появился двухминутный тизер фильма, а в некоторых кинотеатрах всем
желающим показали несколько отрывков общей продолжительностью около 15
минут.

Источник:
   1. Салман Рушди. Дети полуночи.
   2. http://www.annews.ru/news/detail.php?ID=191844

Вопрос 3:
В 1975 году была создана межправительственная группа в составе министров
внутренних дел государств-членов ЕЭС. Группа занималась вопросами
терроризма, радикализма, экстремизма и насилия. Назовите город, в
котором эта группа была создана.

Ответ:
Рим.

Комментарий:
Группа называлась TREVI (terrorism, radicalism, extremism, violence);
фонтан Треви - одна из главных достопримечательностей Рима.

Источник:
   1. http://www.krugosvet.ru/enc/istoriya/EVROPESKI_SOYUZ_ES.html?page=0,4
   2. http://en.wikipedia.org/wiki/TREVI
   3. http://en.wikipedia.org/wiki/Trevi_Fountain

Вопрос 4:
В Википедии в начале статьи об истории фашизма упоминается басня,
приписываемая Эзопу. Назовите людей, упоминаемых в ее названии.

Ответ:
Отец и сыновья.

Комментарий:
В начале статьи обычно рассказывается об этимологии понятия; фасция
подобна венику, который не смогли сломать сыновья.

Источник:
   1. http://ru.wikipedia.org/wiki/История_фашизма
   2. http://ru.wikisource.org/wiki/Отец_и_сыновья_(Эзоп/Лев_Толстой)

Вопрос 5:
В стихотворении "Надпись на ИКСЕ" утверждается, что "червь противнее губ
человека". Один из самых известных ИКСОВ был изготовлен в десятом веке.
Какие три слова мы заменили на ИКС?

Ответ:
Кубок из черепа.

Комментарий:
Согласно "Повести временных лет", из черепа убитого князя Святослава
печенеги сделали кубок...

Источник:
   1. http://az.lib.ru/b/bajron_d_g/text_0200.shtml
   2. http://ru.wikipedia.org/wiki/Куря_(печенежский_хан)

Вопрос 6:
   <раздатка>
   Поднимаюсь предложить, сэр:
   Видят ли гуси Бога?
   "Девять человек объясняют?" - "Девять человек" - киваю я.
   Машину или кота я увидел?
   Гони быстро, безопасная машина!
   </раздатка>
   Американский пародист Эл Янковик пишет собственные тексты на
известные песни. Перед вами дословные переводы нескольких строчек из его
пародии на песню Боба Дилана. Ответьте абсолютно точно, каким коротким
словом Эл назвал эту пародию, если это название также могло стать ее
строчкой.

Ответ:
Bob.

Зачет:
Боб.

Комментарий:
Текст Эла Янковика для этой песни состоит полностью из палиндромов:
   "Rise to vote, sir" [райз ту воут сёр],
   "Do geese see God?" [ду гиз си год],
   "Do nine men interpret? - Nine men, I nod" [ду найн мен интёрпрет? -
найн мен, ай нод],
   "Was it a car or a cat I saw?" [уоз ит э кар ор э кэт ай со],
   "Race fast, safe car" [рейс фаст сейф кар].
   В качестве названия взято имя пародируемого объекта - Боб, которое
также является палиндромом.

Источник:
   1. Weird Al Yankovic, альбом "Poodle Hat" (2003), песня "Bob".
   2. http://www.azlyrics.com/lyrics/weirdalyankovic/bob.html

Вопрос 7:
Дуплет.
   1. Афоризм, приписываемый Николаю Фоменко, гласит, что у вас всегда
есть два выхода, даже если с вами СДЕЛАЛИ ЭТО. Герой стихотворения в
переводе Самуила Маршака, с которым СДЕЛАЛИ ЭТО, спрыгнул перед этим с
дерева. С какого именно дерева?
   2. Афоризм Алексея Калинина не рекомендует смотреть много рекламы
"Сникерса", иначе глаза СДЕЛАЮТ ЭТО. Герой шуточной сказки Ирины и
Леонида Тюхтяевых говорит, что СДЕЛАЛ ЭТО с дерева. С какого именно
дерева?

Ответ:
   1. С ели.
   2. С липы.

Зачет:
   1. Ель.
   2. Липа.

Комментарий:
   1. СДЕЛАТЬ ЭТО - съесть. Героя Маршака соответственно СЪЕЛИ, а
строчкой выше он спрыгнул С ЕЛИ.
   2. СДЕЛАТЬ ЭТО - слипнуться. Герой детской сказки "Зоки и Бада"
посчитал, что "слипнуться" означает "упасть с липы": "Ух, я с той липы
слипнулся, так слипнулся, с самого верха".

Источник:
   1. Афоризм из коллекции Николая Фоменко (справедливости ради -
авторство не у всех известно).
   2. С. Маршак. Произведения для детей.
   3. Афоризм Алексея Калинина.
   4. И. и Л. Тюхтяевы. Зоки и Бада.

Вопрос 8:
Один из комментаторов Збигнева Бжезинского, упоминая перед этим одну из
его известных книг, уподобляет зоны гегемонии США в Евразии ТАКИМ ИКСАМ,
а территории, где американское господство еще только предстоит
установить, - СЯКИМ ИКСАМ. Назовите ТАКИЕ ИКСЫ и СЯКИЕ ИКСЫ в правильном
порядке.

Ответ:
Белые клетки, черные клетки.

Комментарий:
Упоминается знаменитая книга Бжезинского "Великая шахматная доска".

Источник:
http://russway.narod.ru/articles/bjezin.htm

Вопрос 9:
(pic: 20121014.jpg)
   ОН упоминается журналом "Вокруг света" в описании улитки, изображение
которой вы видите. Википедия пишет, что на гербе Кубы под НИМ находится
щит. В честь какой исторической области ОН получил свое название?

Ответ:
Фригия.

Комментарий:
ОН - фригийский колпак, символ революции и революционеров. На гербе Кубы
фригийский колпак заменяет корону. Журналисту "Вокруг света" раковина
улитки Каринарии напомнила фригийский колпак.

Источник:
   1. http://www.vokrugsveta.ru/vs/article/7244/
   2. http://ru.wikipedia.org/wiki/Герб_Кубы
   3. http://ru.wikipedia.org/wiki/Фригийский_колпак

Вопрос 10:
Отрывок из эссе Валерии Новодворской "Рыбный день для державы": "Но вот
идеалист Некрасов опять зовет оппонента, уже в "Отечественные записки".
Какие два слова в этой цитате пропущены?

Ответ:
Карась, ерша.

Комментарий:
В оригинале цитата такая: "Но вот карась-идеалист Некрасов опять зовет
оппонента-ерша, уже в "Отечественные записки"".

Источник:
http://lib.rus.ec/b/143856/read#t8

Вопрос 11:
Телекомментатор Василий Уткин однажды сравнил выпрыгивающих вместе на
мяч Серхио Рамоса, Пепе и Карлеса Пуйоля с ИКСОМ. В некоторых случаях к
ИКСУ ставится пандус. Назовите слово, которое мы заменили на ИКС.

Ответ:
Пьедестал.

Комментарий:
Три человека в полете напоминали ступени пьедестала (центральный
выпрыгнул выше всех); пандус может ставиться для спортсменов с
ограниченными возможностями.

Источник:
Комментарий к матчу 1/4 финала Кубка Испании "Реал Мадрид" -
"Барселона", 19.01.2011 г.

Вопрос 12:
(pic: 20121015.jpg)
   Перед вами работа цифрового художника Антонио де Оливейра "Образ в
небе". Кто, по замыслу автора, изображен на ней?

Ответ:
Джон Леннон.

Комментарий:
Название работы в оригинале - "Imagination in the sky" - является
отсылкой к песне 'Imagine'.

Источник:
http://www.toonpool.com/cartoons/imagination%20in%20the%20sky_125828

Вопрос 13:
Дуплет.
   1. Владелец и основатель одного ресторана, офтальмолог Игорь Медведев
считает, что человек, принимающий пищу в НЕЙ, гораздо лучше и тоньше
различает особенности блюд. Если верить одному высказыванию, можно
предположить, что в этот ресторан в основном ходят молодые люди.
Назовите ЕЕ.
   2. В одном ресторане названия блюд представляют собой известные
картины. Например, "Махой обнаженной" назвали малосольную семгу,
"Жирафами в огне" - мексиканские лепешки. Название какой картины
получили в этом меню крупно нарезанные помидоры, сладкий перец и редис
под соусом на выбор: сметана, майонез домашний, масло оливковое,
бальзамическая заправка?

Ответ:
   1. Темнота.
   2. "Купание красного коня".

Комментарий:
   1. Отличительной особенностью ресторана является возможность
принимать пищу в кромешной темноте. Ну а темнота - друг молодежи.
   2. Все эти овощи красные, и они "купаются" в соусе.

Источник:
   1. http://www.uvoice.ru/themes/2/vtemnote.html
   2. http://www.club.ru/msk/detailed/clubs/art_garbage/menu

Вопрос 14:
В фильме "Пираты" упоминаются две огромные одинаковые пушки. Когда одна
из них стреляет - другая должна молчать, иначе судно может
перевернуться. Какие мифические имена носят эти пушки?

Ответ:
Кастор и Поллукс.

Зачет:
Кастор и Полидевк.

Комментарий:
Близнецы-пушки не могут стрелять одновременно, так же как, согласно
одной из версий мифа, Поллукс и Кастор не могли одновременно находиться
на Олимпе.

Источник:
   1. http://ru.wikipedia.org/wiki/Пираты_(фильм,_1999)
   2. http://ru.wikipedia.org/wiki/Диоскуры

Вопрос 15:
(pic: 20121016.jpg)
   В повести "Тихий Дон" один из станичников рассуждал о проблеме выбора
между красными и белыми. Глава профсоюзов милиционеров Михаил Пашкин в
одном из интервью высказался о переименовании российской милиции в
полицию. Впрочем, они оба в своих рассуждениях исходили из ложных
предпосылок, если верить данным из находящейся перед вами таблицы. В
первой и второй строках этой таблицы заретушировано по одному слову.
Восстановите эти слова в правильном порядке.

Ответ:
Хрен, редька.

Комментарий:
Содержание углеводов (в том числе моно- и дисахаридов) намекает на тот
факт, что хрен редьки все-таки слаще.

Источник:
   1. М. Шолохов. Тихий Дон.
   2. Х. Мураками. Зеленый зверь.
   3. http://www.health-diet.ru/base_of_food/sostav/423.php
   4. http://www.health-diet.ru/base_of_food/sostav/301.php

Вопрос 16:
В 50-х годах XIX века в США полусекретные организации "Орден
объединенных американцев" и "Орден звездно-полосатого флага" образовали
националистическую партию "Незнаек". Основное правило, установленное для
членов этой партии, было использовано спустя почти полтора столетия
другой организацией. Назовите эту организацию.

Ответ:
"Бойцовский клуб".

Комментарий:
Суть правила - никому не рассказывать о партии (или, соответственно, о
клубе).

Источник:
   1. http://ru.wikipedia.org/wiki/Know_Nothing
   2. http://en.wikipedia.org/wiki/Know_Nothing
   3. http://ru.wikipedia.org/wiki/Бойцовский_клуб_(роман)

Вопрос 17:
Анализируя известную картину, искусствоведы сравнивают полотенце на ЕГО
голове с терновым венцом Христа, а нанесенную ЕМУ рану уподобляют следу
копья сотника ЛонгИна. Назовите ЕГО.

Ответ:
[Жан-Поль] Марат.

Комментарий:
Речь идет о картине "Смерть Марата".

Источник:
http://www.vokrugsveta.ru/vs/article/6964/

Вопрос 18:
(pic: 20121017.jpg)
   В 2006 году в Новой Зеландии была проведена компания по сбору
средств, в рамках которой было выпущено 20 тысяч таких пакетиков. Мы не
спрашиваем, на борьбу с чем собирались средства. Назовите одним словом
содержимое пакетиков.

Ответ:
Кетчуп.

Комментарий:
(pic: 20121018.jpg)
   Боролись с противопехотными минами; вот такое "наглядное"
доказательство опасности мин...

Источник:
http://www.theinspirationroom.com/daily/2007/ketchup-campaign-against-landmines/

Вопрос 19:
[Ведущему: отточия в тексте вопроса никак не выделять.]
   Цитата из произведения Николая Костомарова: "Дочери царей Михаила
Федоровича и Алексея Михайловича, людей крайне набожных <...>,
естественно, были осуждены на тюремное заключение при жизни своих отцов
и выходили только в церковь". Какую букву мы заменили в предыдущем
предложении?

Ответ:
Е.

Комментарий:
Заключение, конечно, было теремным...

Источник:
http://stepanov01.narod.ru/library/kostomarov/biographies/sofia.htm

Вопрос 20:
Американский писатель Кристофер Бакли назвал КРУГЛЫЙ СТОЛ "пупом
всемирной истории". Сейчас там можно увидеть бюсты Линкольна и Мартина
Лютера Кинга. Какие два слова мы заменили словами "КРУГЛЫЙ СТОЛ"?

Ответ:
Овальный кабинет.

Комментарий:
Похож на пуп по форме... Каждый президент оформляет свое рабочее место
по собственному вкусу. Линкольн и Мартин Лютер Кинг - кумиры Барака
Обамы.

Источник:
   1. http://www.e-reading.org.ua/bookreader.php/101512/Bakli_-_Den%27_bumeranga.html
   2. http://www.fashiony.ru/page.php?id_n=50445

Вопрос 21:
(pic: 20121019.jpg)
   В одном из разделов интернет-магазина, торгующего различной
фан-атрибутикой, автор вопроса увидел футболку с рисунком, который вы
видите, и надписью-призывом. Если к этой надписи добавить одну букву,
получится фраза, ставшая известной благодаря одному американскому
фильму. Назовите этот фильм.

Ответ:
"Форрест Гамп".

Комментарий:
"Run, Forrest! Run!" [ран форест ран] - знаменитая фраза из кинофильма
"Форрест Гамп". "Forest" (с одной "r") - по-английски "лес". На картинке
лес убегает от бульдозера. Футболка на сайте продается в разделе
атрибутики к фильмам.

Источник:
   1. http://www.teehugger.com/Run_Forest_Run_p/thas1017.htm
   2. http://www.imdb.com/title/tt0109830/quotes?item=qt0373725

Вопрос 22:
Цитата из книги Бориса Акунина "Декоратор": "Днем я ничем не отличаюсь
от некрасивых, жалких, суетливых. Я виртуоз ЕЕ, им ни за что не
догадаться, что я из другой породы". Известно, что из-за НЕЕ можно
спутать вице-короля и монарха. Назовите ЕЕ одним словом.

Ответ:
Мимикрия.

Комментарий:
Вице-король и монарх - это в данном случае бабочки.

Источник:
   1. http://lib.ru/RUSS_DETEKTIW/BAKUNIN/dekorat.txt
   2. http://ru.wikipedia.org/wiki/Вице-король_(бабочка)

Вопрос 23:
Цитата из репортажа об острове Науру: "Основой нового правительства
стали ИГРЕКИ. Президент - штангист, министр иностранных дел и финансов -
регбист, министр транспорта и телекоммуникаций - чемпион по
пауэрлифтингу". Согласно парадоксальному названию статьи, посвященной
национальной безопасности России, "ИГРЕКИ - слабое звено". Назовите
ИГРЕКОВ.

Ответ:
Силовики.

Источник:
   1. http://www.novayagazeta.ru/data/2010/145/18.html
   2. http://www.vz.ru/politics/2010/11/23/449603.html

Вопрос 24:
(pic: 20121020.jpg)
   Изменив одну букву в существующем слове, восстановите
одиннадцатибуквенное слово-неологизм, которое мы заретушировали на этой
фотографии.

Ответ:
Нелеготимна.

Комментарий:
"Команда жуликов и воров" на этой позитивной ноте прощается с вами. :-)

Источник:
http://www.forbes.ru/sobytiya-photogallery/lyudi/78030-iskusstvo-protesta/photo/10

Тур:
21 тур. "Летающий цирк Пеннивайза" (Москва)

Редактор:
Владислав Король (Москва)

Инфо:
Редактор благодарит за помощь в работе над пакетом
редактора-координатора марафона Ивана Старикова (Безансон), Наталию
Новыш (Санкт-Петербург), Серафима Шибанова (Москва), а также тестеров
Сергея Дорофеева (Москва), Александра Колышкина (Санкт-Петербург),
Анвара Мухаметкалиева (Алма-Ата), Андрея Одегова (Киров), Евгения
Рубашкина (Санкт-Петербург), Андрея Солдатова (Москва), Сергея
Терентьева (Санкт-Петербург), Марию Трошкову (Санкт-Петербург).

Вопрос 1:
На постере 1990 года ОН выглядит как ОН. Какие четыре буквы мы
пропустили в предыдущем предложении?

Ответ:
О, К, Л, У.

Комментарий:
Речь идет о фильме "Оно" по одноименному роману Стивена Кинга. В этой
экранизации Оно принимает облик Танцующего клоуна Пеннивайза. В слове
"Оно" мы пропустили букву "О", а в слове "клоун" - буквы "К", "Л" и "У".
Команда "Летающий цирк Пеннивайза" приветствует вас и желает хорошей
игры!

Источник:
http://en.wikipedia.org/wiki/It_(1990_film)

Автор:
Владислав Король, Серафим Шибанов (Москва)

Вопрос 2:
Внимание, словами "ИКС" и "ИГРЕК" заменены другие два слова.
   "Представьте, что ИКС - это вы, мешок - это всё то, что есть у вас и
о чем вы беспокоитесь. А ИГРЕК заставляет вас об этом забыть". О каком
произведении идет речь в этой цитате?

Ответ:
"Ежик в тумане".

Комментарий:
ИКС - ежик, ИГРЕК - туман. Мнение зрителя из США о мультфильме Юрия
Норштейна.

Источник:
http://f5.ru/onionas/post/245811

Автор:
Владислав Король (Москва)

Вопрос 3:
Газетная статья о пожарах на подмосковных торфяниках называлась
"КУРИЛЬСКИЕ ОСТРОВА". Какие два слова мы заменили словами "КУРИЛЬСКИЕ
ОСТРОВА"?

Ответ:
Огненная земля.

Комментарий:
Название одного архипелага заменено названием другого. А неосторожное
обращение с огнем тут ни при чем.

Источник:
"Известия", 23.07.2010 г.

Автор:
Владислав Король (Москва)

Вопрос 4:
(pic: 20121021.jpg)
   В первом предложении статьи, заголовок которой вы видите, упоминаются
ОНА и ОН. Назовите ЕЕ или ЕГО.

Ответ:
Сова.

Зачет:
Жаворонок.

Комментарий:
На картинке жаворонок будит сову.

Источник:
Какой-то женский журнал, вырезка утеряна.

Автор:
Владислав Король (Москва)

Вопрос 5:
Психологи советуют при разговоре с незнакомым человеком не давать ТАКИХ
ответов на вежливые вопросы, задаваемые для поддержания беседы, - иначе
наладить контакт будет сложно. Воспроизведите в своем ответе строку из
стихотворения 1784 года, в которой все восемь слов ТАКИЕ.

Ответ:
"Я царь - я раб - я червь - я бог!".

Комментарий:
ТАКОЙ - это односложный. Строка из стихотворения Гавриила Державина
"Бог".

Источник:
   1. http://polbu.ru/lounders_communications/ch18_all.html
   2. http://www.rvb.ru/18vek/derzhavin/01text/021.htm

Автор:
Владислав Король, Серафим Шибанов (Москва)

Вопрос 6:
Один журналист заметил, что раньше слова "скатертью дорога" были
пожеланием доброго пути. А ЭТИ ТРИ СЛОВА и сегодня в устах
провинциальной торговки семечками звучат не как издевка, а как
руководство к действию. Напишите ЭТИ ТРИ СЛОВА.

Ответ:
Держи карман шире.

Источник:
http://www.vokrugsveta.ru/quiz/675/

Автор:
Владислав Король (Москва)

Вопрос 7:
"Творчество - это отрочество", - считал поэт Андрей Вознесенский, редко
использовавший ЭТО. Поэт Дмитрий Пригов, напротив, обычно ЭТО
использовал. Ответьте точно, какое слово хочет убрать с мемориальной
доски поэта Иосифа Бродского его друг, разделяющий мнение Андрея
Вознесенского.

Ответ:
Александрович.

Комментарий:
Андрей Вознесенский писал: "Нет у поэтов отчества. // Творчество - это
отрочество". Пригов всегда величал себя Дмитрием Александровичем.
Бродский - тоже Александрович.

Источник:
   1. http://www.stihi.ru/2006/07/09-808/
   2. http://misha-banjoman.livejournal.com/46115.html
   3. "Известия", 24.05.2010 г.

Автор:
Владислав Король (Москва)

Вопрос 8:
Песня о НЕМ была популярна в середине девяностых. ОН стал названием
газетной статьи о кёрлинге. Назовите ЕГО двумя словами.

Ответ:
Гранитный камушек.

Зачет:
Гранитный камешек, гранитный камень.

Комментарий:
Песня группы "Божья коровка".

Источник:
"Финансовые известия", 05.08.2010 г.

Автор:
Владислав Король (Москва)

Вопрос 9:
Говоря об экранизации произведения 1966 года, Татьяна Бонч-Осмоловская
употребила слова "рассказ в рассказе в рассказе в рассказе". Назовите
любого из заглавных героев этого произведения.

Ответ:
Розенкранц.

Зачет:
Гильденстерн.

Комментарий:
Процитированная статья называется "Фракталы в литературе: в поисках
утраченного оригинала". Первая итерация наблюдается еще в "Гамлете"
(пьеса "Мышеловка" дублирует ключевые события оригинальной пьесы -
убийство короля и брак цареубийцы с королевой). В пьесе Стоппарда актеры
на репетиции разыгрывают еще и пантомиму на тот же сюжет, а в
экранизации пьесы на сцене появляются еще и марионетки.

Источник:
http://www.textonly.ru/case/?article=9255&issue=16

Автор:
Владислав Король (Москва)

Вопрос 10:
Говоря о невозможности ЕГО создания, Татьяна Бонч-Осмоловская заметила,
что ограниченный человеческий разум не способен справиться с задачей
даже третьего уровня сложности. Назовите ЕГО четырьмя словами.

Ответ:
Венок венков венков сонетов.

Комментарий:
Написанные венки венков сонетов (225 стихотворений) существуют. Фрактал
следующего уровня сложности должен содержать 2455 связанных
стихотворений.

Источник:
http://www.textonly.ru/case/?article=9255&issue=16

Автор:
Владислав Король (Москва)

Вопрос 11:
На вопрос "Что надо делать в 11 часов 11 минут 11 секунд 11 ноября 2011
года?" поступило шуточное предложение ДЕЛАТЬ ЭТО. Назовите человека,
который в 1975 году предложил ДЕЛАТЬ ЭТО.

Ответ:
Булат Окуджава!!!!1111.

Комментарий:
РепьЮниты (натуральные числа, состоящие только из единиц) часто
используются для обозначения экспрессивного восклицания. Стихотворение
"Пожелание друзьям" ("Давайте восклицать") написано в 1975 году.

Источник:
   1. http://a-s-y-a.livejournal.com/90061.html
   2. http://www.bokudjava.ru/D_1.html
   3. http://lurkmore.to/1111

Автор:
Владислав Король (Москва)

Вопрос 12:
(pic: 20121022.jpg)
   Играя вынужденную семерную против программы "Марьяж", автор вопроса
радостно прокомментировал возникшую ситуацию тремя словами. В другой
ситуации эти три слова произнес герой анекдота. Напишите эти три слова.

Ответ:
"Хорошо, что пополам!".

Комментарий:
Легко видеть, что вынужденная семерная в этой пульке в программе
"Марьяж" играется, только если у вистующих старшие козырные бубны на
разных руках. Герой анекдота, врезавшийся на мотоцикле в столб,
радовался, что в формуле кинетической энергии есть множитель 1/2.

Источник:
   1. ЛОАВ.
   2. http://www.anekdot.ru/id/14628/

Автор:
Владислав Король (Москва)

Вопрос 13:
   <раздатка>
   В зеркале отражалась его широкая, светло-серая спина, теневые
перехваты на сгибе рукава, желтые пряди приглаженных волос.
   В зеркале отразилось ее зеленое платье, нежная шея под темной
тяжестью шиньона, блеск мелких жемчужин.
   Он надел макинтош, шляпу, содрогнулся, увидев себя в зеркале,
подхватил чемоданы и вышел в коридор.
   </раздатка>
   Мы раздали вам три цитаты из романа, законченного в 1928 году.
Назовите этот роман.

Ответ:
"Король, дама, валет".

Комментарий:
В трех приведенных цитатах в зеркалах отражаются герои романа - Драйер,
Марта и Франц. Трельяж - это и трехстворчатое зеркало, и король, дама,
валет одной масти.

Источник:
Владимир Набоков. Король, дама, валет.

Автор:
Владислав Король (Москва)

Вопрос 14:
Забавно, но герой известного произведения сумел-таки использовать для
колдовства несколько птичьих перьев, подобранных на дороге. Назовите это
произведение.

Ответ:
"Калевала".

Комментарий:
Лемминкяйнену найденные перья пригодились. А эстонцу из анекдота
подобранная на дороге дохлая ворона не пригодилась.

Источник:
   1. http://www.kalevala.ru/songs/song26a.shtml
   2. http://www.anekdot.ru/id/109114/

Автор:
Наталия Новыш (Санкт-Петербург)

Вопрос 15:
ОН сидел при Викторе Эммануиле III, при Гарри Трумэне и при Дуайте
Эйзенхауэре. Назовите ЕГО фамилию.

Ответ:
Паунд.

Комментарий:
Как и Фунт из "Золотого теленка", американский поэт Эзра Паунд сидел при
нескольких правителях. 3 мая 1945 года он был арестован итальянскими
партизанами за поддержку режима Муссолини, затем отправлен в США, где
отдан под суд за пропаганду фашизма. После признания невменяемым Паунд
до 1957 года содержался в психиатрической больнице тюремного типа.

Источник:
http://en.wikipedia.org/wiki/Ezra_Pound

Автор:
Владислав Король (Москва)

Вопрос 16:
Узнав о том, что последнюю дальневосточную стеллерову корову съел "некий
Попов с товарищами", один писатель заметил, что этот "некий Попов" повел
себя недальновидно. Если бы он после трапезы поступил подобно нашему
современнику - то мог бы стать "тем самым Поповым" или "небезызвестным
Поповым". Назовите этого современника.

Ответ:
Евгений Гришковец.

Комментарий:
"От имени, отчества и биографии осталось только жалкое и тощее "некий".
А вот создал бы книгу под названием "Как я съел стеллерову корову",
стал, может быть, "тем самым", или "известным", или даже
"небезызвестным"". Действие произведения Евгения Гришковца "Как я съел
собаку" также происходит на Дальнем Востоке.

Источник:
Андрей Аствацатуров. Люди в голом.

Автор:
Владислав Король (Москва)

Вопрос 17:
Один житель Эдинбурга полушутя-полусерьезно заявил, что для него важнее
подруги не только любимый ИКС, но даже ненавистный ИГРЕК. ИКС и ИГРЕК -
два слова, начинающиеся на одну и ту же букву. Напишите любое из этих
слов.

Ответ:
"Хартс".

Зачет:
"Хиберниан".

Комментарий:
Шотландские футбольные клубы, участники принципиального эдинбургского
дерби. Есть отечественный аналог этой старой шутки:
   - Тебе твой "Спартак" дороже меня!
   - Знаешь, пожалуй, мне даже ЦСКА дороже тебя.

Источник:
Ирвин Уэлш. Преступление.

Автор:
Владислав Король (Москва)

Вопрос 18:
После исламской революции 1979 года в Иране некоторое время
использовались купюры времен правления шаха, на которых появился ИКС. Ни
до революции, ни после нее ИКС в Иране не использовался. Назовите ИКС
двумя словами, начинающимися на одну и ту же букву.

Ответ:
Красный крест.

Комментарий:
Сейчас вместо красного креста в Иране используют традиционный для
исламских стран красный полумесяц, а до этого - герб "Лев и Солнце". На
купюрах старого образца сначала портрет шаха перечеркивали двумя
красными линиями, а затем запечатывали черной краской.

Источник:
http://ru.wikipedia.org/wiki/Иранский_риал

Автор:
Владислав Король (Москва)

Вопрос 19:
(pic: 20121023.jpg)
   <раздатка>
   Авторы музыкального сопровождения - Daft Punk - ___________ в фильме
и исполнили не самую последнюю роль.
   </раздатка>
   В розданной вам цитате из отзыва о фильме "Трон. Наследие" мы
пропустили глагол. Напишите этот глагол.

Ответ:
Засветились.

Комментарий:
В прямом и в переносном смысле.

Источник:
http://kartaheno.livejournal.com/161760.html

Автор:
Владислав Король (Москва)

Вопрос 20:
Игорь Губерман заметил, что в эпоху застоя Москва "истекала евреями"
через НЕЕ. Назовите ЕЕ одним словом.

Ответ:
Вена.

Комментарий:
   Сегодняшний день лишь со временем
   откроет свой смысл и цену;
   Москва истекает евреями
   через отверстую Вену.
   Через Вену уезжало большинство эмигрантов, прямого авиасообщения
между СССР и Израилем не было.

Источник:
http://lib.ru/GUBERMAN/guberman.txt

Автор:
Владислав Король (Москва)

Вопрос 21:
В одном из эпизодов спектакля по пьесе Янки Купалы актер уносит со сцены
кукол, изображающих польских солдат. На выходе он оборачивается и
бросает в сторону зрителей моток красной тесьмы. Назовите двумя словами
то, чем покрыта в этот момент сцена.

Ответ:
Карта Белоруссии.

Комментарий:
Рассекает карту пополам красной тесьмой. По Рижскому договору после
советско-польской войны запад Белоруссии отошел к Польше.

Источник:
   1. Спектакль "Тутэйшыя" Гродненского кукольного театра.
   2. http://ru.wikipedia.org/wiki/Файл:Caricature_for_Riga_Peace_1921.png

Автор:
Владислав Король (Москва)

Вопрос 22:
Упомянув ЕЕ и ЕГО, поэт-патриот Феликс Чуев заявил, что у России есть
большие неиспользованные резервы. Назовите ЕЕ или ЕГО.

Ответ:
Царь-пушка.

Зачет:
Царь-колокол.

Комментарий:
   У нас еще в достатке матерьяла,
   Который мы не пустим на распыл,
   У нас еще Царь-пушка не стреляла,
   У нас еще Царь-колокол не бил!

Источник:
http://www.duel.ru/200123/?23_7_4

Автор:
Владислав Король (Москва)

Вопрос 23:
Внимание, слова "МЛАДШИЙ" и "СТАРШИЙ" являются заменами.
   В песне Василия Шумова МЛАДШИЙ рифмуется с обманом, а СТАРШИЙ весьма
вольно рифмуется с враньем. В финале другой песни было заявлено, что
МЛАДШЕМУ СТАРШИМ не бывать. Напишите любое из замененных слов.

Ответ:
Капитан.

Зачет:
Майор.

Комментарий:
"Всё обман всё обман / положиться больше не на кого / сказал мне капитан
/ всё вранье всё вранье / верить никому нельзя / подтвердил майор". В
конце песни Высоцкого "Случай в ресторане" в адрес капитана звучат слова
"Никогда ты не будешь майором". Звание "капитан" относится к младшему
офицерскому составу, а "майор" - к старшему. "Вольно" - небольшая
подсказка.

Источник:
   1. http://golovin.evrazia.org/?area=works&article=57&page=814
   2. http://vysockiy.ouc.ru/sluchaj-v-restorane.html

Автор:
Владислав Король (Москва)

Вопрос 24:
   <раздатка>
   Это просто чудо: синие полоски
   На моей озябшей голове.
   От собачьей чумки чебурашки дохнут,
   Эта песня просто ______
   &nbsp;
   Рифмовка "abcb".
   </раздатка>
   Странная песня "Откровение" заканчивается странным семибуквенным
словом. Напишите это слово.

Ответ:
Открове.

Комментарий:
Егор Летов, из раннего.

Источник:
http://www.gitaristu.ru/accords/letter/rus_g/Grazhdanskaya_Oborona/Otkrovenie

Автор:
Владислав Король (Москва)

Тур:
22 тур. "Саморазгрызающийся полубатон" (Москва)

Вопрос 1:
(pic: 20121024.jpg)
   Ответьте максимально точно одним словом, чем занимается человек,
визитку которого мы вам раздали.

Ответ:
Разводами.

Комментарий:
Посередине визитки перфорация: ее можно разорвать надвое, но контакты
юриста будут на обеих половинках.

Источник:
http://www.cardonizer.com/business_cards/james_mahon

Автор:
Владимир Цветков (Москва)

Вопрос 2:
Англичанин Тим Бернерс-Ли очень быстро и непонятно разговаривает,
поэтому его швейцарские коллеги обычно ДЕЛАЛИ с ним ЭТО. После того как
Вьетнам в 1940-е годы захватил часть территории Камбоджи, камбоджийский
король Монивонг в последние месяцы жизни отказывался ДЕЛАТЬ ЭТО. Какие
два слова мы заменили словами "ДЕЛАТЬ ЭТО"?

Ответ:
Говорить по-французски.

Комментарий:
Французский - неродной для Бернерса-Ли, поэтому на нем он не мог так
быстро говорить; Монивонг не без основания полагал, что Вьетнаму
помогали французские власти.

Источник:
   1. http://habrahabr.ru/blogs/it_bigraphy/129518/
   2. Д.П. Чандлер. Брат номер один: Политическая биография Пол Пота. -
Екатеринбург: Ультра Культура, 2005. - С. 42.

Автор:
Наиль Фарукшин (Навои - Москва)

Вопрос 3:
В фильме "Влюбленный Шекспир" показана вражда театров "Занавес" и
"Роза". В одном из эпизодов фильма проповедник, обличая артистов,
восклицает: "[ПРОПУСК] лицедеев!". Заполните пропуск четырьмя словами
максимально точно.

Ответ:
"Чума на оба дома...".

Комментарий:
Услышавший эту фразу Шекспир по сюжету фильма заимствовал ее для своей
пьесы.

Источник:
Фильм "Влюбленный Шекспир" (1998), реж. Дж. Мэдден.

Автор:
Наиль Фарукшин (Навои - Москва)

Вопрос 4:
На одном коллаже, опубликованном в ЖЖ-сообществе antropology_ru,
животное идет по изображению четырех людей. Назовите это животное.

Ответ:
Зебра.

Комментарий:
Коллаж представляет собой как бы вывернутую обложку альбома "Битлз"
"Abbey Road".

Источник:
http://anthropology-ru.livejournal.com/600465.html

Автор:
Софья Филина (Москва)

Вопрос 5:
Блиц.
   Владимир Бабенко рассказывает о Сторожевой башне сардинской крепости
Миолан.
   1. Какое прозвище получил цокольный этаж этой башни, в котором
располагались карцеры?
   2. Какое прозвище получила верхняя площадка башни, из которой
открывался превосходный вид?
   3. Какое прозвище получила тюремная камера над цокольным этажом?

Ответ:
   1. Ад.
   2. Рай.
   3. Чистилище.

Зачет:
   1. Преисподняя.

Источник:
В.Г. Бабенко. Маркиз де Сад: Философ и распутник. - М.: АСТ-ПРЕСС, 2007.
- С. 132-133.

Автор:
Наиль Фарукшин (Навои - Москва)

Вопрос 6:
Автор вопроса считает, что Бахус не пользовался РЕФЛЕКТОРОМ, чтобы
принудить ЮПИТЕРА освободить Юнону. Какие слова мы заменили словами
"РЕФЛЕКТОР" и "ЮПИТЕР"?

Ответ:
Кратер, Вулкан.

Комментарий:
Увидев, что Вулкан уродлив, родившая его Юнона отреклась от него и
сбросила его со скалы. Выжив и решив отомстить, бог кузнечного ремесла
выковал в подарок своей матери трон, сев на который та оказалась к нему
прикованной. Чтобы заставить Вулкана освободить Юнону, Юпитер послал
Бахуса опоить кузнеца и привести его на Олимп. Бахус вряд ли пользовался
кратером - сосудом для смешивания вина, ведь чистым вином можно было
быстрее опоить бога. Рефлектор - часть осветительного прибора юпитера, и
примерно похож на кратер у вулкана.

Источник:
   1. http://ru.wikipedia.org/wiki/Гефест
   2. http://ru.wikipedia.org/wiki/Кратер_(сосуд)
   3. Общие соображения автора вопроса.

Автор:
Наиль Фарукшин (Навои - Москва)

Вопрос 7:
Название книги гроссмейстера Бронштейна, посвященной его партиям с
шахматными компьютерами, состоит из трех слов. Второе слово названия -
"против". Назовите первое и третье.

Ответ:
Давид, Голиаф.

Комментарий:
Голиафом Давид Бронштейн назвал шахматные компьютеры.

Источник:
Д.И. Бронштейн, С.Б. Воронков. Давид против Голиафа. - М.: Рипол
Классик, 2002.

Автор:
Наиль Фарукшин (Навои - Москва)

Вопрос 8:
Неграмотный герой миниатюры Наринэ Абгарян принял встреченные им в
стихотворении плеоназмы за спазмы. Какие слова мы заменили словами
"плеоназмы" и "спазмы"?

Ответ:
Буколики, колики.

Источник:
http://greenarine.livejournal.com/40635.html

Автор:
Владимир Цветков (Москва)

Вопрос 9:
Изобретатель Франк Маседа изобрел машинку для того, чтобы ДЕЛАТЬ ЭТО -
причем не для противозаконных целей, а чтобы избавить от бактерий,
плесневых грибов и других форм жизни поверхность... Поверхность чего?

Ответ:
Денег.

Комментарий:
Он придумал машинку для отмывания денег - в прямом смысле.

Источник:
"Discovery", 2011, N 10. - С. 10.

Автор:
Наиль Фарукшин (Навои - Москва)

Вопрос 10:
Некий Жан-Батист Перес доказывал, что ИКС на самом деле не существовал и
был лишь персонификацией ИГРЕКА. Ведь ИКС, как и ИГРЕК, родился на
острове в Средиземном море, его мать звали Летиция, да и имена ИКСА и
ИГРЕКА созвучны. Назовите ИКСА и ИГРЕКА.

Ответ:
Наполеон [Бонапарт], Аполлон.

Комментарий:
Аполлон тоже родился на острове в Средиземном море, его мать звали Лето.
Также в книжке Переса утверждалось, что три сестры Наполеона - это
грации, а четыре брата - времена года.

Источник:
П.Н. Берков. О людях и книгах. - М.: Книга, 1965. - С. 89.

Автор:
Наиль Фарукшин (Навои - Москва)

Вопрос 11:
"ОНА" - название настольной игры, которую часто называют
"коммунистической Монополией". ОНА упоминается в статье об айтыше -
своеобразном состязании двух акынов. Назовите ЕЕ одним словом.

Ответ:
Очередь.

Комментарий:
В настольной игре всё нужно покупать, отстояв очередь. Акыны выступают
по очереди.

Источник:
   1. http://ru.wikipedia.org/wiki/Очередь_(игра)
   2. http://ru.wikipedia.org/wiki/Айтыш

Автор:
Наиль Фарукшин (Навои - Москва)

Вопрос 12:
В фильме Криса Коламбуса недавно приехавший в США автомеханик говорит
своей клиентке: "Давайте посмотрим вашу вольво". Какое слово мы заменили
в тексте этого вопроса?

Ответ:
Гинеколог.

Комментарий:
Он очень плохо знал русские слова. :-)

Источник:
Фильм "Девять месяцев" (1995), реж. Крис Коламбус.

Автор:
Наиль Фарукшин (Навои - Москва)

Вопрос 13:
   Нет ветра сегодня, нет мочи стоять.
   Согрелась вода. Душно, жарко.
   Термометр поднялся аж на сорок пять,
   Без воздуха вся кочегарка!
   Действие песни "Раскинулось море широко" происходит в Красном море.
Какую французскую фамилию упоминает Марк Блау, комментируя эти строчки?

Ответ:
Реомюр.

Комментарий:
45 градусов Цельсия - это для Красного моря еще не очень жарко. Видимо,
в песне имеется в виду 45 градусов по Реомюру (56 градусов Цельсия).

Источник:
М. Блау. От добермана до хулигана.

Автор:
Наиль Фарукшин (Навои - Москва)

Вопрос 14:
Жителей греческой колонии Массилии (ныне - Марсель), куда часто
отправляли обучать своих детей римляне, часто называли ТАКИМИ. ТАКИМ был
отрицательный персонаж русского фольклора. Какое слово мы заменили
словом "ТАКОЙ"?

Ответ:
Трехъязычный.

Зачет:
Трехъязыкий.

Комментарий:
Они знали греческий, латынь и галльский. Персонаж - Змей Горыныч.

Источник:
Н. Будур. Повседневная жизнь инквизиции в средние века. - М.: Молодая
гвардия, 2011. - С. 23.

Автор:
Наиль Фарукшин (Навои - Москва)

Вопрос 15:
До начала 1960-х годов Владимир Войнович работал пастухом, столяром,
авиамехаником, а после этого стал популярным поэтом-песенником. По этому
поводу он шутил, что ОНА его не коснулась. Назовите ЕЕ максимально
точно.

Ответ:
Денежная реформа [1961 года].

Комментарий:
В результате денежной реформы 1961 года стоимость рубля возросла в 10
раз и один ноль у всех цен убрали. С популярностью к Войновичу пришли и
высокие гонорары: он получал 500-600 рублей как до реформы, так и после.

Источник:
   1. В.Н. Войнович. Антисоветский Советский Союз. - М.: Материк, 2002.
- С. 216.
   2. http://ru.wikipedia.org/wiki/Войнович,_Владимир_Николаевич

Автор:
Наиль Фарукшин (Навои - Москва)

Вопрос 16:
В статье о макетах бронетехники Тим Скоренко пишет, что в условиях
современной войны обмануть противника легче, чем победить его в честном
бою. Какое слово мы заменили в этом вопросе?

Ответ:
Надуть.

Комментарий:
Макеты - пневматические.

Источник:
"Популярная механика", 2011, N 7. - С. 95.

Автор:
Наиль Фарукшин (Навои - Москва)

Вопрос 17:
Заполните пропуски в стихотворении Владимира Резниченко:
   К русичам явились [пропуск 1],
   В дар преподнесли им [пропуск 2].

Ответ:
"... половцы...", "... пол-овцы".

Источник:
http://samlib.ru/r/reznichenko_w_e/vdolesovdolli.shtml

Автор:
Владимир Цветков (Москва)

Вопрос 18:
[Ведущему: XX:YY - это значение времени в данный момент в Пущино минус 1
час. Например, если в Пущино сейчас 13:44, то XX:YY - "двенадцать часов
сорок четыре минуты".]
   Внимание, в данном вопросе содержится замена.
   По данным Википедии, в XX:YY значительная власть в империи была
сосредоточена у евнухов. Напишите два слова, которые мы заменили на
XX:YY.

Ответ:
Минское время.

Комментарий:
Речь идет о времени династии Мин и минской империи. XX:YY - время в
Минске в момент зачтения вопроса.

Источник:
http://ru.wikipedia.org/wiki/Империя_Мин

Автор:
Владимир Цветков (Москва)

Вопрос 19:
Дуплет.
   (pic: 20121025.jpg)
   1. Перед вами взгляд жителя метрополии. Напишите существительное,
которое мы скрыли за знаком вопроса.
   (pic: 20121026.jpg)
   2. Какое существительное мы закрыли на карикатуре?

Ответ:
   1. Австралиец.
   2. Креационист.

Источник:
Интернет.

Автор:
Полина Гершберг (Москва)

Вопрос 20:
Цитата из Васи Уткина: "Пуйоль принял мяч и ТОТЧАС ПАСАНУЛ". Какие два
слова мы заменили в комментарии Васи?

Ответ:
Немедленно выбил.

Комментарий:
"Пуйоль принял мяч и немедленно выбил". Уменьшительное "Вася" должно
было напомнить вам о Веничке Ерофееве, на которого и аллюзировал Уткин.

Источник:
http://www.sports.ru/conference/utkin/132031404.html

Автор:
Наиль Фарукшин (Навои - Москва)

Вопрос 21:
"Бестолковый словарь" называет валюту Евросоюза ЕЮ. "ОНА" - шуточная
разновидность математической игры "Рассада". Назовите ЕЕ двумя словами.

Ответ:
Брюссельская капуста.

Источник:
   1. http://ledinoi-ogonj.sitecity.ru/stext_1703185426.phtml
   2. http://ru.wikipedia.org/wiki/Рассада_(игра)

Автор:
Владимир Цветков (Москва)

Вопрос 22:
Внимание, в вопросе есть замена.
   Согласно проекту китайского правительства, Харбин сможет стать первым
ТАКИМ городом благодаря низкой среднегодовой температуре, протекающей
неподалеку реке и бесперебойной подаче энергии. Другой ТАКОЙ город
находится далеко-далеко от Харбина и расположен приблизительно в 60 км
над поверхностью. Какое слово мы заменили словом "ТАКОЙ"?

Ответ:
Облачный.

Комментарий:
Харбин станет облачным городом, потому что в нем планируется разместить
вычислительные центры для облачных технологий. Наилучшее положение для
таких центров - города с низкой температурой и водой для водяного
охлаждения. Другой облачный город расположен на планете Беспин, он парит
в облаках и в нем происходит действие пятого и шестого эпизодов
"Звездных войн".

Источник:
   1. http://www.cnews.ru/news/line/index.shtml?2011/07/25/448636
   2. http://ru.starwars.wikia.com/wiki/Облачный_город

Автор:
Владимир Цветков (Москва)

Вопрос 23:
Герой одного фильма - музыкант - хочет стать каскадером. Режиссер
предупреждает, что тот может сменить скрипку на... На какой струнный
инструмент?

Ответ:
На арфу.

Комментарий:
Считается, что в раю все играют на арфах.

Источник:
Фильм "Поющие под дождем" (1952), реж. С. Донен, Дж. Келли.

Автор:
Полина Гершберг (Москва)

Вопрос 24:
Продолжая известную фразу, Виктор Гюго говорил, что булыжник,
вывороченный из мостовой, - это ИКС народов. Какие два слова мы заменили
ИКСОМ?

Ответ:
Последний довод.

Комментарий:
Последний довод королей - пушки, последний довод народов - булыжник,
вывороченный из мостовой.

Источник:
http://www.bezsekretov.com/item500

Автор:
Владимир Цветков (Москва)

Тур:
23 тур. "Не Мигая" (Москва)

Вопрос 1:
В статье о фильме "Апокалипсис сегодня" рассказывается о предпремьерных
показах. Фрэнсис Форд Коппола демонстрировал фильм английским режиссерам
Кингу Видору и Фрэнку Капре и японцу Акире Куросаве в надежде получить
от них советы. Согласно статье, режиссеры "вставили свои два ПЕРВЫХ и
одну ВТОРУЮ". Что мы заменили на "ПЕРВЫЙ" и "ВТОРАЯ"?

Ответ:
Пенс, иена.

Комментарий:
Так как режиссеры иностранцы, они вставляли не "свои пять копеек", а
национальные валюты. Вопросы про "Апокалипсис сегодня" на этом
закончены.

Источник:
Журнал "EMPIRE", 2011, N 5.

Автор:
Игорь Яшков

Вопрос 2:
   <раздатка>
   GTA Возвращается
   </раздатка>
   Перед вами заголовок новости на сайте ag.ru [эй-джи точка ру] от 25
октября 2011 года, посвященной анонсу очередной части игры "GTA"
[джи-ти-эй]. Мы заменили в нем одну букву и, чтобы вы заметили эту
замену, сделали раздатку. Напишите замененную букву.

Ответ:
V.

Комментарий:
Новость посвящена выходу пятой части игры.

Источник:
http://www.ag.ru/news/id/id22739

Автор:
Игорь Яшков

Вопрос 3:
В 2011 году Владимир Сорокин стал лауреатом премии "Новая словесность".
Статья об этом в газете "Коммерсантъ" называется "Владимир Сорокин УШЕЛ
НЕСОЛОНО ХЛЕБАВШИ". Какие три слова в названии статьи мы заменили?

Ответ:
Остался с "Носом".

Зачет:
Остался с носом.

Комментарий:
Премия "Новая словесность" имеет сокращенное название "НОС".

Источник:
http://www.kommersant.ru/doc/1576805

Автор:
Игорь Яшков

Вопрос 4:
Парадоксальный вопрос.
   Прослушайте цитату из статьи о сибирских поселках: "Парикмахер Лена
Пчелкина стрижет в шести <...> поселениях. Поскольку она единственный
парикмахер в здешних местах, то ...". Закончите цитату тремя словами,
начинающимися на одну и ту же букву.

Ответ:
"... себя стрижет сама".

Зачет:
По смыслу с учетом требования "три слова на одну и ту же букву".

Комментарий:
Существует известный парадокс брадобрея, который бреет только тех
мужчин, которые не бреются сами. Спрашивается, бреет ли этот брадобрей
сам себя.

Источник:
http://rusrep.ru/article/2011/09/07/superheroes/

Автор:
Игорь Яшков

Вопрос 5:
Внимание, в вопросе есть замена.
   В статье "Йемен под катом" утверждается, что "политическую обстановку
Йемена без ста грамм не объяснишь". Какое слово в цитате мы заменили?

Ответ:
Разжуешь.

Комментарий:
В заголовке статьи обыгрывается название легкого наркотика ката,
популярного в Йемене и представляющего собой мелкие листики, которые
необходимо жевать для получения необходимого эффекта.

Источник:
http://rusrep.ru/article/2011/11/29/yemen/

Автор:
Игорь Яшков

Вопрос 6:
Продолжим тему.
   У американской группы "Ministry [мИнистри] есть альбом под названием
"АЛЬФА ложки" - по одной из версий, это намек на наркотическую
зависимость участников группы. Напишите два слова, которые мы заменили
на АЛЬФУ.

Ответ:
Темная сторона.

Комментарий:
Печенье имеет вид "Звезды смерти" из "Звездных войн". Кроме того,
название является намеком на известную фразу про темную сторону и
печеньки. Альбом группы "Ministry" называется "Dark Side of the Spoon" -
это не только отсылка к названию альбома "Pink Floyd", но и намек на
закопченную сторону ложки, которую нагревают при растворении героина.

Источник:
   1. http://yegorka.livejournal.com/1688799.html
   2. http://ru.wikipedia.org/wiki/Dark_Side_of_the_Spoon

Автор:
Игорь Яшков

Вопрос 7:
   <раздатка>
   REALITY
   </раздатка>
   Одна из моделей профессиональных видеокамер, с помощью которой с 2009
года снимаются самые современные фильмы для большого экрана, называется
"REALITY" [реАлити]. В этом названии мы заменили три на две. Напишите
исходное название.

Ответ:
3ALITY.

Зачет:
THREEALITY.

Комментарий:
Камера, использовавшаяся, например, при съемках "Хоббита", позволяет
снимать трехмерное видео. В названии обыгрывается сочетание трех
измерений и слова "реальность". В раздатке мы действительно заменили три
на две - число 3 на две буквы RE.

Источник:
http://www.3alitytechnica.com/

Автор:
Игорь Яшков

Вопрос 8:
Внимание, в вопросе есть замены.
   В стихотворении Егора Сальникова рассказывается об игре, которая
называлась "Дай взятку республиканцу". В песне Аллы Пугачевой
рассказывается, как республиканец получился вместо бытового прибора.
Какие слова заменены на "дай взятку" и "республиканец"?

Ответ:
Купи слона.

Комментарий:
Слон - символ республиканской партии США. В стихотворении Егора
Сальникова "Счастье" есть строка: "Игра называлась Купи слона!". В песне
Аллы Пугачевой "Волшебник-недоучка" есть строки: "Сделать хотел утюг -
слон получился вдруг".

Автор:
???

Вопрос 9:
(pic: 20121027.jpg)
   Перед вами фотография актера, в 2010 году вернувшегося к актерской
карьере после объявления об окончательном уходе. Назовите его фамилию.

Ответ:
Феникс.

Комментарий:
Актер Хоакин Феникс в 2010 году снялся в псевдодокументальном фильме
Кейси Аффлека о собственном уходе.

Источник:
   1. http://www.etoday.ru/2010/08/fotograf-mark-abrahams.php
   2. http://www.celebrific.com/11717/joaquin-phoenix-is-back/

Автор:
Игорь Яшков

Вопрос 10:
В статье братьев Тур, опубликованной 1 января 1937 года, говорится, что
когда происходит ЭТО, кажется, что стенки царапает когтями молодая
кошка. Какие два слова мы заменили на "ЭТО"?

Ответ:
Брожение вина.

Зачет:
Брожение шампанского.

Комментарий:
Ну о чем может быть новогодняя статья? Имеется в виду процесс
образования игристого вина в специальном сосуде - акротофоре.

Источник:
"Известия", 01.01.1937 г. - С. 2.

Автор:
???

Вопрос 11:
Категория "Романы 2009 года" на сайте Википедии содержит среди прочих
следующие ссылки: "Вселенная Метро 2033", "Метро 2034", "12 несогласных"
и "1984". Какой символ мы заменили в предыдущем предложении?

Ответ:
Q.

Комментарий:
Роман Харуки Мураками "1Q84", вышедший в 2009 году.

Источник:
http://ru.wikipedia.org/wiki/Категория:Романы_2009_года

Автор:
???

Вопрос 12:
Мы не спрашиваем у вас, почему Киев - мать городов русских. В
стихотворении лета 1937 года говорится:
   Кто трудами бывших грабарей,
   Тех, кто техникой сложнейшей горд,
   Сделал [ТРИ СЛОВА ПРОПУЩЕНО]
   Красную Москву, чудесный город!
   Назовите первое и третье пропущенные слова.

Ответ:
Порт ... морей.

Комментарий:
В стихотворении, посвященном строительству канала Москва - Волга,
говорится о Москве как о порте "трех морей". Волго-Донской канал был
открыт позже, в 1952 году. Первая фраза вопроса - отсылка к фильму "О
чем говорят мужчины" ("Почему Киев - мать городов русских? А почему
Москва - порт пяти морей?").

Источник:
"Правда", 14.07.1937 г. - С. 4.

Автор:
???

Вопрос 13:
   <раздатка>
   Женщина, какую вы частенько навещаете в "Сладостных чарах",
информировала Сеньора Президента о вашем революционном "фанфаронстве".
   </раздатка>
   Вопрос задает Джефферсон.
   Перед вами цитата из романа Мигеля Астуриаса "Сеньор Президент", в
которой мы заменили одну букву. Какую?

Ответ:
Р.

Комментарий:
Героя зовут Мигель Фарфан. Его, соответственно, обвиняли в
"фарфаронстве". Имя Джефферсон - намек на перуанского футболиста
Джефферсона Фарфана, играющего за "Шальке".

Источник:
   1. М.А. Астуриас. Сеньор Президент.
   2. http://ru.wikipedia.org/wiki/Фарфан,_Джефферсон

Автор:
???

Вопрос 14:
Комментируя один из матчей Национальной хоккейной лиги, комментатор,
извинившись, заметил, что на трибунах "яблоку негде упасть". Команда из
какого города принимала участие в игре?

Ответ:
Нью-Йорк.

Комментарий:
НХЛ - североамериканская хоккейная лига. Речь идет о "Зимней классике" -
2012, в которой играли "Филадельфия Флайерс" и "Нью-Йорк Рейнджерс".

Источник:
"Филадельфия Флайерс" - "Нью-Йорк Рейнджерс", эфир телеканала "Россия
2", 02.01.2012 г.

Автор:
???

Вопрос 15:
"Тромплёй" в переводе с французского означает "обман зрения". Этим
словом называют прием в искусстве, целью которого является создание
оптической иллюзии того, что изображенный объект реален. Назовите
литературного персонажа, который от голода, а не из шалости, испортил
такой "тромплёй".

Ответ:
Буратино.

Комментарий:
Очаг в каморке папы Карло вполне можно назвать "тромплёем".

Источник:
   1. http://ru.wikipedia.org/wiki/Обманка
   2. http://sheba.spb.ru/lib/buratino.htm#4

Автор:
Кира Полевая

Вопрос 16:
Закончите анекдот двумя словами, которые должны являться мечтой каждого
знатока.
   Мама - сыну: "Если завтра за контрольную получишь пятерку, разрешу
тебе играть в компьютер весь день. Если получишь четверку - только до
семи вечера. Если тройку - только один час. Если двойку - вообще не
подойдешь к компьютеру. Ну а если схватишь кол, придется тебе...".

Ответ:
"... почитать книгу".

Источник:
http://www.anekdot.ru/id/189291/

Автор:
Максим Егоров

Вопрос 17:
Собака породы "лабрадор", в отличие от некоторых других пород, свое
название получила не по названию места, где была выведена. Назовите
остров, на котором был выведен лабрадор.

Ответ:
Ньюфаундленд.

Комментарий:
Собака породы Ньюфаундленд уже была. :-)

Источник:
http://ru.wikipedia.org/wiki/Лабрадор_ретривер

Автор:
Алексей Полевой (гордый хозяин лабрадора Кими)

Вопрос 18:
   <раздатка>
   В любом случае, костров Магеллан углядел много, на землю эту решил не
сходить на всякий случай.
   </раздатка>
   Это фраза из статьи, рассказывающей об открытии архипелага Огненная
земля. Какое слово в ней мы заменили?

Ответ:
Пожарный.

Комментарий:
Решил не сходить на всякий пожарный.

Источник:
http://znaeteli.ru/2010/08/kto-vinovat-magellan/

Автор:
Виталий Захарик

Вопрос 19:
Она спала на чердаке, под самой крышей, на колючей соломенной подстилке.
А на чем она сидела вечером, после работы?

Ответ:
На ящике с золой.

Зачет:
По слову "зола".

Комментарий:
Речь о Золушке.

Источник:
Ш. Перро. Золушка.

Автор:
Максим Егоров

Вопрос 20:
Внимание!
   Недостоверная информация.
   Что-то предполагающее понимание по догадке.
   Основная форма организации обучения по отношению к незлым особям
мужского пола.
   Назовите слово, известное вам с детства, к которому всё это имеет
отношение, согласно произведению классика.

Ответ:
Сказка.

Комментарий:
Сказка - ложь, да в ней намек, добрым молодцам урок.

Источник:
http://www.rvb.ru/pushkin/01text/03fables/01fables/0801.htm

Автор:
Максим Егоров

Вопрос 21:
Игроки "Что? Где? Когда?", как известно, очень не любят вопросы, которые
они уже слышали. Поэтому этот вопрос, задававшийся пять раз известной
героиней произведения 1833 года, наверное, знатокам не понравился бы. Мы
не просим отвечать на этот вопрос. Воспроизведите этот вопрос из четырех
слов абсолютно точно.

Ответ:
"Чего тебе надобно, старче?".

Комментарий:
"Сказка о рыбаке и рыбке" А.С. Пушкина.

Источник:
http://www.rvb.ru/pushkin/01text/03fables/01fables/0799.htm

Автор:
Максим Егоров

Вопрос 22:
В дореволюционной России это делалось с помощью метровых белых палок.
После революции палки, конечно, поменяли цвет на идеологически верный -
красный. В 1922 году длина палок была уменьшена (в половину) до 49 см, а
в 1930 году их вообще упразднили и возродили только в 1939 году. А что
делали при помощи вышеупомянутых палок?

Ответ:
Регулировали движение.

Комментарий:
Палка приобрела черно-белую окраску и трансформировалась в жезл
регулировщика.

Источник:
http://znaeteli.ru/2010/11/polosatye-palochki/

Автор:
Михаил Куценко

Вопрос 23:
[Ведущему: так как ударения в слове "пропуск" и пропущенном слове не
совпадают, выделить голосом слова "на педали", чтобы игроки смогли
почувствовать ритм и рифму стиха.]
   Стихи Андрея Вознесенского отличаются яркой, до парадоксальности,
образностью. Прослушайте отрывок из стихотворения, посвященного
скромному представителю одной творческой профессии, в котором мы дважды
пропустили одно и то же слово в разных падежах:
   И летел он на [ПРОПУСКЕ],
   Нажимая на педали,
   У [ПРОПУСКА] есть одно крыло...
   Догадавшись, какой не предназначенный для полетов предмет, мы
заменили словом "ПРОПУСК", назовите профессию героя стихотворения
абсолютно точно, тем более что поэт настоятельно советует его полюбить.

Ответ:
Пианист. Незачет: музыкант.

Комментарий:
Летел он на РОЯЛЕ.

Источник:
А. Вознесенский, "Полюбите пианиста".

Автор:
Людмила Гречаник

Вопрос 24:
В ноябре 2010 года в ДК Московского энергетического института проходил
Всероссийский открытый чемпионат по спидкубингу. Мировой рекорд в
спидкубинге на тот момент принадлежал австралийцу и составлял 6,77...
Чего?

Ответ:
Секунды.

Комментарий:
От англ. "speedcubing" - скоростная сборка кубика Рубика, соревнования
проходят под эгидой WCA (Всемирной ассоциации кубика); данный рекорд
установил мальчик Феликс Земдегс из Австралии.

Источник:
"Смена", 2010, N 12.

Автор:
Ольга Неумывакина

Тур:
24 тур. "С потолка" (Москва)

Вопрос 1:
[Ведущему: кавычки не озвучивать!]
   Писателя Ярослава Гашека очень любили в Советском Союзе. И
рассказывали про него, в частности, такую историю. Однажды во время
чрезвычайного положения полицейский патруль обнаружил в карманах у юного
Ярослава Гашека это "оружие". Гимназиста собирались расстрелять, но
потом всё же отпустили, когда Гашек пояснил, что приобрел его для
школьной коллекции. Что это было за оружие?

Ответ:
Булыжник.

Зачет:
Камень.

Комментарий:
Дескать, приобрел для минералогической коллекции. Комиссар-чех сделал
вид, что поверил, и отпустил.

Источник:
В. Татаринов. Неисправимый анархист и народный затейник. / Я. Гашек.
Похождения бравого солдата Швейка. - М.: Эксмо, 2006. - С. 7.

Автор:
Николай Чудаков

Вопрос 2:
Внимание, в вопросе есть замена.
   В фильме "Мама напрокат" происходит такой диалог.
   - Как тебя зовут?
   - [пропуск].
   - Разве это имя? Так меня мама называет, потому что я люблю ОТДЫХАТЬ.
   Догадайтесь, какое слово мы заменили словом "ОТДЫХАТЬ", и назовите
это имя.

Ответ:
Соня.

Комментарий:
ОТДЫХАТЬ - спать.

Источник:
Фильм "Мама напрокат" (2010).

Автор:
Максим Егоров

Вопрос 3:
По мнению остроумных участников "Квартета И", эта знаменитая фраза из
четырех слов на самом деле является первым в мире слоганом обувной
фирмы. Напишите эту фразу.

Ответ:
"Каждой твари по паре".

Источник:
Самая смешная книга: мексиканские негодяи и не только... / Театр Квартет
И. - М.: АСТ, 2008.

Автор:
Маргарита Рыськова

Вопрос 4:
После зимней Олимпиады на одном интернет-сайте появилась статья,
посвященная разочарованию в одном из видов спорта. Заголовок статьи
всего на одну букву отличается от популярной строчки из песни об этом
виде спорта. Напишите этот заголовок.

Ответ:
"Рус не играет в хоккей".

Автор:
???

Вопрос 5:
Сын и наследник правителя гуинской кочевой державы Маодунь выпросил у
отца отряд отборной конницы и стал обучать его в духе беспрекословного
повиновения. Однажды он пустил стрелу в любимого скакуна. Некоторые
воины решили, что Маодунь ошибся, и не выстрелили вслед. Им тут же
отрубили головы. Позже Маодунь выстрелил в любимую жену и вновь казнил
тех, кто не последовал его примеру. Зато третий выстрел Маодуня дружно
повторили все его воины. А в кого была направлена эта стрела?

Ответ:
В отца Маодуня.

Зачет:
В правителя и т.п. по смыслу.

Комментарий:
Таким вот образом Маодунь и пришел к власти.

Источник:
В. Никонов, Ю. Худяков. "Свистящие стрелы" Маодуня и "Марсов меч"
Аттилы. - СПб.: Петербургское востоковедение, 2004. - С. 21-22.

Автор:
Владимир Крикунов

Вопрос 6:
Рассказывая о своей поездке в одну латиноамериканскую страну, ЖЖ юзер
hectop рассказал, что купил там много дешевой и качественной одежды. Он
сделал вывод, что экономику этой страны можно поддерживать не только
своим... Закончите его фразу словом, которое в именительном падеже
является названием литературного произведения.

Ответ:
"... носом".

Комментарий:
Речь идет о Колумбии.

Источник:
http://hectop.livejournal.com/

Автор:
???

Вопрос 7:
В романе "Капитан Фракасс" про одного второстепенного персонажа говорят,
что его украсили, чтобы тот не потерялся. Назовите одного из тех двоих,
которые были ошеломлены при виде аналогичного украшения,
"принадлежавшего" прекрасной даме.

Ответ:
Атос.

Зачет:
Д'Артаньян.

Автор:
Николай Чудаков

Вопрос 8:
Знаменитый сыщик Филиппов раскрыл одно очень сложное дело исключительно
с помощью новой науки - дактилоскопии. Лев Лурье так пишет об этом:
"Филиппов понимает: как ни иронизируй над ПЕРВЫМ и его научными
методами, а ВТОРОЕ началось". Назовите ПЕРВОГО и ВТОРОЕ.

Ответ:
[Шерлок] Холмс, двадцатое столетие.

Источник:
Лев Лурье. Преступления в стиле модерн.

Автор:
???

Вопрос 9:
   <раздатка>
   Несколько лет назад вышел сборник литературы под названием "Поколение
NEXT".
   </раздатка>
   В одном из слов этой фразы мы заменили две буквы. А в другом слове -
добавили одну лишнюю букву. Напишите два измененных нами слова в
первоначальном виде.

Ответ:
Сетература, NET.

Автор:
Николай Чудаков

Вопрос 10:
Создатели сайта, посвященного поиску и предложению работы в сфере
танцев, назвали его неологизмом, который всего одной буквой отличается
от реально существующего слова, более широкого по смыслу. Напишите этот
неологизм по-русски или по-английски.

Ответ:
Фридансер.

Зачет:
Freedancer.

Автор:
Николай Чудаков

Вопрос 11:
Рассказывая о возникновении монофелитства, Хью Кеннеди приводит
шотландский анекдот: "В маленький городок приходит чужеземец и
спрашивает местного жителя: сколько у вас церквей (религиозных течений).
Тот отвечает: было две, но потом мы [ПРОПУСК], и теперь их три". Какой
глагол мы пропустили?

Ответ:
Объединили.

Зачет:
Объединились.

Комментарий:
Объединение приняла только часть приверженцев каждой из существовавших
религий.

Источник:
http://www.litmir.net/br/?b=145688&p=3

Автор:
Николай Чудаков

Вопрос 12:
(pic: 20121028.jpg)
   С раздатки мы убрали две фамилии, обозначавших авторов этих
"пирамид". Назовите вторую фамилию.

Ответ:
Леннон.

Комментарий:
Пирамида слева - это пирамида человеческих потребностей Абрахама Маслоу.
По-английски потребности - "needs", а "All you need is love" -
знаменитый призыв из песни "The Beatles".

Источник:
(pic: 20121029.jpg)

Автор:
???

Вопрос 13:
Согласно интернет-шутке, ОН был самым успешным тренером покемонов. ОН не
только "собрал их всех", но и сделал это два раза. Назовите ЕГО.

Ответ:
Ной.

Комментарий:
Покемоны - это такие зверушки из известного мультсериала, где целью
главного героя было собрать всех покемонов по одному экземпляру, но ни
ему, ни другим персонажам этого не удалось. А вот Ной на своем ковчеге
смог собрать "всех" зверушек, да еще и в двух экземплярах.

Источник:
(pic: 20121030.jpg)

Автор:
???

Вопрос 14:
   <раздатка>
   - Я бывший солдат! Я убивал людей!
   - Ты же был врачом!
   - Бывали ТАКИЕ ОНИ!
   </раздатка>
   Перед вами цитата из сериала "Шерлок", по мотивам произведения Артура
Конан-Дойля. Согласно пессимистическому афоризму журналистки Эрмы
Бомбек, ТАКИЕ ОНИ чередуются с очень ТАКИМИ. Что мы заменили на "ТАКИЕ
ОНИ"?

Ответ:
Плохие дни.

Комментарий:
Собственно на раздатке дан диалог между Холмсом и Ватсоном в современном
варианте "Записок о Шерлоке Холмсе". Несмотря на то, что Ватсон, скорее
всего, шутил, день, когда военврачу приходится убивать людей с оружием в
руках, сложно назвать хорошим. Журналистка Эрма Бомбек писала, что "день
на день не приходится: плохие дни чередуются с очень плохими".

Источник:
   1. Сериал "Шерлок", s02e01 "Скандал в Белгравии".
   2. http://www.aphorism.ru/author/a1848.shtml

Автор:
???

Вопрос 15:
(pic: 20121031.jpg)
   Перед вами рекламные плакаты одного предприятия. Что это за
предприятие?

Ответ:
Типография.

Комментарий:
Если приглядеться, можно увидеть, что эти картинки состоят из
типографских символов (букв, скоробок, черточек и т.д.) Закрыта часть
слогана "May the force of typography be with you".

Источник:
http://www.bite.ca/bitedaily/2010/11/the-force-of-typography/

Автор:
???

Вопрос 16:
Заполните пропуски в стихотворении Петра Орлова, посвященном Богу:
   С неба хлынул дождь, как из ведра.
   По домам попрятались старушки.
   Мир - Твоя веселая [пропуск],
   Мы - Твои волшебные [пропуск].

Ответ:
Игра, игрушки.

Источник:
П.В. Орлов. Глазами души. - Киев, 2010. - С. 120.

Автор:
Максим Егоров

Вопрос 17:
[Ведущему: слова "Игрик" и "Игрок" читать с ударением на первый слог.]
   8 декабря 1982 года в Уфе по на улице Игрика прошла первая
нелегальная демонстрация в память об Игроке. Кого мы заменили на Игрика
и Игрока?

Ответ:
Ленин, Леннон.

Автор:
???

Вопрос 18:
Среднестатистический ИКС - это 35 литров воды, 20 килограммов углерода,
4 литра аммиака, 1,5 килограмма извести, 800 граммов фосфора, 250
граммов соли, 100 граммов селитры, 7,5 граммов фтора, 3 грамма кремния,
15 других элементов в малых количествах и 5 граммов ИГРЕКА. В среднем
ИГРЕКА из одного ИКСА должно хватить примерно на два ЗЕТА. Вспомнив
балладу XX века, назовите ЗЕТ.

Ответ:
Гвоздь.

Комментарий:
Тут дан химический состав человека. Соответственно, ИГРЕК - это железо,
которого в организме среднего человека около 5 граммов. А баллада - это
"Баллада о гвоздях" Тихонова.

Источник:
   1. Состав тела человека (конкретые данные взяты из аниме "Fullmetal
Alchemist").
   2. Н. Тихонов. Баллада о гвоздях.

Автор:
???

Вопрос 19:
Гюго обессмертил себя литературными произведениями, а Циолковский -
изобретениями. Но было у них кое-что общее - они оба умерли на...
Закончите предложение тремя словами.

Ответ:
"... улице имени себя".

Автор:
???

Вопрос 20:
(pic: 20121032.jpg)
   Что мы закрыли от вас на этой картинке?

Ответ:
Значок кубического корня.

Зачет:
Значок квадратного корня (так в оригинале, хотя кубический вернее).

Источник:
http://www.liveinternet.ru/users/neistorik/post194405570/

Автор:
???

Вопрос 21:
В этом известном вам с детства произведении слово "за" повторяется два,
потом три, потом четыре, потом пять, потом шесть раз. Что это за
произведение?

Ответ:
"Репка".

Комментарий:
Дедка за репку, бабка за дедку...

Источник:
Сказка "Репка".

Автор:
Максим Егоров

Вопрос 22:
Прослушайте отрывок из песни группы "Ундервуд":
   И время идет; небо тушит луну
   В пепельнице мостовой.
   И ТАКАЯ Ю ловит волну
   От ТАКОЙ Б.
   В сталинские времена отключать ТАКУЮ Ю в квартире без особых причин
было запрещено. Что мы заменили на Ю и Б?

Ответ:
Точка, запятая.

Источник:
   1. Песня группы "Ундервуд" "Это судьба".
   2. http://ru.wikipedia.org/wiki/Абонентская_радиоточка

Автор:
???

Вопрос 23:
(pic: 20121033.jpg)
   Вам была роздана шуточная картинка, найденная автором вопроса в
Интернете. Мы не спрашиваем, в какой стране происходят изображенные
события. Напишите название легендарного сингла, которое мы от вас
скрыли.

Ответ:
Anarchy In The UK.

Источник:
   1. http://hugleikurdagsson.tumblr.com/post/8778818671/anarchy-in-the-uk
   2. http://ru.wikipedia.org/wiki/Anarchy_in_the_U.K.

Автор:
???

Вопрос 24:
По российскому законодательству личная библиотека осужденного может
войти в состав конфискованного имущества. Но некоторые книги
конфискованными быть не могут. Укажите хотя бы два названия таких книг.

Ответ:
Библия, Коран.

Зачет:
По смыслу.

Комментарий:
Не конфискуются, поскольку считаются предметами культа.

Источник:
Документальный фильм "Поле чудес. "МММ" возвращается", эфир телеканала
"РТР-планета", 25.05.11 г.

Автор:
Владимир Крикунов

Тур:
25 тур. "Вкусно и хорошо" (Ивантеевка)

Вопрос 1:
По словам Зигмунда Фрейда, "... цивилизация зародилась в тот момент,
когда рассерженный человек бросил ЭТО вместо камня". В известных русских
пословицах ЭТО считается менее ценным, чем его противоположность, и
более неуловимым, чем некое существо. Назовите ЭТО.

Ответ:
Слово.

Комментарий:
"Слово - серебро, молчание - золото"; "Слово не воробей, вылетит - на
поймаешь". Это первый вопрос от нашей команды, а, как известно, "в
начале было слово".

Источник:
   1. http://www.vokrugsveta.ru/encyclopedia/index.php?title=Фрейд%2C_Зигмунд
   2. http://www.poskart.ru/molchanie-zoloto.html
   3. http://www.poskart.ru/slovo-ne-vorobei.html

Автор:
Андрей Распитин, Олег Басов

Вопрос 2:
Высокий помост для ораторов, желавших выступить на народном собрании в
Древнем Новгороде, назывался "степень ТАКАЯ". Какой город иногда
называют "ТАКИМ городом"?

Ответ:
Рим.

Комментарий:
ТАКАЯ - вечная; как известно, народное собрание в Новгороде называлось
"вече".

Источник:
   1. http://www.lomonosov-fund.ru/enc/ru/encyclopedia:0137985:article
   2. http://ru.wikipedia.org/wiki/Рим

Автор:
Елена Межнева

Вопрос 3:
[Ведущему: кавычки не читать.]
   Первое путешествие [пропуск] на корабле было совершено немногим более
чем за 1080 дней. Первыми результатами поиска автора вопроса в Интернете
по ключевым словам "[пропуск]" были ссылки на интересные факты из
"[Пропуск]", которых за 150 лет накопилось немало. Заполните пропуск
двумя словами.

Ответ:
Вокруг света.

Комментарий:
Имеется в виду не только путешествие вокруг света Магеллана-Элькано, но
и российский журнал; исчисление в днях может служить намеком на
экранизированный роман "Вокруг света за 80 дней".

Источник:
   1. http://ru.wikipedia.org/wiki/Магеллан,_Фернан
   2. http://ru.wikipedia.org/wiki/Элькано,_Хуан_Себастьян

Автор:
Олег Басов

Вопрос 4:
Дуплет.
   Внимание, в вопросах есть замены.
   1. Словами "АЛЬФА без АЛЬФЫ" именуют не только ТАКИЕ АЛЬФЫ, но и
финский праздник Юханнус, в ходе которого зажигается большой костер.
Какие два слова мы заменили словами "ТАКИЕ АЛЬФЫ"?
   2. По версии исследователей университета Миссури, один из "ДРУГИХ
ИКСОВ" наступил в США 19 мая 1780 года из-за сильных пожаров в Канаде.
Какие слова мы заменили словами "ДРУГИЕ ИКСЫ"?

Ответ:
   1. Белые ночи.
   2. Черные дни.

Источник:
   1. http://finland.ice-nut.ru/finland054.htm
   2. http://www.visitsuomi.com/sp/262/
   3. http://ru.wikipedia.org/wiki/Праздник_летнего_солнцестояния#.D0.A4.D0.B8.D0.BD.D0.BD.D1.8B:_.D0.AE.D1.85.D0.B0.D0.BD.D0.BD.D1.83.D1.81
   4. http://ru.wikipedia.org/wiki/Чёрный_день

Автор:
Олег Басов

Вопрос 5:
В конце XVII века Франц Яковлевич Лефорт ДЕЛАЛ ЭТО, когда хотел
объясниться с Петром I. До сих пор некоторые пользователи сотовых
телефонов ДЕЛАЮТ ЭТО, в том числе из-за отсутствия определенных
настроек. Что именно делают?

Ответ:
Пишут по-русски латинскими буквами.

Зачет:
Пишут транслитом.

Источник:
   1. Н. Павленко. Франц Лефорт (Серия "ЖЗЛ"). - М.: Молодая гвардия,
2009.
   2. http://forum.megafonmoscow.ru/?message=146293#newmsg
   3. Личный опыт автора.

Автор:
Анастасия Дёмкина

Вопрос 6:
В одной статье о Валерии Синельникове описаны его мысли о том, что для
людей становится необходимым согласовывать свою жизнь с природными
циклами. В названии этой статьи есть слова "[пропуск 1] жить [пропуск
2]". Пропуски отличаются друг от друга пробелом и написанием одной
буквы. Заполните оба пропуска.

Ответ:
"Пора...", "... по Ра".

Комментарий:
По словам Синельникова, людям надо организовывать свою жизнь в
соответствии с движением Солнца.

Источник:
http://www.rojdenierus.ru/forum/viewtopic.php?p=74993

Автор:
Елена Межнева

Вопрос 7:
Вопрос задает группа "Агата Кристи".
   У великого князя Михаила Павловича были довольно натянутые отношения
с супругой. Как-то князю задали вопрос о возможности празднования им
события, именуемого сейчас серебряной свадьбой. На этот вопрос Михаил
Павлович ответил: "Нет, любезный, я подожду еще пять лет и тогда
отпраздную годовщину моей...". Закончите ответ князя двумя словами.

Ответ:
"... тридцатилетней войны".

Комментарий:
Действительно, "я на тебе, как на войне". :-)

Источник:
   1. http://www.abhoc.com/arc_an/2005_04/298/
   2. http://www.amdm.ru/akkordi/agata_kristi/3610/kak_na_voine/

Автор:
Анастасия Дёмкина

Вопрос 8:
Ошибка переводчика с английского привела к появлению в одной из серий
мультфильма о Сонике слов, согласно которым Черный рыцарь "пожелтел".
Любопытно, но в сценических костюмах одного из известной тройки желтый
цвет практически отсутствует. Что же на самом деле сделал Черный рыцарь?

Ответ:
Струсил.

Комментарий:
Переводчики неверно перевели английское идиоматическое выражение.

Источник:
Личный опыт автора вопроса.

Автор:
Анастасия Дёмкина

Вопрос 9:
Ваш ответ на этот вопрос может привести к тому, что в 2130 году
чемпионом мира по "Что? Где? Когда?" станет сборная Мексики.
   Прослушайте цитату из отзыва на одну из книг Фаулза: "Для него
счастье заключается в коллекционировании красивых мертвых тел... ИХ".
Назовите ИХ одним словом.

Ответ:
Бабочки.

Комментарий:
В первом предложении вопроса обыгрывается так называемый "эффект
бабочки".

Источник:
http://fantlab.ru/work28519

Автор:
Олег Басов

Вопрос 10:
Современный рекламный слоган, выполненный в революционном стиле,
представляет собой двустишие, первая строка которого призывает: "Даешь
жилье матросам и штатским". На каком московском бульваре находится
недвижимость, которая рекламируется таким образом?

Ответ:
На Кронштадтском бульваре.

Комментарий:
Полностью реклама звучит так: "Даешь жилье матросам и штатским - /
Недвижимость на бульваре Кронштадтском".

Источник:
Реклама на канале "ТВЦ".

Автор:
Анастасия Дёмкина

Вопрос 11:
Слово "ТАКОЕ" используется, например, для описания армии Мухали сразу
после взятия штурмом города Цзянчжоу. Статья журнала "Вокруг света" под
названием "ТАКОЕ пространство" рассказывает о животных, расселившихся по
территории Чернобыльской зоны отчуждения. О каком пространстве идет
речь?

Ответ:
Об озверевшем.

Источник:
   1. http://ru.wikipedia.org/wiki/Монгольско-цзиньская_война
   2. http://www.vokrugsveta.ru/vs/article/7389/

Автор:
Елена Межнева

Вопрос 12:
[Ведущему: сделать паузу перед словом "время".]
   Внимание! В вопросе мы пропустили слово из пяти букв. Время.

Ответ:
Какое.

Источник:
Личный опыт автора вопроса.

Автор:
Олег Басов

Вопрос 13:
Среди образчиков черного юмора можно найти употребление этих двух слов,
например, по отношению к убитому взрывом бомбы генерал-губернатору
Москвы Сергею Александровичу и некоему немецкому профессору. Эти же два
слова употребили журналисты "Воронежского курьера" в названии статьи,
подзаголовок которой гласил: "Региональную "Стратегию 2020" обсуждают 18
групп экспертов". Воспроизведите эти два слова.

Ответ:
Пораскинуть мозгами.

Зачет:
Раскинуть мозгами.

Комментарий:
Немецкий профессор - Плейшнер.

Источник:
   1. http://www.1905.su/2009/08/
   2. http://www.vysokovskiy.ru/story/mozgami/
   3. http://www.abireg.ru/?idnews=18210&newscat=18

Автор:
Андрей Распитин

Вопрос 14:
В астрономии это заблуждение господствовало до 1977 года и было
опровергнуто исследованиями "Вояджера-1". Однако среди журналистов это
заблуждение по-прежнему в ходу. Так, в 2005 году благодаря этому
заблуждению журналист Дмитрий Гулютин охарактеризовал некий объект
Солнечной системы названием произведения, опубликованного в середине
1950-х годов. Воспроизведите это название.

Ответ:
"Властелин Колец".

Комментарий:
До 1977 года считалось, что Сатурн - единственная планета Солнечной
системы, обладающая кольцами; сейчас доказано наличие колец у Юпитера,
Урана и Нептуна.

Источник:
   1. http://ru.wikipedia.org/wiki/Кольца_планет
   2. http://www.vokrugsveta.ru/vs/article/570/
   3. http://ru.wikipedia.org/wiki/Властелин_колец

Автор:
Анастасия Дёмкина

Вопрос 15:
Родиной ПЕРВОЙ считают Китай, родиной ВТОРОГО - Южную Америку. Плоды
ПЕРВОЙ появляются на наших прилавках преимущественно зимой, а плоды
ВТОРОГО - преимущественно летом и осенью. Однако, несмотря на эти
различия, для описания плодов ПЕРВОЙ иногда используют сравнение с
плодами ВТОРОГО, а среди сортов ВТОРОГО есть сорт под названием
"ПЕРВАЯ". Как называются плоды ПЕРВОЙ и ВТОРОГО?

Ответ:
Хурма, помидоры.

Комментарий:
ВТОРОЙ - это томат, плоды которого именуются помидорами.

Источник:
   1. http://ru.wikipedia.org/wiki/Хурма
   2. http://ru.wikipedia.org/wiki/Томат
   3. http://www.dietgid.ru/main/zdorovoe-pitanie/poleznye-produkty/56-khurma-poleznaja-obmanshhica.html
   4. http://www.virasti-sam.com/ovosh_pomidor_sorta.php

Автор:
Елена Межнева

Вопрос 16:
Согласно стихотворной присказке шута Балакирева, в народе считали, будто
с третьей стороны Санкт-Петербурга был мох, а с четвертой стороны - это
короткое слово. Это же слово стало именем мультперсонажа-пессимиста,
который после раската грома жалуется: "Я от грохота оглох".
Воспроизведите это слово.

Ответ:
Ох.

Комментарий:
Непросто приходилось строителям новой столицы.

Источник:
   1. http://f5.ru/partyofblondes/post/377722
   2. http://mults.spb.ru/mults/?id=260

Автор:
Анастасия Дёмкина

Вопрос 17:
Недавно ученые подтвердили, что ОН у слонов представляет собой
сесамовидную кость, которая скрывается в жировой подошве слоновьей
ступни. С людьми всё сложнее - так, например, Анне Болейн, для того
чтобы скрыть ЕГО, пришлось ввести моду на длинные рукава. Назовите ЕГО
двумя словами.

Ответ:
Шестой палец.

Источник:
   1. http://lenta.ru/news/2011/12/23/sixth/
   2. http://www.vokrugsveta.ru/vs/article/6122/
   3. http://ru.wikipedia.org/wiki/Полидактилия

Автор:
Анастасия Дёмкина

Вопрос 18:
Внимание, в вопросе есть замены.
   В романе Достоевского "Бесы", в главе "У ТАКИХ" рассказывается о
деятельности подпольного политического кружка. В наше время, по мнению
блогера Dlindele, в связи со сложившейся в стране ситуацией вокруг
выборов, известный возглас, в котором упоминаются ТАКИЕ, скоро
приобретет новый, позитивный смысл. Какое слово мы заменили на "ТАКИЕ"?

Ответ:
Наши.

Комментарий:
Имеется в виду возглас "Наших бьют".

Источник:
   1. http://az.lib.ru/d/dostoewskij_f_m/text_0080.shtml
   2. http://f5.ru/narodnaya-mudrost/post/388115

Автор:
Анастасия Дёмкина

Вопрос 19:
Вопрос задает комэск Титаренко.
   Может сложиться впечатление, что обладание этим устройством негативно
сказывается на человеке. Так, для ведущей программы "Слабое звено" это
устройство стало причиной перелома пальца ноги. А в 2011 году некий
китаец из-за этого устройства лишился почки. Назовите это устройство.

Ответ:
iPad.

Зачет:
Интернет-планшет, планшетный компьютер.

Комментарий:
Комэск Титаренко в фильме "В бой идут одни "старики"" в ответ на вопрос
"Ты когда себе планшет заведешь?" заявил: "Так в бою сапог надежнее".
Ведущая сломала палец, уронив на него iPad, а семнадцатилетний китаец
ради покупки iPad продал свою почку.

Источник:
   1. http://lenta.ru/news/2011/03/14/kiseleva/
   2. http://www.baltinfo.ru/2011/06/04/V-Kitae-podrostok-prodal-pochku-chtoby-kupit-iPad-2-209464
   3. Х/ф "В бой идут одни "старики"".

Автор:
Анастасия Дёмкина

Вопрос 20:
Статья, посвященная становлению сафари как развлечения для богатых,
называется "ПАСТЕРНАКИ класса люкс". Слово "ПАСТЕРНАК" - это еще и
название фильма-экранизации, снятого в 1990 году. Какое слово мы
заменили словом "ПАСТЕРНАК"?

Ответ:
Зверобой.

Комментарий:
Пастернак - это не только поэт, но и растение, как и зверобой.

Источник:
   1. http://www.vokrugsveta.ru/vs/article/7560/
   2. http://ru.wikipedia.org/wiki/Зверобой_(фильм)

Автор:
Александр Митрюхин

Вопрос 21:
Однажды, заметив играющих детей, Владимир Маяковский попробовал принять
участие в их игре. Когда писателя на этом поймали серьезные люди, он
смутился и спросил: "Ну что, гожусь я в ИКСЫ?". Какое слово мы заменили
на ИКСЫ?

Ответ:
Классики.

Комментарий:
Дети играли в классики.

Источник:
В. Маяковский. Стихотворения и поэмы (Хрестоматия школьника). - М.:
Астрель, 2001.

Автор:
Анастасия Дёмкина

Вопрос 22:
В начале прошлого века ИХ представители сеяли страх среди членов
царского правительства. В декабре прошлого года ОНИ получили более 13%.
Назовите ИХ.

Ответ:
Эсеры.

Комментарий:
В первом случае имеются в виду социалисты-революционеры, чья боевая
организация совершила ряд известных терактов, а во втором -
справедливороссы, получившие на выборах в Думу 13,24% голосов, которых
также иногда именуют "эсеры".

Источник:
   1. http://ru.wikipedia.org/wiki/Партия_социалистов-революционеров
   2. http://ru.wikipedia.org/wiki/Справедливая_Россия

Автор:
Анастасия Дёмкина

Вопрос 23:
[Демонстрируется черный ящик. Ведущий открывает его, внутри оказывается
еще один, а внутри него третий (размером не меньше упаковки
"МанЧГКина"). В третьем черном ящике должна находиться иголка.]
   По словам Козьмы Пруткова, АЛЬФА является результатом самого первого
шага человека. АЛЬФА одного персонажа часто тесно связывается с тем, что
находится в последнем черном ящике. Назовите этого персонажа.

Ответ:
Кощей [Бессмертный].

Зачет:
Кащей [Бессмертный].

Комментарий:
Показанная цепочка ящиков представляет собой своеобразный аналог существ
и предметов, в которых хранилась смерть Кащея.

Источник:
   1. http://greatwords.ru/quote/448/
   2. Эдуард Успенский. Вниз по волшебной реке.
   3. Сказка "Царевна-лягушка".
   4. http://ru.wikipedia.org/wiki/Кощей

Автор:
Олег Басов

Вопрос 24:
Статья новостного сайта utro.ru о мерах, которые Медведев пообещал
принять в ответ на неудачные запуски ракет и спутников, называется:
"Медведев устроит [ПРОПУСК]". В пропуске в устойчивое словосочетание из
двух слов вставлено одно прилагательное. Восстановите пропуск.

Ответ:
"... разбор космических полетов".

Комментарий:
Тур закончен, можно начинать "разбор полетов".

Источник:
http://www.utro.ru/articles/2011/11/26/1013175.shtml

Автор:
Олег Басов

Тур:
26 тур. "ТРУпоморы" (Архангельск - Северодвинск - Коломна)

Вопрос 1:
Журнал "ИКС" издавался на украинском языке с 1927 года. Первый номер
журнала "ИГРЕК" вышел в Перми в 2010 году. "ИГРЕК и ИКС" - название
нескольких ресторанов в России, а встретить ИКС и ИГРЕК можно в
подавляющем большинстве мест общепита. Что мы заменили на ИКС и ИГРЕК?

Ответ:
Перец, соль.

Зачет:
В любом порядке.

Источник:
   1. http://uk.wikipedia.org/wiki/Перець_(журнал)
   2. http://www.saltt.ru/magazine
   3. http://www.google.ru/search?q=соль+перец

Автор:
Александр Цыганов (Архангельск)

Вопрос 2:
В клинике, где пытаются лечить болезни методом наложения проекций
произведений живописи, Зиновий Зиник предложил использовать Ван Гога для
лечения ушей, а ЕГО - для борьбы с прыщами. Если сравнивать с Ван Гогом,
ЕГО жизнь была будничной. Назовите ЕГО.

Ответ:
[Жорж-Пьер] Сёра.

Комментарий:
По сравнению с Ван Гогом его жизнь действительно была, как серые будни.
А лицо, усыпанное прыщами, - вполне себе пуантилизм.

Источник:
http://www.openspace.ru/literature/events/details/30465/

Автор:
Александр Цыганов (Архангельск)

Вопрос 3:
Герой Александра Иличевского видит в ЭТОМ СЛОВЕ горы, кольцо, полумесяц,
черного коня и детский крик. Напишите ЭТО СЛОВО, если оно - имя
собственное.

Ответ:
Москва.

Комментарий:
"Москва - рогатое слово, - ... "М" - это Воробьевы горы, пила
кремлевской стены. "О" - Садовое, Бульварное, Дорожное кольцо. "С" -
полумесяц речной излучины. "К" - трамплины лыжные, кремль, конь черный.
"Ва" - уа, уа, - детский крик, вава".

Источник:
А. Иличевский. Матисс. - М.: Время, 2008. - С. 301-302.

Автор:
Иван Нефёдов (Архангельск)

Вопрос 4:
[Ведущему: название автомобиля произнести максимально четко.]
   Как известно, почти каждая марка автомобиля имеет кроме логотипа еще
и слоган. В радиорекламе автомобиля Suzuki SX 4 звучит слоган, состоящий
из фразеологизма, в который добавлено две буквы. В одном из словарей
этому фразеологизму дается такое толкование: "Полностью располагать
собой; будучи независимым, свободным от каких-либо обязанностей,
поступать так, как хочется". Воспроизведите выражение, прозвучавшее в
рекламе.

Ответ:
На все SX 4 стороны.

Источник:
   1. ЛОАВ.
   2. http://phraseology.academic.ru/4913/

Автор:
Александра Головина (Архангельск)

Вопрос 5:
По мнению Карла Брюллова, на известном портрете изображен какой-то ИКС,
а не поэт. Согласно одной "энциклопедии", к ИКСАМ можно причислить и
товарища человека, изображенного на портрете. Согласно сведениям другой
энциклопедии, ИКС - это "семейный компьютер". Назовите ИКС.

Ответ:
Денди.

Комментарий:
Художник посчитал, что Пушкин на портрете Кипренского не похож на самого
себя, а является иллюстрацией образа "денди". Товарищ Пушкина, согласно
"энциклопедии русской жизни" "Евгений Онегин", был денди, а Википедия
(свободная энциклопедия) определяет "Денди" как семейный компьютер.

Источник:
   1. http://www.nearyou.ru/100kartin/100karrt_45.html
   2. А.С. Пушкин. Евгений Онегин.
   3. http://ru.wikipedia.org/wiki/Денди_(значения)

Автор:
Кирилл Титов (Архангельск)

Вопрос 6:
Внимание, в вопросе есть замена.
   Своим названием секта "Козявок" обязана характерным движениям,
совершаемым в такт одной из духовных песен, обозначаемым ими как
"духовная пляска". Другое объединение "козявок", известное многим из вас
с детства, после завоевательного похода стало заниматься волейболом.
Какое слово мы заменили на "Козявки"?

Ответ:
Прыгуны.

Комментарий:
Смысл замен - Прыгуны (лат. Longitarsus) - род жуков (Coleoptera) из
подсемейства козявок (Galerucinae) в семействе листоедов
(Chrysomelidae).

Источник:
   1. Серия "Энциклопедия преступлений и катастроф": тайные общества и
секты.
   2. http://ru.wikipedia.org/wiki/Волшебная_страна_(Волков)#.D0.9D.D0.B0.D1.80.D0.BE.D0.B4.D1.8B

Автор:
Кирилл Титов (Архангельск)

Вопрос 7:
[Ведущему: отточия игнорировать.]
   Цитата из автобиографии Агаты Кристи, относящаяся к детским годам
писательницы:
   - Самые плохие, по-моему, - это пики, - сказала я. <...> - Трефы
немного лучше. <...> А бубны, - я подумала немного, - а бубны - это
просто статисты, - заявила я безапелляционно <...> победным тоном.
   Какое слово мы заменили в цитате?

Ответ:
Викторианским.

Комментарий:
"Заявила я безапелляционно <...> викторианским тоном". Детство Агаты
Кристи проходило во времена королевы Виктории.

Источник:
http://lib.rus.ec/b/29690/read

Автор:
Валерия Комаровская (Коломна)

Вопрос 8:
Журнал "Мир фантастики" свой номер, посвященный подростковым книгам,
назвал "Поколение ЕГО". На самом деле представителям ЕГО поколение уже
за тридцать лет. Назовите ЕГО.

Ответ:
Гарри Поттер.

Источник:
"Мир фантастики", 2009, N 9.

Автор:
Александр Цыганов (Архангельск)

Вопрос 9:
Роман Каплан считает, что единственным примером ЭТОГО в России является
Колобок. Существует ресторан ЭТОГО под названием "Колобок". Мы не
спрашиваем, о чем идет речь. Ответьте, что было основано в 1940 году
двумя выходцами из Шотландии.

Ответ:
"Макдональдс".

Источник:
   1. http://www.novayagazeta.ru/data/2009/067/19.html
   2. http://kolobok.spb.ru/
   3. http://ru.wikipedia.org/wiki/McDonald%E2%80%99s

Автор:
Александр Цыганов (Архангельск)

Вопрос 10:
Перед зданием Калининградского государственного технического
университета расположена крайне реалистичная скульптурная группа
"Борющиеся зубры". Назовите праздник, в преддверии которого в целях
избегания вандализма и святотатства усиливают охрану памятника.

Ответ:
Пасха.

Комментарий:
Зубры изображены действительно реалистично. И существует студенческая
традиция красить им яйца.

Источник:
http://www.rudnikov.com/article.php?ELEMENT_ID=19416

Автор:
Александр Цыганов (Архангельск)

Вопрос 11:
На одной из фотографий девушка, высоко подняв руку, расстегивает молнию
на правом боку своего красного открытого платья. Мы не спрашиваем вас,
какая именно татуировка видна из-под молнии, хотя вы без труда ответили
бы на такой вопрос. Каким словом латинского происхождения названа эта
фотография?

Ответ:
Рекурсия.

Комментарий:
А там татуировка, изображающая, как девушка, высоко подняв руку,
расстегивает молнию на правом боку своего красного открытого платья. А
под молнией татуировка...

Источник:
   1. http://ibigdan.livejournal.com/8858241.html
   2. http://ru.wiktionary.org/wiki/рекурсия

Автор:
Елена Ганькова (Северодвинск)

Вопрос 12:
До середины 20-х годов прошлого века части Новосибирска, стоящие на
разных берегах, находились в разных ИХ. Героиня произведения Бориса
Рыжего уточняла у попутчика, сразу ли потемнеет при пересечении ЕГО.
Назовите ЕГО.

Ответ:
Часовой пояс.

Комментарий:
До середины 20-х годов прошлого века в городе Новосибирске было
одновременно два часовых пояса. Город стоит на реке Оби, по которой и
проходит часовой меридиан.

Источник:
   1. http://www.metronsk.ru/vopros/957
   2. http://magazines.russ.ru/znamia/1999/4/ryz.html

Автор:
Александр Цыганов (Архангельск)

Вопрос 13:
В течение одного дня марта 2009 года в магазине техники для дачи была
скидка на бензопилы, косы, ледорубы, мини-вилы и секаторы. Мы не просим
вас назвать того, кто был изображен на рекламе, сообщающей о скидке.
Назовите кинофраншизу, начавшуюся в 1980 году.

Ответ:
"Пятница 13".

Комментарий:
Всё это, да еще и со скидкой, вполне могло помочь Джейсону Вурхизу. А
вот фирменного мачете не продавали со скидкой. Не сезон, видимо.

Источник:
   1. http://ibigdan.livejournal.com/7613205.html
   2. http://ru.wikipedia.org/wiki/Джейсон_Вурхиз

Автор:
Александр Цыганов (Архангельск)

Вопрос 14:
Цитата: "Пошли седьмые сутки. Я смотрела в Книге рекордов Гиннесса.
Одиннадцать". Догадавшись, о каком рекорде говорит героиня, назовите
фильм 1984 года, в котором прозвучала эта реплика.

Ответ:
"Кошмар на улице Вязов".

Комментарий:
Рекорд бессонницы. Персонажи "Кошмара" боялись заснуть.

Источник:
Фильм "Кошмар на улице Вязов" (1984).

Автор:
Никита Коновалов (Коломна)

Вопрос 15:
Внимание, в вопросе есть замены.
   В одном из интервью за 2010 год актер Григорий Добрыгин два раза
упомянул ТРЕХ ИКСОВ. В первом случае ИКСЫ были живые, а во втором -
РАЗГОВОРЧИВЫЕ. Какое слово мы заменили на "РАЗГОВОРЧИВЫЕ"?

Ответ:
Серебряные.

Комментарий:
Речь идет о трех медведях, которых актер повстречал на съемках фильма
"Как я провел этим летом", а в феврале фильм получил трех серебряных
медведей на Берлинском кинофестивале.

Источник:
Журнал "Лиза", N 34, 14 августа 2010 г. - С. 18-19.

Автор:
Иван Нефёдов (Архангельск)

Вопрос 16:
В одном из комиксов "Nichtlustig" [нихтлюстиг] ("Несмешно") немецкого
автора Йоши Зауера почтальон заявляет собаке: "Я знаю, что вы всегда
хотели меня тяпнуть, но, как оказалось, я внезапно вас очень нежно
люблю". Реплика собаки содержит появившееся в первой половине 1970-х
годов словосочетание из двух слов, начинающихся на одну и ту же букву.
Напишите эти два слова.

Ответ:
Стокгольмский синдром.

Комментарий:
(pic: 20121034.jpg)
   23 августа 1973 года бежавший из тюрьмы Ян Эрик Улссон в одиночку
захватил банк "Kreditbanken" [кредитбанкен] в Стокгольме. После
освобождения бывшие заложники заявили, что боялись не захватчиков,
которые ничего плохого им не сделали, а полиции. Также, по некоторым
данным, они за свои деньги наняли адвокатов Улссону. Стокгольмский
синдром характеризуется взаимной или односторонней симпатией,
возникающей между жертвой и агрессором в процессе захвата и применения
(или угрозы применения) насилия.

Источник:
   1. http://ru-nichtlustig.livejournal.com/291752.html
   2. http://ru.wikipedia.org/wiki/Стокгольмский_синдром

Автор:
Александра Головина (Архангельск)

Вопрос 17:
Комментатор одного из хоккейных матчей сказал про игрока, споткнувшегося
на ровном месте, что тот запнулся за ЭТО. В Петербурге ЭТИМ называют
вторую. Назовите ЭТО двумя словами.

Ответ:
Синяя линия.

Источник:
   1. Трансляция хоккейного матча "Лада" - "Авангард" на канале "Спорт",
07.02.2010 г.
   2. http://ru.wikipedia.org/wiki/Московско-Петроградская_линия

Автор:
Дмитрий Мансуров (Москва)

Вопрос 18:
[Ведущему: кавычки в вопросе не озвучивать.]
   Александр Жолковский рассказывает о случае, произошедшем в 1965 году,
когда его знакомый в ресторане внезапно подошел к поднадоевшим
оркестрантам и дал им десятку. "Сыгранное" без особого труда
произведение оказалось вдвое дольше известного "сочинения". Назовите
автора этого "сочинения".

Ответ:
Джон Кейдж.

Комментарий:
Это были 10 минут тишины, по рублю за каждую минуту. В оригинале это
"4'33". Сочинение для вольного состава инструментов".

Источник:
http://magazines.russ.ru/novyi_mi/2009/7/zh8.html

Автор:
Иван Нефёдов (Архангельск)

Вопрос 19:
15 февраля 2010 года "Экспресс-газета online" сообщила, что на
киностудии "Мосфильм" отметили юбилей актрисы Елизаветы, звезды
отечественного кино, сыгравшей в фильмах "Граница. Таежный роман",
"След", "Нежный барс", "По ту сторону волков", "Тайга. Курс выживания" и
многих других. В офисе студии виновницу торжества поджидал роскошный
торт, а главным подарком ей стал большой медальон в виде сердца,
прикрепленный... К чему?

Ответ:
К ошейнику.

Комментарий:
Полное имя актрисы - Елизавета Метель из Волчьего лога Гордость
Мосфильма, она - собака хаски. На медальоне была надпись "Народная
волчица России", так как чаще всего собаке доводилось исполнять роль
волчицы.

Источник:
http://www.eg.ru/daily/cadr/17391/

Автор:
???

Вопрос 20:
В русском переводе одной из серий мультфильма "Family Guy" [фэмили гай]
Питер Гриффин грозит Крису Гриффину пультом с красной кнопкой и
произносит известную фразу из семи слов, лишь словом "взорву"
отличающуюся от оригинала. Напишите автора оригинальной фразы.

Ответ:
Гоголь.

Зачет:
Тарас Бульба.

Комментарий:
"Я тебя породил, я тебя и взорву". Упомянутые Гриффины - отец и сын.

Источник:
Мультсериал "Family Guy", s02e11 "A Picture Is Worth a 1,000 Bucks".

Автор:
Иван Нефёдов (Архангельск)

Вопрос 21:
[Ведущему: первые кавычки в вопросе не озвучивать.]
   В 1930-х годах два еврейских мальчика решили придумать новый "сплав",
который бы обладал качествами библейского Самсона и [цитата]
"уникальными свойствами ковкости, твердости и блеска". И уже в июне 1938
года жители США впервые увидели ЕГО. Назовите ЕГО.

Ответ:
Супермен.

Зачет:
Человек из стали, Последний сын Криптона, Человек завтрашнего дня
(Человек будущего), Первый Гражданин Метрополиса, Гроза преступников,
Большой синий, бойскаут, Си-Кей.

Комментарий:
Называют же Супермена человеком из стали, а чем сталь не сплав?

Источник:
   1. М. Чабон. Приключения Кавалера и Клея. - СПб.: Амфора, 2006. - С.
97.
   2. http://www.lookatme.ru/flows/komiksyi/posts/67350-superman

Автор:
Иван Нефёдов (Архангельск)

Вопрос 22:
Блиц.
   Автор Живого Журнала "Блог говорящей шляпы" предположил, как бы
выдающиеся люди ответили на вопрос: "Зачем Герасим утопил свою Муму?". К
примеру, ответ "Это закономерный результат длительного подавления
скрытых сексуальных фантазий" якобы дал Зигмунд Фрейд, а ответ "Я верю,
что настанет день, когда никто больше не сможет утопить Муму" - Мартин
Лютер Кинг. Назовите того, кому принадлежит следующий ответ на вопрос
"Зачем Герасим утопил свою Муму?":
   1. "Чтобы лишний раз подтвердить мой закон".
   2. "Чтобы опроститься и стать ближе к народу".
   3. "Чтобы получить изысканное наслаждение последними минутами ее
жизни и, конечно, этими восхитительными бульканьями за бортом".

Ответ:
   1. Архимед.
   2. Лев Толстой.
   3. Маркиз де Сад.

Источник:
http://ttalkinghat.livejournal.com/51759.html

Автор:
Анна Дуплищева (Северодвинск)

Вопрос 23:
Однажды художнику Левитану нездоровилось, и ученики послали ему шутливое
письмо. В нем было написано, что даже ОНИ соскучились и беспрерывно
кричат: "Где Левитан, где Исаак Ильич!?". Левитана это письмо
развеселило и порадовало: "Передайте ИМ, что как только встану - приеду.
А если будут очень надоедать, попугайте: не только приеду, но и ружье
привезу". И хотя в этом вопросе нет ожидаемой замены, назовите ИХ.

Ответ:
Грачи.

Комментарий:
С грачами более всего ассоциируется другой художник - Саврасов - автор
хрестоматийной картины "Грачи прилетели".

Источник:
С. Пророкова. Левитан. - М.: Молодая гвардия, 1960. - С. 197.

Автор:
Елена Ганькова (Северодвинск)

Вопрос 24:
Внимание, в вопросе есть замена.
   Герой одного из современных телесериалов с сожалением говорит о том,
что в детстве у него из ДЕРЕВЯННЫХ ИГРУШЕК были только шахматные кони. А
медики считают, что присутствие в доме ДЕРЕВЯННЫХ ИГРУШЕК снимает
напряженность, снижает стрессы, способствует нормализации давления и
помогает выздоровлению после операций. Какое словосочетание мы заменили
словами "ДЕРЕВЯННЫЕ ИГРУШКИ"?

Ответ:
Домашние животные.

Источник:
   1. Телесериал "Папины дочки", 26.02.2010 г.
   2. http://www.detstvo.ru/psychology/pets.html

Автор:
Анна Дуплищева (Северодвинск)

Тур:
27 тур. "ЗаПущинная наука" (Пущино - Серпухов)

Редактор:
Евгений Кононенко (Москва)

Вопрос 1:
Героиня "Кентервильского привидения" жаловалась, что, после того как у
нее утащили все краски, оставив лишь непригодные для восстановления
злополучного пятна индиго и белила, ей пришлось рисовать только
навевающие тоску ИХ. ИХ можно наблюдать, например, в Исландии, Намибии и
Турции. Назовите ИХ.

Ответ:
Лунные пейзажи.

Комментарий:
"Лунными пейзажами" называются лишенные растительности ландшафты. Красок
для изображения растительности у героини не осталось.

Источник:
   1. О. Уайльд. Кентервильское привидение.
   2. http://ru.wikipedia.org/wiki/Лунный_пейзаж

Вопрос 2:
Блиц.
   1. Герой Пелевина придумывает новый смысл словам иностранного
происхождения. Так, например, синоптик для него - это составитель
синопсисов. Кого он определил как барственную даму, пропахшую дешевыми
сигаретами?
   2. Кого тот же герой Пелевина определил как любящего Родину крупного
монополиста?
   3. Кого тот же герой Пелевина определил как ненавистника скандальной
телеведущей?

Ответ:
   1. Примадонна.
   2. Патриарх.
   3. Ксенофоб.

Комментарий:
   1. Донна, курящая "Приму".
   2. Патриотически настроенный олигарх.
   3. Ненавидящий Ксению Собчак.

Источник:
В. Пелевин. Empire V.

Вопрос 3:
Цитата из произведения Виктора Пелевина "Empire V": "ИКС во все времена
был для вампиров тем же, чем сакура для японцев, - символом красоты,
совершенной в своей мимолетности". Кого мы заменили на ИКС?

Ответ:
Комар.

Комментарий:
Тоже кровосос...

Источник:
В. Пелевин. Empire V.

Вопрос 4:
Павич в произведении "Вывернутая перчатка" образно описывал ИХ как
"дьяволов с крыльями летучих мышей, что держат перед собой в когтях
собственную мужскую или женскую душу, обучая ее двигаться по кривым
невидимым траекториям". Назовите ИХ.

Ответ:
Вывернутые ветром зонтики.

Зачет:
Зонтики на ветру.

Источник:
М. Павич. Вывернутая перчатка.

Вопрос 5:
По данным сайта bash.org.ru [башорг ру], "системному администратору
городского морга, по ошибке удалившему всю базу данных за несколько
десятков лет, вот уже пятый день снится [ПРОПУСК]". Заполните пропуск
фамилией.

Ответ:
Гоголь.

Источник:
http://bash.org.ru/quote/409966

Вопрос 6:
(pic: 20121035.jpg)
   Для данной продукции предложен остроумный слоган. Напишите закрытую
от вас идиому.

Ответ:
Накликать беду.

Источник:
http://demotivators.to/p/360513/chto-b-ne-naklikat-bedu.htm

Вопрос 7:
(pic: 20121036.jpg)
   Автор вопроса предлагает поместить данную картинку в качестве
иллюстрации к статье Викисловаря, описывающей объясняемое слово как
"существительное среднего рода, несклоняемое, тип склонения "ноль" по
классификации Зализняка, в значении сказуемого выражает крайнюю степень
умиления". Напишите название этой статьи.

Ответ:
Мимими.

Комментарий:
Жаргонный мем, ставший популярным после мультика "Мадагаскар".

Источник:
http://ru.wiktionary.org/wiki/мимими

Вопрос 8:
(pic: 20121037.jpg)
   Данную иллюстрацию автор вопроса увидел на шуточной обложке
произведения русского классика 1869 года. Назовите это произведение.

Ответ:
"Обрыв".

Источник:
http://pikabu.ru/view/novyie_oblozhki_dlya_staryikh_knig_278314

Вопрос 9:
Внимание, в вопросе слова "ПРОПУСК" и "ПРОПУСКАТЬ" являются заменами.
   "Вы меня своими вопросами загоняете в ПРОПУСК". Напишите слово,
замененное на "ПРОПУСК", не забывая, что ПРОПУСКАТЬ нельзя.

Ответ:
Google.

Зачет:
Гугл.

Комментарий:
Игрокам запрещено пользоваться поисковыми системами, в частности
"гуглить".

Источник:
http://www.nextjoke.net/Joke/1273

Вопрос 10:
Узнав, что в испанской версии фильма знаменитая фраза была частично
изменена для сохранения экзотичности, редактор вопроса заметил, что
правильнее было бы перевести ее как "Sayoonara, nene" [саёнАра нЕне].
Напишите четыре слова, звучавшие в оригинале.

Ответ:
Hasta la vista, baby!

Комментарий:
В английском оригинале используется испанский язык, поэтому в испанской
версии его заменили на японский - "Саёнара, бэби". Слово же "детка"
должно остаться на том же языке, на котором всё остальное время говорят
персонажи "Терминатора".

Источник:
http://www.kinokopilka.tv/forum_topics/15007

Вопрос 11:
Видимо, во избежание тавтологии последний раз это имя использовалось на
высшем уровне в XVI веке. Напишите символ, стоящий рядом с именем
последнего его официального обладателя.

Ответ:
V.

Комментарий:
Последний папа римский с именем Сикст - Сикст V (1585-1590).
Согласитесь, Сикст VI - это тавтология.

Источник:
http://ru.wikipedia.org/wiki/Сикст_V

Вопрос 12:
Дмитрий Рогозин в ходе своего прощального приема в Брюсселе посадил в
саду представительской виллы российской миссии при Североатлантическом
альянсе два саженца. Какого рода?

Ответ:
Тополь.

Комментарий:
Назвав их в честь баллистических ракет "Тополь" и "Тополь-М". Тополь -
род растений семейства ивовых, насчитывающий до 35 видов.

Источник:
http://www.govoritmoskva.ru/news.php?id=78732

Автор:
Данила Аладин

Вопрос 13:
   <раздатка>
   A LION WOULD NEVER CHEAT ON HIS MATE, BUT A
   </раздатка>
   "Лев никогда не обманывает своего напарника, но...". Закончите фразу
двумя словами, составляющими имя собственное.

Ответ:
Тайгер Вудс.

Зачет:
Tiger Woods; Tiger Wood.

Комментарий:
Игра слов: "WOOD" и "WOULD" звучат практически одинаково. Знаменитый
гольфист Элдрик Тонт Вудс по прозвищу "Тайгер" славится своей
изобретательностью в игре.

Источник:
http://www.deviantart.com/art/Tiger-Woods-Nike-Ad-169810009

Вопрос 14:
В мультфильме "Энеида", снятом по мотивам сатирической поэмы
Котляревского, Зевс посылает сообщение, на котором ставит пометку -
слово, написанное через дефис. Воспроизведите эту пометку.

Ответ:
Телеграмма-молния.

Источник:
Мультфильм "Энеида" (1991).

Вопрос 15:
В фильме "Куб" в системе из множества комнат заперты Квентин, Холлоуэй,
Ренн, Алдерсон и другие. В русском варианте героями могли бы стать
Владимир, Медведь, Коровьев и однофамилец уроженца Женевы. Назовите
этого уроженца.

Ответ:
[Франц Яковлевич] Лефорт.

Комментарий:
Имена героев фильма "Куб" образованы от названий известных тюрем. Чтобы
они и в русском варианте были "говорящими", следовало бы вспомнить
Владимирский централ, СИЗО "Медведь", ярославские Коровники и, вероятно,
Лефортово.

Источник:
http://ru.wikipedia.org/wiki/Куб_(фильм)

Вопрос 16:
Летнему кафе, находящемуся в середине Пентагона, сотрудники Министерства
обороны США еще во времена Холодной войны дали
остроумно-пессимистическое название. Напишите это название одним словом
на русском языке или двумя на английском.

Ответ:
Эпицентр.

Зачет:
Ground Zero.

Комментарий:
Когда в 50-е на Пентагон были нацелены десятки ядерных боеголовок,
предполагали, что бомба попадет именно туда.

Источник:
http://en.wikipedia.org/wiki/The_Pentagon

Вопрос 17:
Шутка: "Трагическая рекурсия: Оле-Лукойе приснился сам себе". Какие имя
и фамилию мы заменили на Оле-Лукойе?

Ответ:
Фредди Крюгер.

Источник:
http://www.bibo.kz/kipa/438212-tragicheskaja-rekursija-freddi-krjuger-prisnilsja.html

Вопрос 18:
Следуя платоновской традиции, Карл Поппер одну из глав своей книги
оформил в виде бесед с Сократом. В одной беседе Поппер упоминает
математиков Геделя, Тарского, Греллинга. Далее следует диалог:
   Сократ: "Кто все эти математики? Федр никогда не упоминал их имен".
   Поппер: "Пингвины, Сократ. Однако очень способные".
   Какое слово мы заменили на "пингвины"?

Ответ:
Варвары.

Комментарий:
Понятно, что ни один из этих математиков эллином не был. Замена
обусловлена романом Франса "Остров пингвинов".

Источник:
К. Поппер. Предположения и опровержения.

Вопрос 19:
Если верить Википедии, первые открытые соревнования по ЭТОМУ ВИДУ СПОРТА
состоялись в городе Пекос, Техас, в 1883 году. В Зале славы этого вида
спорта увековечены не только выдающиеся спортсмены. На рисунке Павла
Кучиньского наиболее узнаваемый эпизод соревнований помещен в нижней
точке изломанного финансового графика. Назовите этот вид спорта.

Ответ:
Родео.

Комментарий:
График изображает падение акций на бирже, в нижней точке которого,
обыгрывая биржевую лексику, изображены скачки на быке. В Зале славы
родео записываются также знаменитые животные.

Источник:
   1. http://ru.wikipedia.org/wiki/Родео
   2. https://www.facebook.com/pages/Paw%C5%82a-Kuczy%C5%84skiego/158506094232917

Вопрос 20:
Гуляя по Саранску, автор вопроса увидел памятник А.С. Пушкину и, как и
многие до него, поздоровался с ним, после чего фамильярно применил к
памятнику широко известное прозвище, изменив в нем лишь одну букву.
Напишите предложенное автором название памятника.

Ответ:
Санька Золотая Ручка.

Комментарий:
Рука памятника от частых рукопожатий стала блестеть и золотиться.

Источник:
ЛНА.

Вопрос 21:
Описывая в своем блоге футбольный матч "Крылья Советов" - "Динамо",
проходивший в Самаре на залитом дождем поле, Василий Уткин назвал
трудную победу хозяев ТАКОЙ. Какой - ТАКОЙ?

Ответ:
Вымоченной.

Комментарий:
Вымученная и мокрая.

Источник:
http://www.sports.ru/tribuna/blogs/utkin/238731.html

Вопрос 22:
Герой телесериала "Как я встретил вашу маму" задумал в День благодарения
запечь одну индейку внутри другой, и даже придумал этому блюду
четырехсложное название. Мы не знаем, напоминал ли вкус этого блюда
голубятину. Напишите придуманный героем английский неологизм.

Ответ:
Turturkeykey.

Комментарий:
Название отражает суть блюда - одна индейка (turkey) внутри другой.
Turtur - латинское родовое название горлицы.

Источник:
   1. Телесериал "Как я встретил вашу маму", s06e10.
   2. http://ru.wikipedia.org/wiki/Лесные_горлицы

Вопрос 23:
Размышляя о том, как брать вопросы под редактурой Артема Колесова, автор
вопроса проводит аналогию с известным устойчивым выражением. И
предлагает в таких случаях пользоваться не "бритвой Оккама", а другим
инструментом отбора. Назовите ЕГО двумя словами.

Ответ:
Меч Колесова.

Комментарий:
Меч Колесова - специальная лопата для высаживания саженцев. Известно
также выражение "смеяться после слова "лопата"".

Источник:
   1. ЛНА.
   2. http://ru.wikipedia.org/wiki/Меч_Колесова

Вопрос 24:
Прочитав книгу Владислава Крапивина "Дети синего фламинго", сын автора
вопроса заявил, что теперь знает, откуда пошло выражение, которым папа
его будит по утрам. Назовите место, где спали герои Крапивина.

Ответ:
Пушка.

Комментарий:
Из пушки не поднять.

Источник:
ЛНА.


FreeBSD-CVSweb <freebsd-cvsweb@FreeBSD.org>